You are on page 1of 317

[MJDF Study Library]

[PasTest Online MCQs]


[With Explanatory Answers]

i
ad
af
-S
Al
in
ss

[2014]
Ya

Yassin Al-Safadi (Safadi92@hotmail.com)


[Ethics and
i
Law] ad
af
-S
Al

[With Answers]
in
ss
Ya

Yassin Al-Safadi (Safadi92@hotmail.com)


1-Which one of the following best describes the Bolam test?

a) It assesses whether a patient can take in, retain and weigh information in coming to a
decision
b) It is regarding establishing the competency of children to make decisions regarding their
own health without parental involvement
c) It regards breaking confidentiality when deemed in the public best interests
d) It is one component of four in assessing whether there has been a breech in duty of care
« CORRECT ANSWER
e) It regards end-of-life decisions

2-The Bolam principle, based on English Tort law (Bolam vs Friern Hospital Management
Committee [1957]) has been used in determining whether acts by a doctor/nurse or other
health professional are in accordance with the accepted practice in these largely self-
regulating fields. It is based on the case of R vs Bodkin Adams (1957) in which Dr Adams was

i
ad
on trial for murdering a patient who had made him a beneficiary of her will. Dr Adams had
prescribed this 80-year-old patient heroin and morphine following a stroke. Despite that this
af
was not the first patient to whom he had been a beneficiary of their estate, the judge found
him not guilty. From the following options choose the reason for this?
-S
a) There was insufficient evidence
Al

b) The evidence was tainted


c) The medication was intended for pain relief, not to shorten life « CORRECT ANSWER
d) The doctor was euthanising a sick patient, which is acceptable under law
in

e) Not prescribing these medications for her condition would have been negligent
ss
Ya

3-Fraser guidelines include all of the following except?

a) A child should understand the proposed treatment/advice


b) The child cannot be persuaded to inform their parents
c) The child is likely to engage in, or continue having, sexual intercourse with or without
contraceptives
d) Unless the child receives contraceptives they are likely to suffer harm (either mental or
physical)
e) The child can refuse treatment sanctioned by their parents as they determine their own
best interests « CORRECT ANSWER

Yassin Al-Safadi (Safadi92@hotmail.com)


4-You have been approached by a pharmaceutical company to attend a focus group for a
currently available analgesic that is potentially having its license extended into treating
atypical facial pain. Which one of the following phase of clinical trial is this?

a) Phase 0
b) Phase I
c) Phase II
d) Phase III
e) Phase IV « CORRECT ANSWER

• Comment on this Question

The lifecycle of a drug starts long before it reaches the market and often development also
continues after this time.

• Phase 0 – microdosing in human studies. Gives no information about final effective dose,
but intended to expedite development by showing if the drug has desired in vivo behaviour

i
before proceeding to more expensive phases of trials with larger numbers of participants.

ad
• Phase I – Pharmacokinetics and pharmacodynamic studies in healthy volunteers. Small
number of participants (tens). af
-S
• Phase II – Aimed to demonstrate efficacy of new agent. Use real patients with the
disease. Compared against existing drug in hundreds of participants
Al

• Phase III – Double blinded and multi-centre trials conducted in thousands of participants.
Demonstrates new drug is superior to current alternatives.
in

• Phase IV – Post marketing surveillance to identify interactions, or may be undertaken to


ss

extend into new patient group/extend existing patents.


Ya

5-A patient tells you in confidence that they have HIV and they do not want anyone informed.
Which one of the following would allow you to break confidentiality?

a) When informing other members of your dental practice in conversation


b) Informing the GP in a letter
c) When you have determined it is in the patient’s best interests despite their wishes
d) When there is a case in the public interest or someone is at risk of serious harm «
CORRECT ANSWER
e) There is never any case for breaking confidentiality

Comment on this Question

Yassin Al-Safadi (Safadi92@hotmail.com)


This situation is permissible by the GDC. They advise that you should try to get patient consent
wherever possible and inform them before the breach. You should be prepared to defend your
decision. The third option would also be correct only if the patient was at risk of serious harm
from himself or herself.

6-A relative of a patient who has recently died has asked you to discuss matters from your
patient’s records. Which one of the following best describes your duty of confidentiality in this
scenario

a) Your duty of confidentiality is still binding to your deceased patient « CORRECT ANSWER
b) Your duty of confidentiality no longer applies as the patient is deceased
c) You are allowed to discuss confidential matters with relatives
d) You should get permission from the GDC to discuss these issues
e) You should contact your indemnifier and obtain permission

i
Comment on this Question

ad
The same caveats apply to that of living patients, ie breach in the public interest or if somebody
is at risk of serious harm. af
-S
http://www.gdc-uk.org/NR/rdonlyres/FFD61DA5-A09E-4B38-
8FFBBA342E9F0AF4/16689/147164_Patient_Conf.pdf
Al
in

7-A patient informs you that they wish to make a complaint regarding their treatment. Which
one of the following is level one of the NHS complaints procedure?
ss

a) Local resolution « CORRECT ANSWER


Ya

b) Independent review
c) Referral to the ombudsman
d) Involvement of GDC
e) Letter of acknowledgement

Comment on this Question

Local resolution is the first and best step in complaint resolution. Most complaints, if possible,
should be dealt with at this level. This involves, meetings between the complainant and the
health professional involved, facilitated by designated complaints manager (ie practice
manager).

Yassin Al-Safadi (Safadi92@hotmail.com)


8-If complainant is not satisfied with resolution attempted at local/primary care trust level
then which one of the following might be the next step in the complaints procedure?

a) Write to your local MP


b) Involve the media
c) Involve the ombudsman (health service commissioner) « CORRECT ANSWER
d) Re-attempt local resolution
e) Contact the GDC dental complaints service

Comment on this Question

The ombudsman is a civil servant who works independently of the NHS. They report to
Parliament about how the NHS functions and it is their decision to further investigate any
complaints they deem suitable for escalation.

9-Which one of the following personal health conditions might you be required to notify the

i
ad
GDC about?

a) Angina af
b) Asthma
-S
c) Diabetes
d) HIV « CORRECT ANSWER
Al

e) Psoriasis

Comment on this Question


in

If you have or believe a colleague has a serious communicable disease that places patients at
ss

risk, it is recommended that you inform the appropriate occupational health department/GDC.
While maintaining your own confidentiality is important, the overriding concern is patient safety
Ya

- as Hippocrates said ‘First, do no harm’.

10-An elderly confused lady with known Alzheimer’s disease is unable to understand the
details of a root canal procedure. You deem her to be incompetent to consent for this
procedure. Which one of the following could consent on her behalf?

a) Her son
b) Her husband
c) Her solicitor
d) Nobody unless they have enduring power of attorney « CORRECT ANSWER
e) You, as her dentist

Yassin Al-Safadi (Safadi92@hotmail.com)


Comment on this Question

Nobody can consent for an incompetent patient, although doctors/dentists may act in patients’
best interests. The new Mental Capacity Act (2007) has established that a competent patient
may nominate an ‘enduring power of attorney’ to make medical decisions on their behalf should
they become
incompetent.http://www.direct.gov.uk/en/DisabledPeople/HealthAndSupport/YourRightsInHeal
th/DG_10016888

11-Which one of the following is not an element of valid consent?

a) Being informed
b) Being competent
c) Lack of coercion
d) Autonomy in making a decision

i
e) Decision is irreversible once the consent form is signed « CORRECT ANSWER

ad
Comment on this Question
af
This statement is not true. The other four components are essential parts of valid consent for any
-S
medical procedure/investigation. A patient may withdraw consent without penalty at any time
before the procedure.
Al
in

12-You are the SHO covering the routine oral surgery list, the patient has been anaesthetised
and is on the operating table. During the time-out check, it is noticed that the patient, who
ss

does not speak any English, has not signed the consent form. Which one of the following is the
appropriate course of action?
Ya

a) Continue with the operation


b) Have the patient’s husband sign the consent form
c) Ask the patient to sign the consent form after the operation
d) Do not proceed, wake the patient up « CORRECT ANSWER
e) Sign the consent form yourself on the patient’s behalf

Comment on this Question

There are few defences for carrying out surgery without prior consent, which, except in the
context of an emergency and a patient unable to consent, is considered battery (see Mill’s harm
principle). Nobody is able to consent on behalf of a competent patient. The law is different in
the case of non-competent patients, but capacity to consent must be considered in all
circumstances. In this case, consider that a non-English speaking patient will not necessarily

Yassin Al-Safadi (Safadi92@hotmail.com)


understand what they are being consented for – therefore, a consent form is not valid – signed
or not.

13-A colleague has a bout of gastroenteritis on the same day that he is due to present an oral
presentation at a national conference. You agree to give the talk on his behalf, but you notice
that there are multiple facial photographs of a patient with ‘blanked out eyes’, which are
fundamental to the talk. You enquire whether your colleague has obtained the patient’s
consent to use these photographs, and he admits that he forgot to ask them for this.

What is the correct course of action?

a) As the eyes are blanked out and it is therefore more difficult to recognise the patient,
you give the presentation, as you are worried that cancelling the talk at this late stage
will appear unprofessional.
b) You apologise to the organisers, explain that you cannot give the talk, and ask whether

i
you can have a slot on a different day, to give you time to obtain consent from the

ad
patient in the photographs.« CORRECT ANSWER
c) You ask the conference IT technician what you should do, and he tells you that it is too
af
late to find another speaker and that you should make your way to the lecture theatre.
-S
d) You give the lecture and afterwards phone the General Dental Council to obtain their
advice on how to handle such a situation in future presentations.
Al

e) You speak to your colleague again. He says that the patient in the photograph was very
compliant with all her treatment, and he is sure that she will not mind the picture being
used for educational purposes.
in
ss

Comment on this Question

The General Dental Council (GDC) has clear guidelines on the use of sensitive patient data. It is
Ya

not acceptable to presume that consent would be given, and blanking out the patient’s eyes in
the photographs will not prevent her from being identified. There is an ethical responsibility to
ensure that informed patient consent has been obtained and documented. In this scenario,
breaching this and then later asking the GDC for advice would be indefensible. Although it is your
colleague’s presentation, it is your responsibility to ensure that you are acting appropriately.

15-Gillick competence is best described as:

a) A child under 18 years of age who attends the GP surgery/family planning clinic because
they wish to access a termination of pregnancy will need to be accompanied by an
adult.

Yassin Al-Safadi (Safadi92@hotmail.com)


b) A child under the age of 16 years can give valid consent without parental knowledge
provided that certain defined criteria are met, which include the child having capacity to
give consent.« CORRECT ANSWER
c) A dentist can accept consent from a child under 16 years of age to remove five of her
teeth, even if the dentist does not feel that this is in the child’s best interest.
d) All children under 16 years of age are able to access contraception without informing
their parents or guardian.
e) Parents should not be informed if their child is taking the contraceptive pill, as the
healthcare professional has to respect the confidentiality of the child under any
circumstances.

Comment on this Question

The accepted age of consent is 16 years. However, Gillick competence specifies that valid consent
may be taken from a child under 16 years of age, without parental agreement, if the treatment is
in the best interest of the child and the child has capacity to give consent. It is of paramount
importance that the child is encouraged to inform their parents or guardian about the decision,

i
ad
as this will result in safer care of the child.

16-You have been asked to attend the maxillofacial hospital ward round to learn more about
af
hospital-based dentistry. The registrar is bleeped to an emergency, and asks you to consent
-S
Mrs Davis for her imminent surgery in his absence. She is due for an operation to have part of
her tongue removed secondary to a squamous-cell carcinoma. You have never seen this
Al

operation performed, but understand some of the general complications of surgical


procedures. The registrar has left and the nurse is asking you to complete the consent form as
in

soon as possible so that Mrs Davis’ surgery is not delayed.


ss

What is the best course of action in this scenario?


Ya

a) You apologise to the nurse for any disruption and explain that you are not able to take
consent from a patient for a procedure that you have never seen performed and do not
fully understand.« CORRECT ANSWER
b) You bleep the registrar while he is dealing with the emergency and tell him that you are
unhappy about doing as he has requested.
c) You ensure that the surgery goes ahead as planned, as requested by your senior
colleague.
d) You inform the patient of the general complications that might arise from surgery, and
tell her that the surgeon who will be operating will reiterate them when she gets to
theatre. You then obtain her signature.
e) You tell the patient that you need to wait for a more senior doctor to go through the
consent form with her, but obtain her signature anyway as she says that she does not
want to know any of the risks of the procedure, as this information will make her too
anxious.

Yassin Al-Safadi (Safadi92@hotmail.com)


Comment on this Question

It is irresponsible and indefensible to take consent from a patient when you do not fully
understand the procedure. It is ultimately the responsibility of the person who will be performing
the procedure, who is often the best person to take informed consent. Even though some
patients would rather not know about the risks, it is good practice to explain these briefly to
them, as you will regret not doing so if the patient later decides to make a complaint. The
consent process may not be further completed when the patient gets to theatre, and a signature
on the form may result in other healthcare professionals failing to discuss the issue again.

17-You explain to your patient that a root canal and crown are needed to stabilise her
decayed tooth. She says that the tooth is not painful and that she would prefer to wait for a
year or two before this procedure is performed. Your patient is 15 years old and asks you not
to tell her mother, who is in the waiting room, as it might ‘upset her’.

What is the best course of action?

i
a) You ask the patient to wait while you discuss the situation with an experienced

ad
colleague. « CORRECT ANSWER
b) You explain that ideally the tooth needs work now, and you ask the patient why she is
af
reluctant to give her consent. If she still declines to give it, you accept this and arrange
-S
an appointment for her in 6 months’ time.
c) You explain to the patient that she is not able to give or refuse consent for treatment as
Al

she is only 15 years old.


d) You invite the patient’s mother in to discuss the topic immediately.
e) You reassure the patient that you will not discuss the matter with her mother, but
in

emphasise to her that you are not happy about leaving the work for a year or two.
ss

Comment on this Question


Ya

Although some parts of the other answers are correct, none of them are entirely so, and seeking
further advice is the most sensible action. Refusal of recommended treatment is a difficult
decision for a 15-year-old patient. It is possible for her to be competent (according to the Gillick
criteria), but it would be advisable to try to understand her reasons for refusal and to encourage
her to agree to the involvement of her mother. If the daughter refuses to give permission for her
mother to be involved, the situation becomes more difficult, and you should maintain
confidentiality with regard to your patient, but discuss the situation with an experienced
colleague before deciding what to do next.

18-You overhear the secretaries discussing the fact that the dentist at a nearby small practice
has recently been subject to investigation for negligence. One of your patients also hears this
and asks your opinion about it, as her husband is thinking of joining that practice.

What should you do?

Yassin Al-Safadi (Safadi92@hotmail.com)


a) Ask the patient to wait while you go and question the secretaries about what they
know, so that you can make an informed decision.
b) Call the nearby practice from a different room and ask what has been happening,
explaining that you have a concerned patient.
c) Explain that you think that the dentist in question may have made some mistakes
recently and that it would be unsafe for your patient’s husband to join that practice.
d) Politely inform your patient that it would be wrong for you to comment on this, given
the limited information that is available, but tell her that she is entitled to speak to the
practice concerned if she wishes to obtain further information.« CORRECT ANSWER
e) Try not to become involved, and dismiss the patient’s worries swiftly and curtly.

Comment on this Question

This is a difficult situation, and ultimately it concerns the professionalism of the reception staff,
who should refrain from making such allegations. In view of the very limited information
available, it is important for you to remain non-judgemental and at all times polite towards the
patient. She is asking for your professional advice, so suggesting another way for her to access

i
ad
information is helpful to her. Investigations into alleged negligence do occur, but such claims
may turn out to be false accusations, so you should remain impartial until the appropriate bodies
have concluded their investigations.
af
-S
19-Evidence-based practice is best defined as:
Al

a) A part of clinical governance that is research based and not relevant to daily clinical
practice.
b) Knowledge that has been passed down from one generation to the next and which has
in

become the professional ‘norm.’


ss

c) Strict guidelines that must be followed in order to avoid litigation.


d) The judicious use of current best evidence when making decisions about the care of
Ya

individual patients. « CORRECT ANSWER


e) The random application of small-trial research in order to make the best decisions
possible for a patient.

Evidence-based practice is a key part of clinical governance and is certainly relevant to daily
clinical practice. Increasingly, guidelines for ‘best practice’ are being issued and these often
highlight the evidence underlying the recommendations. Although guidelines should be followed
where possible, they will not cover every specific situation that you will encounter, so
professional judgement and previous clinical experience should always be used.

20-You work one session a week for the student dental practice at a campus university. A
student has seen a notice that invites healthy volunteers to undergo a revolutionary dental
treatment for tooth whitening. He asks whether you think he should take part, and tells you
that if he does not make some money soon he may have to drop out of university.

10

Yassin Al-Safadi (Safadi92@hotmail.com)


What is the best advice you can offer him?

a) Advise him to ask his own GP whether he is healthy enough to take part in the research.
b) Explain that he must weigh up the pros and cons of taking part in the research and
discuss the safety of the procedure in detail with those involved. Emphasise that there
are services available at the university to help students who have financial problems.«
CORRECT ANSWER
c) Offer him a part-time job at the student practice and offer to give him a discounted
tooth-whitening procedure.
d) Tell him that there are other ways to make money, and that participating in research
can be very dangerous.
e) Tell him that you have never heard of anyone having problems with tooth-whitening
procedures, and that it should be fine for him to take part.

Comment on this Question

The rights of individuals involved in medical research must be protected, and fully informed

i
ad
consent must be obtained. The individual should not be coerced into taking part, and the
financial reimbursement that is offered should not prevent them from acting rationally and
responsibly. Offering your own advice is inappropriate, as you do not know enough about the
af
intended procedure. Equally, although the student’s GP may ascertain that he is in good health,
-S
he is not in a position to give advice on whether the student should take part in the research.
Al

21-You are a newly qualified dentist and have been asked to sign forms to claim for work that
has been carried out in your practice not only by yourself but also by several of your
colleagues, as they are too busy to undertake this administrative task themselves. Your
in

practice manager says that it is fine for you to sign the forms, but you are unsure about this.
ss

What is the next best course of action?


Ya

a) Ask for a transfer to a different practice.


b) Carry on signing the forms and make a note to check with your supervisor as soon as
possible whether this is acceptable.
c) Consult your defence organisation, but carry on signing the forms until you get a chance
to speak to them.
d) Consult your defence organisation immediately. « CORRECT ANSWER
e) Phone your friend, who is also a newly qualified dentist, and ask for their advice.

Comment on this Question

If you are in any doubt about correct practice, ask for advice. Your signature provides proof of
understanding and acknowledgement of these procedures, and you may be held accountable in
the future. It is appropriate to refuse to sign for the work of other practitioners until you have
sought advice. Although your peers are a good starting point for advice in some circumstances,

11

Yassin Al-Safadi (Safadi92@hotmail.com)


they are at the same stage in their career as you, and they may not know the best course of
action.

22-In adult practice, for how long are clinical records generally kept?

a) Indefinitely.
b) 2 years.
c) 100 years.
d) Until 11 years after a treatment episode. « CORRECT ANSWER
e) Until 25 years after a treatment episode.

Comment on this Question

Some patient categories require different minimum retention periods. Generally speaking,
however, records should be kept for 11 years after the treatment episode, as keeping them
indefinitely would cause problems with regard to safe storage, ease of access, etc.

According to the Medical Protection Society, if the records belong to a child, they are usually kept

i
ad
"Until the patient’s 25th birthday, or 26th if they were 17 when treatment ended, or 8 years after
patient’s death if death occurred before 18th birthday."
af
Further information on this subject is available on the Medical Protection Society website and
-S
the Department of Health website, see for example 'Records management: NHS code of
practice'.
Al

23-You are the on-call maxillofacial surgery registrar and are called to the Emergency
Department to see a patient who has been assaulted. He is a 41-year-old Somalian man who
in

speaks little English and who has a clearly fractured mandible and several deep lacerations to
ss

his face. You think that he needs to have his mandible plated and the lacerations
irrigated/debrided and closed in theatre. Unfortunately, it is difficult to explain to him what
Ya

you intend to do, due to his lack of English. You think that he is able to understand you if you
use a combination of diagrams and hand gestures.

What is the best course of action in this scenario?

a) Discuss the situation with a senior colleague and ask them to make the decision.
b) Discuss the situation with the patient’s GP.
c) Find an interpreter who can check the patient’s comprehension of the situation. «
CORRECT ANSWER
d) Proceed with the surgery, as it is in the patient’s best interests and he has probably
understood enough of what you have told him.
e) Wait for a family member to explain your intended treatment to the patient, when they
arrive after the weekend.

Comment on this Question

12

Yassin Al-Safadi (Safadi92@hotmail.com)


It is often better to have an independent interpreter translate for you, rather than a family
member, to avoid the possibility of other agendas being represented and the patient being
coerced into undergoing treatment, rather than being allowed to express their own wishes.

24-You are the senior house officer on call, and you are about to scrub in and assist your
registrar, who is preparing to perform a surgical tracheostomy on an ITU patient. Your
colleague has been behaving strangely during the evening, and you think that you can smell
alcohol on his breath. It is the middle of the night and there are no other registrars or
immediate senior colleagues in the hospital.

What is the best course of action in this scenario?

a) Confront the registrar with the allegation in private.


b) Inform the theatre staff that you will be performing the operation on your own.
c) Phone your consultant and ask them to come in urgently and intervene, based on your
suspicions. « CORRECT ANSWER
d) Phone your defence organisation to check your medico-legal position.

i
ad
e) Tell the anaesthetic registrar about your suspicions, so that they do not anaesthetise the
patient.

Comment on this Question


af
-S
You have a duty of care to your patient and a duty of responsibility to your colleague. If you
suspect that a senior colleague is not fit to work, you must act quickly to protect your patient
Al

from potential harm, as it would be indefensible to do otherwise. Asking your consultant to come
in because your colleague is not fit to work is the best choice here. These matters need to be
in

handled with the utmost sensitivity.


ss

25-You are the junior house officer for the maxillofacial team. While organising the theatre
list, you have been asked to check consent forms for the patients who are going to theatre.
Ya

You realise that the patient who has just gone to theatre has signed a consent form for the
wrong procedure, due to a mix-up over paperwork. By the time you get to theatre the patient
is already anaesthetised and the surgeons are scrubbed.

What is the best course of action in this scenario?

a) Alert the surgeons and theatre staff to the paperwork mix-up. « CORRECT ANSWER
b) Change the details on the consent form to make them correspond to the procedure that
is to be performed.
c) Discuss the case with your defence organisation.
d) Fill in an incident report form and contact the risk management team.
e) Say nothing and leave, as the surgeons and theatre staff will certainly have deduced by
now that the wrong procedure was stated on the form.

Comment on this Question

13

Yassin Al-Safadi (Safadi92@hotmail.com)


By alerting them you will protect the patient from potential harm. It is also possible that the mix-
up has been spotted and the consent form corrected or rewritten before the patient was
anaesthetised.

In any event, the best course of action is to inform your senior colleagues so that they have the
information necessary ahead of time to prevent potential wrong-site surgery.

26-A patient with poor dentition that has caused infective endocarditis is to have his upper
incisors removed. He is unhappy with your explanation and wishes to make a complaint to
your senior colleagues. You feel that you have explained the proposed treatment politely and
that he is being unreasonable.

What is the best course of action in this scenario?

a) Continue to explain to the patient that you did not mean to upset him, and assure him
that your registrar will only tell him the same thing.
b) Give the patient a leaflet for the Patient Advice and Liaison Service (PALS), and advise

i
him to raise the matter with them.

ad
c) Give the patient literature on different types of denture that will be available to him.
d) Listen to the patient, inform your registrar about what has happened, and ask them to
af
speak to the patient.« CORRECT ANSWER
-S
e) Reassure the patient that you are qualified to determine the correct course of
treatment.
Al
in
ss

27-You are a partner at a dental practice. You notice that your fellow partner smells of alcohol
first thing in the morning before a morning clinic. You confront your partner and they admit
Ya

that they are under a lot of stress. Which one of the following is the next appropriate step?

a) Inform the GDC


b) Inform the DDU
c) Ask your partner to go home and return when sober for further discussion « CORRECT
ANSWER
d) Allow the clinic to proceed as further confrontation will create undue stress
e) Delay the list for long enough for your partner to compose themselves

Comment on this Question

This is the best and most appropriate option at this stage. He should not be allowed to undertake
any clinical work while intoxicated as it poses patient safety risks, and patient safety is the
paramount concern. Obviously it is important to support your colleague through a difficult time,
however it must be made clear that this behaviour cannot be tolerated in the workplace. If you

14

Yassin Al-Safadi (Safadi92@hotmail.com)


found that your colleague had been treating patients under the influence, you must act quickly
to protect the patients from harm.

28-A 4-year-old boy is brought in to your surgery with extensive dental caries. Which one of
the following would not be in keeping with possible neglect?

a) Irregular attendance and repeatedly failed appointments


b) Failure to complete planned treatment
c) Returning in pain at repeated intervals
d) Requiring repeated general anaesthesia for dental extractions
e) Regular clinic attendances for check ups « CORRECT ANSWER

Comment on this Question

While a consumerist model of dental care may suit adults who seek care when in pain, children
rely on their parents to make these decisions for them. Poor dental health and extensive caries in

i
ad
children may cause social problems at school, pain and disturbed sleep. Whether neglect is wilful
is contentious as it can be due to poor education on behalf of the parents.
af
29-Four months following a needle stick injury during surgery from a high risk patient, you
-S
have unfortunately tested positive for hepatitis C. Which one of the following represents the
best course of action at this stage?
Al

a) Inform local occupational health services « CORRECT ANSWER


b) Inform the DDU
in

c) Inform the GDC


ss

d) Protect your own right to confidentiality


e) Seek retraining in a more managerial role
Ya

Comment on this Question

It is possible to continue working as a doctor/dentist with a communicable disease, however


precautions must be exercised and exposure prone procedures limited or restricted. Patient
safety is paramount and is the prime consideration.

30-Which one of the following statements best describes statistical power?

a) This is when a true null hypothesis gets rejected


b) This is when a false null hypothesis gets accepted
c) Refers to the probability that the observed statistical difference has occurred by chance
d) It shows that there is no difference in outcome between two groups after an
intervention in one of the groups
e) The probability of correctly rejecting a false null hypothesis « CORRECT ANSWER

15

Yassin Al-Safadi (Safadi92@hotmail.com)


Comment on this Question

If the power of a study is low, the results are likely to be inconclusive. Factors that affect power
are:

1. the magnitude of change in the outcome being measured;

2. the level of statistical significance chosen at which to analyse these data;

3. the data variability reflected in size of standard deviation; and

4. whether the sample is normally distributed about the mean value.

31-The daughter of an 85-year-old patient of yours comes and asks for a summary of her
mother’s dental history as she is moving to Australia and would like to take the information
with her to give to a new dentist. You have been her mother’s dentist for many years and
know both the daughter and her mother well. Which one of the following is the next best
course of action?

i
ad
a) Print out the records for the daughter so as not to cause offence
b) Explain this is only possible with a fee and that you will find out how much it might cost
af
c) Kindly explain that you will need consent from her mother before proceeding further «
CORRECT ANSWER
-S
d) Explain that under no circumstances may you give out confidential information for
Al

another health professional to use


e) Tell the daughter that, legally, you must have the General Dental Council’s approval
before providing a summary of her notes
in

Comment on this Question


ss

It is not unreasonable for patients to give consent for their own notes to be copied for use by
Ya

another health professional, as this will ensure good patient care. It is not acceptable, however,
to request this on behalf of another patient, even if they are a family member. No one can
consent on behalf of another patient and the GDC advise that the patient’s consent must be
sought if patient information is to be released. There may be a fee involved to cover
administration costs but this is not the point of this question. Furthermore, for advice regarding
confidentiality matters, the GDC are a source of help but there is no law stating each case must
be discussed individually. It is the responsibility of the health professional to be up-to-date with
the latest guidance on legal and ethical matters.

32-A colleague asks if you have some interesting radiographs that he may use in a teaching
session. You have recently had three complex cases with informative imaging and wonder
what your duties to confidentiality are in this setting. Which one of the following is the best
course of action?

16

Yassin Al-Safadi (Safadi92@hotmail.com)


a) Ask the three patients for informed consent to allow their X-rays to be used in teaching
to dental professionals« CORRECT ANSWER
b) Cross off the names/identifying information that appears on each X-ray
c) Refuse to help him with the teaching as it breaches confidentiality
d) Photocopy X-rays out of a text-book and give them to your colleague to avoid offending
him
e) Give him the X-rays but ask that he returns them within 48 h so that they will not be lost

Comment on this Question

Colleagues may ask you to help with teaching or research and this may involve using patient
information. As per GDC advice, the best course of action to enable this is to get informed
consent from the patient and release only the minimum information needed. Removing patient
identifiers eg names/date of birth from the material is good practice, but is not the ‘best’ answer
as consent should always be sought where possible. Photocopying X-rays from a textbook could
be construed as plagiarism and should be avoided.

i
ad
af
33-You are a dental trainee and have been recently shown, just once, how to inject local
-S
anaesthetic for a root canal procedure. You have never done the procedure alone, but the
dentist with whom you are working tells you he needs to make an important phone-call and
Al

implies you should do the procedure alone. You do not feel competent to do this. Which one
of the following is correct of the GDC guidance?
in

a) Seeing a procedure once before attempting alone is generally acceptable


ss

b) You should not undertake procedures that you are not competent to do « CORRECT
ANSWER
Ya

c) If a senior presumes you are competent, even if you are not, then it is their
responsibility if anything goes wrong
d) Refusing to undertake a procedure with which you are uncomfortable will be frowned
upon at your appraisal
e) There is no guidance on this matter

Comment on this Question

There is GDC guidance on this and it states that ‘you have a duty to work within your knowledge,
professional competence and physical abilities’. It goes on to explain that carrying out a
procedure that you are not fully trained for and not competent at, is not acceptable. It is your
responsibility, not your seniors, to recognise that you are not able to carry out this procedure
safely after being shown only once before. If you feel unable to broach the subject with the
dentist with whom you are working, it may be necessary to speak to a senior member in the
practice and if the situation is not easily resolved, you could seek advice from the GDC.

17

Yassin Al-Safadi (Safadi92@hotmail.com)


34-You see a colleague drinking from a whiskey bottle in his car just before his morning
surgery begins as you are walking to work. He always acts very professionally at work and his
patients always sing his praises to you. He has worked at the practice for 30 years and you
only started working at the practice 2 months ago. Which one of the following is the best
immediate response in this situation?

a) As you are new to the practice, it is too early to inform anyone


b) Presume he remains to be a good dentist and what he does in his personal time is none
of your business
c) Ring the GDC immediately and voice your concerns
d) Find a quiet moment during the day and voice your concerns to the colleague in a
supportive manner « CORRECT ANSWER
e) Decide you will monitor his behaviour for the next month and then take further action if
necessary

Comment on this Question

i
ad
You should always put the patient’s interests first, even in difficult situations. While he may be
functioning reasonably well at work and be a popular dentist, taking alcohol before a surgery is
dangerous and is not acceptable under any circumstances. Waiting a further month could put
af
numerous patients at risk and if it is known that you did not raise concerns earlier, your own
-S
registration could be at risk. Approaching the GDC without discussing it first or seeking more
information from your colleague would seem a little hasty but it should be addressed as soon as
Al

possible with the colleague, ie. same day, and then you can decide how next to proceed. If local
action fails or is not possible, it may be sensible to approach the GDC earlier. A senior dentist in
in

your surgery should be involved.


ss

35-After providing local anaesthetic to an 18-year-old gentleman for a routine dental filling, he
says he is too scared to now have the actual procedure and wants to go home. Which one of
Ya

the following would be your next course of action?

a) Acknowledge that dental procedures can cause anxiety but try and carry on with the
filling anyway
b) Convince him to have the procedure any way you can as you have already given the
anaesthetic
c) Recognise that he has withdrawn consent and stop as continuing would constitute
assault « CORRECT ANSWER
d) Go and speak to his mother in the waiting room and gain consent from her to proceed
e) Tell him he cannot return for the procedure if he withdraws consent at this late stage

Comment on this Question

One of the most important principles of ethics is autonomy: the right of the individual to decide
what happens to his or herself. If the patient is competent and withdraws consent, at any stage

18

Yassin Al-Safadi (Safadi92@hotmail.com)


of the procedure and for any reason, then you must not proceed as this is assault. Trying to talk
to the patient to address his concerns is reasonable and he may then, with new information, give
valid consent once more. A patient’s further care should never be compromised if they do not
consent for a particular procedure, even if the health professional does not understand or agree
with, the reasons for withdrawing consent. You cannot ask his mother for consent as no one can
consent for a competent adult over the age of 16 years old.

36-A 35-year-old woman asks for a copy of her dental records and clinical reports to take to a
new dentist as she struggles to get an appointment at your surgery. Which one of the
following is the best course of action?

a) Apologise but explain that you cannot release her records to another dentist
b) Give her the original records and delete her from your patient list
c) Tell her it might take a few weeks but it should be possible
d) Try and persuade her to stay at your practice and offer to always find an appointment

i
ad
for her when she asks
e) Photocopy her records and give her a copy. Keep the originals in a safe place in case you
af
need to refer back to them at a later date« CORRECT ANSWER
-S
Comment on this Question

Reports and records must be released to a patient if they request this. You must respect their
Al

autonomy in wanting to move to a new practice and not providing records/delaying advice
about this is irresponsible. Always keep a copy of the records for yourself in case there are any
in

legal matters that arise as you should be able to reproduce appropriate documentation. Never
make impossible promises to patients in an effort to keep them at your practice. It may not
ss

always be possible to find appointments on demand but you could explain that you and your
Ya

staff will try and be as accommodating as possible.

37-Which one of the following is responsible for ensuring that you maintain professional
development and keep up-to-date with current standards?

a) The most senior dentist in your practice


b) The university from which you qualified
c) The faculty of dental surgery
d) Yourself « CORRECT ANSWER
e) Your clinical supervisor

Comment on this Question

19

Yassin Al-Safadi (Safadi92@hotmail.com)


Health professionals should always strive to better themselves professionally and improve and
maintain their competency wherever possible. Establishments such as universities and the
Faculty of Dental Surgery may complement this ongoing development but should not be relied
upon solely. It is the responsibility of the health professional to keep up-to-date with current
research, guidance and advice.

38-It is Sunday evening and you have been feeling flu-like symptoms with high fever, muscle
aches and a sore throat all weekend. You are concerned you might have Influenza A (H1N1,
swine flu) as your 6-year-old son was diagnosed recently. You are giving a lunchtime
presentation tomorrow at work and do not want to let your team down. Which one of the
following is the next best course of action?

a) Go to work despite feeling so unwell and hope you can make it through the day
b) Call the receptionist at your surgery and tell her you won’t be coming in today but you
would prefer not to say why

i
ad
c) Call your clinical supervisor, explain your symptoms and worries. Speak to your local
virologists/GP to arrange swab taking« CORRECT ANSWER
af
d) Self-prescribe penicillin for your throat and go to work at lunchtime
e) Self-prescribe Tamiflu and discuss this with your local pharmacist
-S
Comment on this Question
Al

The advice regarding symptoms and management of H1N1 influenza are continually changing as
the pandemic progresses. As a health professional, ‘you should not let your own state of health
in

put other patients at risk’ according to the GDC. Influenza is easily spread and if you have
convincing symptoms, it would be recommended to stay off work until either you have a
ss

negative swab, or your symptoms have ceased. The virus is spread by droplet/aerosol
Ya

transmission and increased by close contact, which makes transmission in the dental settings
very likely. It is worth letting your colleagues at the practice know about your symptoms as they
may be experiencing similar symptoms and further action may need to be taken. Self-prescribing
is not appropriate as it may be dangerous for you to be improperly treated and also a risk to your
patients as pencillin nor tamiflu will ensure you are not infectious.
http://www.gdc.uk.org/Our+work/Standards/Influenza+Pandemic+Statement.htm

39-Revalidation is being developed to ensure that dental professionals:

a) Maintain satisfactory standards to maintain their registration with the GDC « CORRECT
ANSWER
b) Are qualified to practice
c) Are not doing too much private work

20

Yassin Al-Safadi (Safadi92@hotmail.com)


d) Are declaring any additional income earned
e) Co-operate with any legal proceedings if called upon

Comment on this Question

The General Dentist Council register already ensures that dentists are qualified to practice so this
is not the aim of revalidation. Similarly, it has not been designed to supervise income/tax
declaration or ensure co-operation with legal proceedings. Revalidation is a new process that
attempts to ensure that dental professionals have up-to-date knowledge and are fit to practise
to stay on the general dental register. It is currently being investigated to ascertain the most
effective way of implementing this.

40-A patient asks you for advice about travelling to Fiji for a once in a lifetime trip. She is often
in need of emergency dental treatment and seeks your advice about how regulated overseas
dental surgeries may be. Which one of the following would be your best advice?

a) All countries conform to the same standard as the UK

i
b) The UK has stringent criteria for fitness to practise which other countries presumably

ad
follow
c) You feel that you are not legally allowed to comment on this matter so refrain from
af
advising
-S
d) She should avoid any overseas travel altogether
e) Other countries may not conform to the same high standards and registration checks as
Al

the UK « CORRECT ANSWER

Comment on this Question


in

In the UK, all dental practitioners must be registered with the General Dental Council to work
ss

and they must adhere to the set standards. It is difficult to say that other countries adhere to
these high standards or have similar regulations. It is the responsibility of the patient to
Ya

investigate the relevant country and the standards to which they adhere. You could advise if you
had useful knowledge in this area or perhaps direct them to the GDC website which has advice.

41-A 21-year-old woman comes to your practice for a routine check-up. She tells you that she
has recently been diagnosed with depression and is looking for work as a model. She feels
that tooth-whitening treatment is the answer to her problems, but she cannot afford to pay
for it. She insists that you perform the procedure as part of her routine NHS care, and hints
that she might harm herself if this cannot be done.

What is the best course of action in this scenario?

21

Yassin Al-Safadi (Safadi92@hotmail.com)


a) As the patient has expressed her intention to self-harm, you ask her permission to speak
to her GP to arrange an urgent appointment for the same day to fully assess her mood
disorder. She agrees to this.« CORRECT ANSWER
b) You discuss her concerns about her teeth but also explain that she cannot demand
treatment.
c) You discuss her concerns, explain that she cannot demand treatment, and advise her to
see her GP in the next few weeks for advice about her depression.
d) You tell her that she cannot have the tooth-whitening treatment on the NHS and that
you will see her at her next routine appointment.
e) You tell her that you will perform the treatment without charge on this occasion, but
that she cannot demand free treatments in the future.

Comment on this Question

You have a duty of care to the patient and she has expressed worrying self-harm ideation.
Patients may not always present with purely dental issues and, as a dentist, you have a
responsibility to direct them to other healthcare professionals if necessary. It would be unsafe to

i
ad
address only the tooth-whitening issue and then send this patient away. She is clearly distressed,
and liaising with her GP will help to ensure safe and appropriate follow-up.
af
-S
42-You have been asked to submit a review paper for publication by your consultant. In the
Al

process of editing it, you find major similarities to a published article that you have recently
read, with some sentences reproduced word for word. You check the references and find that
they do not include the published article, despite the fact that the content is remarkably
in

similar. You discuss this with your consultant and he dismisses your concerns and the
ss

similarity to the article that you have shown him.


Ya

What is your best course of action?

a) Assume that this is just a coincidence and submit the paper as it is without any
modifications.
b) Include the similar-sounding article in the references without telling your consultant,
and then submit the paper.
c) Remove the similar-sounding parts of the submission even though this will weaken the
arguments that have been made.
d) Reword the paper so that it is subtly different from the published article that you had
read.
e) Tell your consultant that the paper requires re-working and cannot be submitted as it
stands. « CORRECT ANSWER

Comment on this Question

22

Yassin Al-Safadi (Safadi92@hotmail.com)


This is the best solution to a difficult dilemma. You must maintain intellectual integrity in
publishing, as the work will be attributed to you. However, it is a difficult balance to achieve, as
you must also preserve the relationships within the team.

43-You are concerned that one of your colleagues is posting information about their working
day on a social networking website. This includes some statements about patient conditions,
as well as critical comments about management by another team within the hospital. These
statements and comments could be construed as controversial, and are potentially viewable
by the public.

What is the best course of action?

a) Contact your colleague’s consultant or immediate senior and discuss the best course of
action.
b) Contact your defence union for an opinion.
c) Contact the hospital’s risk management team.
d) Discuss the matter with your colleague and suggest that they refrain from posting

i
ad
critical statements in the public domain.« CORRECT ANSWER
e) Report your colleague to the General Dental Council in the first instance.

Comment on this Question


af
-S
The limits of free speech are debatable, but a professional attitude must be maintained,
especially when the public can view these statements. One must also consider the potential
Al

damage that could be caused to the hospital or practice and the image of the profession overall.
Regardless of whether critical comments about management have a basis in fact, they should be
in

expressed via the appropriate channels and dealt with constructively. With the advent of social
media, it is now possible to track movements to a hospital or ward, which makes commentating
ss

on interactions with patients a potential breach of confidentiality and thus highly unprofessional.
Ya

44-A patient comes to you in extreme pain following root treatment of their right fourth
upper premolar. The dentist who performed the treatment is not available today. You surmise
that part of the instrument broke off and is stuck within the tooth, and that this is the cause of
the pain. The patient says that he was not informed that this could happen and, although he is
grateful to you for helping him, he wants to know who is at fault.

What should you do?

a) Explain that although this is an unfortunate incident, it is a known complication of


endodontic surgery, and apologise for any pain or inconvenience that it has caused.«
CORRECT ANSWER
b) Explain that you feel that your colleague has been negligent and that this should not
have happened.
c) Explain that your colleague should have noticed that part of the dental tool was missing.

23

Yassin Al-Safadi (Safadi92@hotmail.com)


d) Provide the patient with the information necessary to make a complaint to the dental
practice manager.
e) Tell the patient that this happens all the time, so it is not often mentioned.

Comment on this Question

When speaking to a patient it is never appropriate to 23riticize a colleague, especially if they are
the patient’s longstanding practitioner, as this will damage the practitioner–patient relationship.
It is unfortunate that the patient was not warned that such an incident could occur as a result of
endodontic therapy, and an apology can be offered on behalf of your absent colleague. If the
patient still wishes to make a compliant, obviously the local procedures can be followed, but it is
advisable to defuse the complaint at this stage if possible.

45-You are the maxillofacial registrar working with a disabled wheelchair-bound junior trainee
who has recently joined the rotation. He is very keen to prove that he can perform all of the
necessary procedures without any of the support that has been offered to him. He often
refuses the help of nurses, who have subsequently expressed concerns about his practice in

i
ad
terms of not maintaining sterility during procedures.

What is the best way to handle this situation? af


a) Assign the trainee to jobs that involve minimal patient contact.
-S
b) Discuss the issue with the hospital risk management team.
c) Discuss the performance issues with the trainee and explain that potential patient
Al

safety issues are of paramount importance, and that he should accept the help he has
been offered.« CORRECT ANSWER
in

d) Report the trainee to the General Dental Council as unfit to practise.


e) Stop the trainee undertaking procedures without supervision until you and the
ss

consultants are happy with his technique.


Ya

Comment on this Question

Your junior colleague may be experiencing feelings of deep insecurity because of his disability,
which are the cause of his risk-taking behaviour and refusal of help, as he perceives accepting
assistance as a sign of weakness. This is clearly a sensitive situation, as he may understand the
steps required to undertake a difficult procedure as well as any other trainee, but his disability
could limit his practical execution of the procedure.

There are two problems here, namely patient safety, and the obligation to train a junior
colleague so that they become competent. These issues are closely interrelated, as potential
problems could arise if the trainee is not adequately supported in difficult situations. You must
encourage him to use the help that is available, perhaps by demonstrating that you would do
this even if he did not have a disability.

24

Yassin Al-Safadi (Safadi92@hotmail.com)


It is important to encourage and support the trainee, but at the same time to swiftly address any
patient safety issues that arise or that could potentially occur.

46-You are the senior partner at a practice. You have noticed that your new partner is arriving
at work increasingly late, is abrupt with patients and staff, and is receiving phone calls from
credit card companies during working hours. She appears very tired and anxious.

What is the best action to take initially?

a) Start a disciplinary procedure regarding her late arrival at work.


b) Talk to her in between patients and discuss her change in attitude.
c) Talk to her out of hours and try to find out what the underlying issues are. « CORRECT
ANSWER
d) Talk to the other partners and ask them whether they have noticed any other
behavioural issues.
e) Tell her that it is inappropriate to be receiving personal phone calls during working
hours.

i
ad
Comment on this Question
af
All of the other options might well be useful or possible approaches, but they would not
constitute the best initial action. Your colleague is in distress, and so long as she is not
-S
24eopardizing patient safety you have an obligation to support her. There may be little that you
can offer in terms of material help, but an understanding problem-sharing attitude may be the
Al

first step towards improving her performance.


in

47-Your new senior house officer is constantly staying late and phoning the ward after hours
to check on his patients. He is very diligent with his work but inevitably, being junior, he still
ss

has a lot to learn. He spends a long time with each patient he sees in clinic, and so far he has
perceived your constructive advice as criticism.
Ya

What is the best solution to his problem with time management?

a) Book fewer patients into the clinic, observe a few consultations to find out whether he
is over-investigating, and after the clinic discuss his performance in the context of
learning.« CORRECT ANSWER
b) Give him some simple tasks to do that support the clinic.
c) Book a full clinic, allow it to run late and then critically explain to him that he has slowed
down the clinic.
d) Supportively explain that his working patterns are unhealthy and ultimately not
sustainable.
e) Tell him to take less time with patients.

Comment on this Question

25

Yassin Al-Safadi (Safadi92@hotmail.com)


Your SHO is very diligent but over-cautious. A fine balance needs to be achieved in practice.
However, he is probably over-investigating patients, which in the long term will have
implications for resource allocation. His working patterns are also having an impact on his life
outside work. He may not even perceive them as being a problem, but they are certainly not
sustainable in the long term. Booking a smaller clinic is not a long-term solution, but as a
learning exercise it might be a useful way to broach these issues sensitively.

48-A patient discloses that she has been abused as a child but asks you to keep it to yourself.
What management, if any, is best?

a) Ask the patient’s permission to write in the patient’s notes. Ask if they have sought
advice/guidance about it.« CORRECT ANSWER
b) Do nothing. It was a long time ago.
c) Report it in the patient notes. Do nothing further. The patient notes are confidential.
d) Report the incidence to social services.
e) Write nothing in the notes but discuss with colleagues in the coffee room.

i
ad
Comment on this Question

If a patient reveals sensitive issues about their past or present it is best to try and discuss
af
it with the patient further. They may then disclose that they have sought treatment and
-S
management of the issues arising and are now happy as a result. If they have not, you may need
to guide them to sources of assistance. If they have asked for discretion it is important you
Al

maintain this, and therefore do not break their confidentiality without asking by discussing with
colleagues in an open area. If you discuss the matter further it may or may not be appropriate to
refer to social services, but this is not necessary as an immediate course of action.
in

49-An untrained dental nurse is just starting on reception while she obtains her hepatitis B
ss

immunisation. However one of the trained nurses telephones in sick and this leaves one male
Ya

dentist a nurse down. They are happy to see patients alone as they have examinations all day.
What is the best way to proceed?

a) Cancel all patients. A dentist should always work fourhanded.


b) Use the receptionist as a nurse, as she has hepatitis B immunisation, and leave the new
nurse on reception alone.
c) Work alone with no nurse or chaperone.
d) Work with the new dental nurse nursing throughout.
e) Work with the new dental nurse as chaperone only, as she does not have a hepatitis B
immunisation. « CORRECT ANSWER

Comment on this Question

It is inappropriate to work alone without a chaperone even if the dentist and patient are happy.
This ensures that if there are any issues or allegations in the future there is a third person to
corroborate what happened. A dental nurse cannot nurse or have any exposure to any risk of

26

Yassin Al-Safadi (Safadi92@hotmail.com)


needlestick injuries (which includes decontamination and nursing work) until they have their
hepatitis B immunisation. However, they can act as a chaperone, particularly for male dentists.

50-What is the best description of clinical governance?

a) A framework to ensure all the paperwork for the Care Quality Commission (CQC) is
completed by running audits and maintaining continuing professional development.
b) A framework to provide a systematic approach to maintain high standards. This includes
keeping up-to-date regular team briefings, clinical audit and quality assurance.
c) A framework to provide a systematic approach to maintaining and improving the quality
of care and safeguarding high standards of care in which clinical excellence can flourish.
This includes evidenced-based dentistry, clinical audit, quality assurance and continuing
professional development.« CORRECT ANSWER
d) A systematic approach to maintaining and improving the quality of care by using
evidenced based dentistry, patient feedback and complaints handling.
e) Continually assessing standards through clinical audit and implementing changes to
raise and maintain clinical standards.

i
ad
Comment on this Question
af
Clinical governance is a way of continually increasing standards. It has six pillars that include
-S
clinical audit, quality assurance programmes, evidence-based dentistry and continuing
professional development. It is not just about CQC paperwork but about continually improving
Al

standards and quality of care. It is important to have good complaints handling and patient
feedback, but it is not a key part of clinical governance.
in

51-Your dental therapist is concerned about whether they have done the correct amount of
ss

continuing professional development (CPD). What should they have completed in 5 years?

a) 50 hours verifiable and 100 hours non-verifiable. This includes 5 hours of medical
Ya

emergencies, 5 hours of infection control and 5 hours of radiography.


b) 50 hours verifiable and 100 hours non-verifiable. This includes 10 hours of medical
emergencies, 5 hours of infection control and 5 hours of radiography.« CORRECT
ANSWER
c) 50 hours verifiable and 150 hours non-verifiable. This includes 10 hours of medical
emergencies, 5 hours of infection control and 5 hours of radiography.
d) 75 hours verifiable and 120 hours non-verifiable. This includes 5 hours of medical
emergencies, 5 hours of infection control and 5 hours of radiography.
e) 75 hours verifiable and 125 hours non-verifiable. This includes 10 hours of medical
emergencies, 5 hours of infection control and 5 hours of radiography.

Comment on this Question

All dental care professionals including dental nurses, hygienists and therapists have to complete
a total of 150 hours CPD in 5 years. This is split into 50 hours verifiable and 100 hours non-

27

Yassin Al-Safadi (Safadi92@hotmail.com)


verifiable including 10 hours of medical emergencies, 5 hours of infection control and 5 hours of
radiography.

52-A patient makes an NHS complaint. How quickly does a response and a formal report need
to be completed within?

a) Response within 3 days. Report within 10 days.


b) Response within 3 days. Report within 1 month.
c) Response within 3 days. Report within 6 months. « CORRECT ANSWER
d) Response within 5 days. Report within 6 days.
e) Response within 5 days. Report within 1 month.

Comment on this Question

After a formal written complaint has been made, an acknowledgement needs to be made within
3 days. An investigation into the case should then occur with a formal report at the end. This was
previously completed within 10 days; however it was found to be not long enough and has been

i
extended to within 6 months. This enables a full and proper investigation and report to be made.

ad
The person making the complaint needs to be kept well informed during this period.
af
53-A 7-year-old child attends the practice. He was referred for a full dental clearance under
general anaesthetic at 4 years old. The family had diet and oral hygiene advice at the time but
-S
he has not been seen since and now attends in pain. On examination there is extensive caries
in his LL12 and LR12, and early caries in all 6’s. How would you manage the situation?
Al

a) Complete the restorative work necessary and reinforce good oral hygiene and diet
in

advice.
b) Discuss management with the family. Advise that the rest of the family may need
ss

treatment.
c) Highlight the patient as a possible neglect case and raise concerns with the family, in
Ya

particular about siblings. Refer immediately to community dental service for complex
management. If there are any further issues or concerns refer to social services.«
CORRECT ANSWER
d) Refer for a general anaesthetic for further extractions. Assess the rest of the family for
possible treatment as well.
e) Refer immediately to social services. Do not inform the family and complete the
restorative work necessary.

Comment on this Question

Due to the extensive nature of the work it would be advisable to have assessment and possible
treatment with a paediatric consultant, particularly if extensive caries reaching to the pulp is
suspected. The patient has had previous advice that they have not taken on board; this needs to
be noted. If they fail to attend appointments in the future, a referral to social services needs to
be considered. Any siblings or family members may also have complex problems that are not

28

Yassin Al-Safadi (Safadi92@hotmail.com)


currently being treated and these need to be highlighted. The family may not be aware of the
situation, and therefore it is important that all options and treatment and the seriousness of the
case be discussed with them.

54-A 10-year-old patient attends with their mother who does not speak very good English.
During the appointment the child translates to allow the mother to understand. You need to
complete two fillings on the child. What do you do regarding consent?

a) Ask the auntie who is waiting in the waiting room to come in and assist with the
translation as they have slightly better English.
b) Continue using the child to translate as they seem to understand everything.
c) Continue with treatment, you believe it is the best course of treatment for the child
anyway.
d) Get the patient to attend a different appointment with their stepdad who speaks fluent
English.
e) Hire a translator to come in at a separate appointment to gain consent. « CORRECT
ANSWER

i
ad
Comment on this Question
af
If you do not believe that the patient has Gillick competence to understand all the treatment
-S
then they cannot act as a translator for their parent. It is best to use a third party as a translator
who is not involved with the family as they will not add any bias to the translation. A stepdad
Al

who speaks fluent English is OK but they are unable to consent to a child that is not their own
unless they are a lawful guardian. It is not appropriate to continue with the treatment if they are
not in pain and to book another appointment with a translator is straightforward.
in

55-A patient comes in and asks about tooth whitening. You clarify the new regulations. Which
ss

of the following best fits the new guidelines.


Ya

a) Whitening products equal to <6% hydrogen peroxide are available to use by all
professionals provided the person has have had suitable training.
b) Whitening products equal to <6% hydrogen peroxide are available to use through a
dental professional provided that the patient has had a suitable clinical examination and
guidance.« CORRECT ANSWER
c) Whitening products equal to <10% hydrogen peroxide are available to use through a
dental professional provided that the patient has had a suitable clinical examination and
guidance.
d) Whitening products up to 0.1% hydrogen peroxide are available over the counter; all
bleaching products with a higher concentration are available through a dental
professional.
e) Whitening products up to 1% hydrogen peroxide are available over the counter, but
anything higher than this must be through a dental professional.

29

Yassin Al-Safadi (Safadi92@hotmail.com)


Comment on this Question

New EU guidelines have been published recently stating that products <0.1% hydrogen peroxide
are available over the counter. Anything between 0.1% and 6% hydrogen peroxide is available
only through a dental professional registered with the GDC. All bleaching products over 6%
hydrogen peroxide are still prohibited.

56-Which one of the following is the guiding principle of utilitarianism?

a) It regards duties and rules


b) It regards doing the greatest good for the greatest number
c) The cultivation of virtue within oneself
d) All of the above
e) None of the above

57-Which principle of ethics is best described when considering a patient's ability to think
about and reason their own choices?

i
ad
a) Justice
b) Autonomy « CORRECT ANSWER
c)
d)
Beneficence
Non-maleficence
af
-S
e) None of the above
Al

Comment on this Question


in

Autonomy is one of the key tenets of ethics as described by Childress and Beauchamp. It is more
ss

than just doing what a person wants, but describes a process of rationally evaluating their
options to guide these actions
Ya

58- Which one of the following ethical principles demands the fair allocation of resources in
society and so people to be treated as equals?

a) Justice « CORRECT ANSWER


b) Autonomy
c) Beneficence
d) Non-maleficence
e) None of the above

59- Which one of the following declarations dealing with biomedical research states that 'the
interests ofthe subject must always prevail over the interests of science and society'?

a) Declaration of Independence
b) Human Rights Act (1998)
c) Declaration of Geneva (1948)

30

Yassin Al-Safadi (Safadi92@hotmail.com)


d) Declaration of Helsinki« CORRECT ANSWER
e) Hippocratic oath

i
ad
af
-S
Al
in
ss
Ya

31

Yassin Al-Safadi (Safadi92@hotmail.com)


[Dental
Materials
i
ad
af
-S

MCQs]
Al
in
ss

[With Answers]
Ya

32

Yassin Al-Safadi (Safadi92@hotmail.com)


1- This question is regarding gypsum products. Which one of the following is
most appropriate for the casting of a die on which a crown is to be constructed?

a) Gypsum type I
b) Gypsum type II
c) Gypsum type III
d) Gypsum type IV “ CORRECT ANSWER
e) Gypsum type V

• Comment on this Question

Type IV Gypsum products are high strength, low expansion dental stones. Type I
gypsum products are the least hard and type V products are the most.

2- Please select the most appropriate statement. Addition polymerisation


involving polyvinylsiloxane impression materials:

a) Commonly involves the evolution of a non-polymeric by-product

i
b) Is usually accompanied by some degree of polymerisation shrinkage

ad
c) Does not require any form of initiator for polymerisation to begin
d) Often involves the establishment of inter-chain crosslinking “ CORRECT
ANSWER
af
e) Requires monomers of specific molecular weights for a reaction to occur
-S
• Comment on this Question
Al

Addition polymerisation involves the establishment of inter-chain crosslinking.


in
ss

3-The diagram below shows the rheological properties of fluids and pastes when
extruded from a syringe.
Ya

33

Yassin Al-Safadi (Safadi92@hotmail.com)


Which one of the following is correct?

a) A represents a Newtonian fluid


b) B represents a pseudoplastic fluid
c) C represents a dilatants fluid
d) B represents a dilatant fluid
e) B represents a Newtonian fluid “ CORRECT ANSWER

• Comment on this Question

i
ad
af
-S
This is a fluid in which the viscosity is independent of shear rate
Al

4-With regards to the structure and properties of polymers, which one of the
following is true?
in

a) The temperature at which they were mixed has an effect on the properties of the
set material
ss

b) The presence of fillers has no effect on the properties of the set material
c) Fillers only affect the setting time of the material
Ya

d) The greater the density of the crosslinking, the faster the material is likely to have
set
e) The molecular structure of the pre-polymerisation monomer units has an effect
on the resulting polymer’s properties” CORRECT ANSWER

• Comment on this Question

The molecular structure of the pre-polymerisation monomer units has an effect on the
resulting polymer’s properties.

5-With regards to denture base polymers, which of the following is correct?

a) Ethyl methacrylate is a commonly used denture base material


b) Denture base polymers tend to offer poor colour stability

34

Yassin Al-Safadi (Safadi92@hotmail.com)


c) Denture base polymers exhibit excellent thermal diffusivity
d) Denture base polymers can suffer from porosity defects if not cured appropriately
“ CORRECT ANSWER
e) Denture base polymers are commonly known for their allergenic properties

• Comment on this Question

Denture base polymers can suffer from porosity defects if not cured appropriately.

6-With regards to gold alloys used in dentistry, which one of the following is true?

a) Type I gold alloys are harder than type IV gold alloys


b) Type I gold alloys are stronger than type IV gold alloys
c) Type I gold alloys are appropriate for the construction of full coverage crowns
d) Platinum acts as a scavenger within gold alloys “ CORRECT ANSWER
e) Type IV gold alloys are commonly used for the construction of inlays

• Comment on this Question

i
ad
Type I gold alloys are softer than type IV gold alloys.

af
7- Polymerisation shrinkage when placing composite restorations is reduced by:
-S
a) Using a liner
Al

b) Incremental filling and curing of the cavity “ CORRECT ANSWER


c) Using of a matrix band
in

d) Curing the composite in bulk


e) Using a rubber dam
ss

• Comment on this Question


Ya

Incremental filling and curing of composite up to a depth of 2 mm at a time helps to


reduce polymerisation shrinkage.

8-An advantage of high copper amalgam is:

a) Improved carving
b) Better adhesion to tooth
c) Reduced setting time “ CORRECT ANSWER
d) Improved resistance to caries
e) Greater ease of use in moist conditions

• Comment on this Question

None of the other options are true of high copper amalgams.

9-An advantage of high copper amalgam is:

35

Yassin Al-Safadi (Safadi92@hotmail.com)


a) Improved carving
b) Better adhesion to tooth
c) Reduced setting time “ CORRECT ANSWER
d) Improved resistance to caries
e) Greater ease of use in moist conditions

• Comment on this Question

None of the other options are true of high copper amalgams.

10-Heavy-bodied addition-cured polyvinylsiloxane impression material can


reproduce detail to:

a) 0.1 µm
b) 0.25 µm
c) 0.30 µm
d) 0.50 µm
e) 0.75 µm “ CORRECT ANSWER

i
ad
• Comment on this Question

af
Light-bodied silicones can reproduce detail of 0.25 µm, medium bodied silicones can
reproduce detail of 0.5 µm and heavy bodied silicone can reproduce detail of 0.75 µm.
-S
Al

11-Type I gold alloys contain what percentage of gold?


in

a) 95%
b) 85% “ CORRECT ANSWER
ss

c) 75%
d) 65%
Ya

e) 55%

• Comment on this Question

Type 4 gold alloys are harder than type I alloys and contain 65% gold.

12-The predominant metal in cobalt chromium alloy is:

a) Beryllium
b) Chromium
c) Cobalt “ CORRECT ANSWER
d) Molybdenum
e) Nickel

13-The property that makes cobalt chromium alloys favourable for removable
partial dentures is:

36

Yassin Al-Safadi (Safadi92@hotmail.com)


a) Density
b) Ductility
c) Hardness
d) Rigidity “ CORRECT ANSWER
e) Thermal diffusivity

• Comment on this Question

Cobalt chromium removable partial denture frameworks need to be rigid and so not be
susceptible to permanent deformation. This aids in ensuring their accurate and
reproducible seating and longevity.

14-Cobalt chromium clasps are able to engage tooth undercuts of which depth:

a) 0.25 mm “ CORRECT ANSWER


b) 0.5 mm
c) 0.6 mm
d) 0.75 mm

i
ad
e) 0.9 mm

• Comment on this Question


af
Cobalt chromium alloys are rigid and brittle in thin section. If the undercut is 0.25 mm in
-S
depth cobalt chromium is the material of choice. For undercut depths of 0.5 mm
stainless steel is the material of choice for clasps and for undercuts of 0.75 mm wrought
Al

gold is the material of choice.

15-The catalyst in the setting reaction of addition-cured polyvinylsiloxane is:


in

a) Calcium sulphate
ss

b) Hydroxyl terminal groups on pre-polymer chains


c) Lead dioxide and terminal and pendant thiol groups on pre-polymer chains
Ya

d) Platinum a salt “ CORRECT ANSWER


e) Ester derivative of aromatic sulphonic acid salt

16-The reaction involving tissue conditioners upon mixing is:

a) A chemical reaction
b) A physical reaction “ CORRECT ANSWER
c) Involves a reaction where crosslinking occurs between polymer chains
d) Is a reversible reaction
e) Is notable for being an exothermic reaction

• Comment on this Question

37

Yassin Al-Safadi (Safadi92@hotmail.com)


Upon mixing the powder and the liquid, the resultant reaction is a purely physical one
and no chemical reaction per se occurs.

17-The caveo-surface angle for an amalgam restoration is:

a) 20o
b) 40o
c) 60o
d) 90o “ CORRECT ANSWER
e) 110o

• Comment on this Question

It is recommended that the ideal caveo-surface angle is 90o. A caveo-surface angle


between 70–90o is thought to reduce the risk and manifestation of marginal breakdown.

18-The light from composite curing units is typically of which approximate


wavelength:

i
ad
270 nm

370 nm af
470 nm “ CORRECT ANSWER
-S
570 nm
Al

670 nm
in

• Comment on this Question


ss

The light required to cure composite is usually in the blue range of the electro-magnetic
spectrum, which is typically in the range of 470–475 nm.
Ya

19-In terms of bonding composite to dentine, the hybrid layer is:

a) The layer of composite that is bonded to enamel


b) The portion of enamel that is bonded to the composite
c) The thin layer of resin between the dentine and composite
d) The thin zone of resin tags that impregnates the exposed dentinal tubules “
CORRECT ANSWER
e) None of the above

20-With regards to tooth shade, value may be defined as:

a) A particular colour or variety of colour


b) The intensity or saturation of colour

38

Yassin Al-Safadi (Safadi92@hotmail.com)


c) That optical property where two colours appear to be matching under the same
light source, but have different spectral reflectance
d) The relative lightness or darkness of a colour “ CORRECT ANSWER
e) The temperature of the light under which a shade is recorded

• Comment on this Question

A defines hue, B defines chroma and C describes the property metamerism.

21-A Nayyar core is constructed from:

a) Amalgam “ CORRECT ANSWER


b) Ceramic
c) Composite
d) Glass ionomer cement
e) Gold

i
ad
• Comment on this Question

Nayyar cores are constructed to provide retention for crowns in endodontically treated
af
teeth with insufficient supra-gingival tooth tissue. They are constructed by removing 3–4
-S
mm of radicular gutta percha, then packing these with amalgam and then building the
tooth up.
Al

22-The bond strength of composite to enamel is:


in

a) 0–5 MPa
b) 10–15 MPa
ss

c) 20–30 MPa “ CORRECT ANSWER


d) 45–55 MPa
Ya

e) 60–65 MPa

• Comment on this Question

The bond strength of composite to enamel varies between 20–30 MPa based on current
literature. This range includes both one step etch and bond as well as separate etch and
bond systems.

23-Definitive posts for post and cores restorations are NOT commonly made of:

a) Acrylic “ CORRECT ANSWER


b) Composite
c) Gold alloy
d) Carbon fibre
e) Stainless steel

• Comment on this Question

39

Yassin Al-Safadi (Safadi92@hotmail.com)


A variety of new materials are used for the construction of definitive posts in
endodontically treated teeth. Acrylic is not commonly used in the construction of
definitive posts.

24-In relation to resin-retained bridges the term retainer describes:

a) The cement attaching the wing to the tooth


b) The prosthetic tooth within the bridge
c) The tooth to which the bridge is secured
d) The wing portion of the bridge that is attached or cemented to the natural tooth “
CORRECT ANSWER
e) None of the above

• Comment on this Question

The wing on a resin retained bridge is the retainer component.

25-Which one of the following does not form a component of cobalt chromium

i
removable partial dentures?

ad
a) Abutment “ CORRECT ANSWER
b)
c)
Clasp
Connector
af
-S
d) Reciprocal arm
e) Rest
Al

• Comment on this Question


in

The term abutment refers to the tooth that is used to retain a bridge. B, C, D and E
options are all components of a cobalt chromium removable partial denture.
ss

26-Which material is not commonly used for the construction of clasps on


Ya

removable partial dentures?

a) Acrylic
b) Cobalt chromium
c) Gold alloy
d) Stainless steel
e) Titanium “ CORRECT ANSWER

• Comment on this Question

Titanium is not commonly used in the construction for clasps for dentures. While cobalt
chromium, stainless steel and gold alloy are commonly used for clasp construction,
acrylic clasps are also available as a non-metallic alternative and can be used to provide
a component that is tooth coloured. These are however somewhat brittle and prone to
fracture.

40

Yassin Al-Safadi (Safadi92@hotmail.com)


27-Tissue conditioners should not be used:

a) As liners in surgical stents and under dentures in sites where surgery has been
carried out
b) As a long-term denture liner in cases of painful dentures “ CORRECT ANSWER
c) For recording functional impressions, such as for piezography
d) On baseplates in sites where a surgical defect may need to be obturated,
particularly if traumatised
e) To allow irritated soft tissue, such as sites with denture induced hyperplasia, to
resolve before recording impressions

• Comment on this Question

Tissue conditioners are a soft and compliant material, composed of polymer, monomer
and plasticizer. Over a period of the time, following mixing, the plasticizer leaches out of
the body of the gel and the material becomes harder and less gel-like. In light of this
progressive hardening, its use is not indicated in procedures where the gel is likely to be
present in the mouth for prolonged periods of time. Tissue conditioners are used most

i
ad
often in scenarios that are preparatory for procedures such as impressions or to allow
healing following surgery.
af
28-The minimum thickness of gold alloy in the construction of a cuspal coverage
-S
gold restoration is:

a) 0.5 mm “ CORRECT ANSWER


Al

b) 1 mm
c) 1.5 mm
in

d) 1.7 mm
e) 2 mm
ss

• Comment on this Question


Ya

The minimum thickness of gold alloy for a cuspal coverage restoration is 0.5 mm.

29-The marginal configuration for a metal finishing lining on a crown is:

a) A bevel
b) A chamfer “ CORRECT ANSWER
c) A feather edge
d) A shoulder
e) A shoulder with bevel

• Comment on this Question

A chamfer finish is the marginal configuration for a metal-based crown. Ceramo-metallic


and all porcelain crowns have shoulder marginal configurations. Chamfers afford the
laboratory a definitive and easy to visualise margin, with adequate space for metal for
the restoration. It also allows ease of finishing and polishing of the crown on the die.

41

Yassin Al-Safadi (Safadi92@hotmail.com)


30-With regards to impression materials which one of the following is a
hydrocolloid?

a) Alginate “ CORRECT ANSWER


b) Polyether
c) Polyethylene
d) Polyvinylsiloxane
e) Impression compound

• Comment on this Question

Hydrocolloids can be reversible and irreversible; alginate is an example of an irreversible


hydrocolloid and agar is an example of a reversible hydrocolloid.

31-Which type of glass ionomer cement is used for luting agents?

a) Type I “ CORRECT ANSWER


b) Type II

i
c) Type III

ad
d) Type IV
e) Type V

• Comment on this Question


af
-S
The glass ionomer cements (GIC) represent a reaction between a polyalkenoic acid and
Al

an aluminosilicate powder. They are classified by function as follows:

• type I – for luting cements


in

• type II – for restorations


ss

• type III – for liners and bases


Ya

• type IV – for fissure sealants

• type V – for orthodontic cements

• type VI – for core build-up.

32-Which type of glass ionomer cement is used for fissure sealants?

a) Type I
b) Type II
c) Type III
d) Type IV “ CORRECT ANSWER
e) Type V

• Comment on this Question

42

Yassin Al-Safadi (Safadi92@hotmail.com)


The glass ionomer cements (GIC) represent a reaction between a polyalkenoic acid and
an aluminosilicate powder. They are classified by function as follows:

• type I – for luting cements

• type II – for restorations

• type III – for liners and bases

• type IV – for fissure sealants

• type V – for orthodontic cements

• type VI – for core build-up.

33-Which type of glass ionomer cement is used for restorative materials?

a) Type I
b) Type II “ CORRECT ANSWER

i
c) Type III

ad
d) Type IV
e) Type V

• Comment on this Question


af
-S
The glass ionomer cements (GIC) represent a reaction between a polyalkenoic acid and
Al

an aluminosilicate powder. They are classified by function as follows:

• type I – for luting cements


in

• type II – for restorations


ss

• type III – for liners and bases


Ya

• type IV – for fissure sealants

• type V – for orthodontic cements

• type VI – for core build-up.

34-Which of the following is an advantage of glass ionomer cements?

a) They are brittle


b) They are insoluble
c) They have a setting time of 1–2 minutes
d) They have excellent lustre
e) They release fluoride “ CORRECT ANSWER

• Comment on this Question

43

Yassin Al-Safadi (Safadi92@hotmail.com)


Glass ionomer cements (GIC) are extremely useful materials. Their disadvantages are
that they are water sensitive and soluble, and they are brittle and prone to wear.
However, their adhesion to dentine is good, and they have a similar coefficient of
thermal expansion to that of dentine, so form an excellent seal, although their physical
properties make them unsuitable for occlusal loading. GIC are easy to use and have a
setting time of 6–8 minutes, which makes them easier to manipulate into a cavity.
However, their main benefit is fluoride release.

35-What is the main component of glass ionomer cement?

a) Alumina
b) Aluminium fluoride
c) Calcium fluoride
d) Silica “ CORRECT ANSWER
e) Sodium fluoride

• Comment on this Question

i
ad
The composition of glass ionomer cement is as follows:

• silica 41.9%

• alumina 28.6%
af
-S
• calcium fluoride 15.7%
Al

• sodium fluoride 9.3%


in

aluminium fluoride 1.6%.


ss

36-What is the wavelength of blue light?

a) 30 nm
Ya

b) 250 nm
c) 350 nm
d) 470 nm “ CORRECT ANSWER
e) 500 nm

• Comment on this Question

The wavelength of blue light is 460–490 nm.

37-Which of the following changes results in the formation of a cermet cement?

a) Addition of HEMA
b) Addition of silver powder “ CORRECT ANSWER
c) Change of the alumina-silicate glass
d) Mixing with a composite cement 50:50 mix
e) Use of polyvinyl acid

44

Yassin Al-Safadi (Safadi92@hotmail.com)


• Comment on this Question

These composites with added silver were designed to have an ideal combination of the
properties of ceramic and metal parts, hence the name ‘cermet.’

38-Which of the following, if added to zinc oxide eugenol (ZOE), changes it into
Kalzinol?

a) 10% hydrogenated resin “ CORRECT ANSWER


b) Increased zinc oxide
c) Ortho-ethoxybenzoic acid added to liquid
d) Polyacrylic acid added to liquid
e) Zinc acetate

• Comment on this Question

Kalzinol is a resin-bonded ZOE with increased strength.

39-What is the sintering temperature of porcelain?

i
ad
a) 300 °C
b) 550 °C af
c) 1000 °C
-S
d) 1500 °C
e) 2000 °C “ CORRECT ANSWER
Al

• Comment on this Question


in

Sintering is the process by which powdered porcelain is heated to just below its melting
point, so that the particles fuse to form a solid, which then cools and shrinks firmly on to
ss

a metal substructure.
Ya

The actual melting point of porcelain is 2400–2600 °C.

40-What is the melting point of cobalt-chrome?

a) 300 °C
b) 550 °C
c) 1000 °C
d) 1500 °C “ CORRECT ANSWER
e) 2000 °C

• Comment on this Question

This alloy of cobalt and chromium is extensively used in dentures. Its melting point is
1500 °C.

41-Which of the following is a problem associated with composite cements?

45

Yassin Al-Safadi (Safadi92@hotmail.com)


a) Aesthetic factors
b) Depth of light cure is limited to 6 mm
c) Hydrophilic properties
d) Polymerisation shrinkage of 0.5%
e) Temperature rise during light curing “ CORRECT ANSWER

• Comment on this Question

Composite cements show polymerisation shrinkage of 1–4%. They are hydrophobic, and
the depth of light cure is limited to 2– 3 mm.

Aesthetic factors are the advantage of composites.

42-Which of the following is a problem associated with composite cements?

a) Aesthetic factors
b) Depth of light cure is limited to 6 mm
c) Hydrophilic properties

i
d) Polymerisation shrinkage of 0.5%

ad
e) Temperature rise during light curing “ CORRECT ANSWER

• Comment on this Question af


-S
Composite cements show polymerisation shrinkage of 1–4%. They are hydrophobic, and
the depth of light cure is limited to 2– 3 mm.
Al

Aesthetic factors are the advantage of composites.


in

43-How deep do composite tags penetrate into enamel to provide


micromechanical retention?
ss

a) 10 μm
20 μm
Ya

b)
c) 50 μm “ CORRECT ANSWER
d) 75 μm
e) 100 μm

• Comment on this Question

With acid etch techniques and dentine bonding agent, composite can produce good
adhesion to dentine through ‘tags’, which fill the dentinal tubules to a depth of
approximately 50 μm.

44-Titanium is osseointegrating, so what is the gap between bone and metal?

a) 5 nm
b) 20 nm “ CORRECT ANSWER
c) 40 nm
d) 75 nm

46

Yassin Al-Safadi (Safadi92@hotmail.com)


e) 100 nm

• Comment on this Question

Titanium will osseointegrate with bone. However, the bone is not fully covered on
electron microscopy, as there is a small gap of 20 nm. It is the lack of soft tissue
between bone and implant that defines osseointegration.

45-What is the purity of gold used for direct restorations?

a) 4 carat
b) 10 carat
c) 14 carat
d) 18 carat
e) 24 carat “ CORRECT ANSWER

• Comment on this Question

For direct dental restorations, gold foil (24-carat gold) is used, whereas cast gold (14- or

i
ad
18-carat gold) is used for indirect restorations.

46-What change converts a glass ionomer cement into a resin-modified form?


af
a) Addition of HEMA “ CORRECT ANSWER
-S
b) Addition of silver powder
c) Change of the alumina-silicate glass
Al

d) Mixing with a composite cement 50:50 mix


e) Use of polyvinyl acid
in

• Comment on this Question


ss

Hydroxyethyl methacrylate (HEMA) is added to glass ionomer cement to create a resin-


Ya

modified form. The properties of the latter include light-curing ability and improved
physical strength. A modern example is Fuji Plus.

47-What is added to zinc oxide eugenol (ZOE) to change it into an accelerated


form (eg Sedinol)?

a) 10% hydrogenated resin


b) Increased zinc oxide
c) Ortho-ethoxybenzoic acid added to liquid
d) Polyacrylic acid added to liquid
e) Zinc acetate “ CORRECT ANSWER

• Comment on this Question

The addition of zinc acetate to ZOE decreases the setting time to about 5 minutes.

48-What is the casting temperature of gold?

47

Yassin Al-Safadi (Safadi92@hotmail.com)


a) 300 °C
b) 550 °C
c) 1000 °C “ CORRECT ANSWER
d) 1500 °C
e) 2000 °C

• Comment on this Question

The ‘lost wax’ technique, which dates back to ancient times, is used for dental casting.
Wax is invested and melted, leaving a dead space into which liquid gold (14- to 18-carat)
is poured. This is then spun, cooled, and the investment material cleaned off.

Gold melts at 1064.18 °C, which is equivalent to 1947.52 °F or 1337.33 K.

49-Which of the following is not found in stainless steel?

a) Carbon
b) Chromium

i
c) Molybdenum

ad
d) Nickel
e) Titanium “ CORRECT ANSWER

• Comment on this Question


af
-S
Stainless steel is an alloy that consists mainly of iron and carbon. It also contains
Al

chromium (which makes it resistant to scratching and tarnishing), nickel (which provides
a smooth polished surface) and molybdenum (which helps to maintain a sharp cutting
edge, as for example is required in a scalpel).
in

50-What is the polymerisation shrinkage of composites?


ss

a) 0.25%
Ya

b) 2% “ CORRECT ANSWER
c) 8%
d) 10%
e) 12%

• Comment on this Question

The polymerisation reaction results in shrinkage, which leads to the formation of a


crevice in which bacterial growth can occur, leading to secondary decay. The reduction
of polymerisation shrinkage is an important area of research, and although new ‘low-
shrinkage’ composites have been developed by altering the molecular and bulk
composition of the resin, they still shrink by 1–4%.

51-What is the particle size of microfine composites?

a) 0.01 μm

48

Yassin Al-Safadi (Safadi92@hotmail.com)


b) 0.04 μm “ CORRECT ANSWER
c) 3.5 μm
d) nm
e) 10 nm

• Comment on this Question

2.5–5.0 μm is the particle size of macrofilled composites.

52-The surface of a ceramic restoration must be prepared with what before repair
with a composite resin restoration material? Select one option only.

a) A fine diamond bur


b) An ultrasonic scaler
c) Hydrofluoric acid “ CORRECT ANSWER
d) No preparation is required
e) Phosphoric acid

i
ad
• Comment on this Question
af
Hydrofluoric acid is required to etch the surface of ceramic restorations before repair.
-S
53-When required, which one of the following is the preferred lining material
below a composite resin restoration?
Al

a) Calcium hydroxide
in

b) Copal ether
c) Resin-modified glass ionomer cement “ CORRECT ANSWER
ss

d) Zinc oxide eugenol


e) Zinc polycarboxylate
Ya

• Comment on this Question

Resin-modified glass ionomer cement (RMGIC) is the preferred lining material from the
above. Composite resin can bond to RMGIC. Zinc oxide eugenol plasticises composite
and must not be used.

54-Which one of the following statements is correct?

a) Impression compound records more detail than alginate


b) Low viscosity silicones cannot be used in conjunction with an alginate base “
CORRECT ANSWER

49

Yassin Al-Safadi (Safadi92@hotmail.com)


c) Polyesters are the impression material of choice in patients with periodontal
disease
d) Polysulphides have a short setting time
e) Zinc oxide eugenol can be used in fully dentate impressions

• Comment on this Question

Alginate records more detail than impression compound. Zinc oxide eugenol is inflexible
and should not be used for dentate impressions due to difficulty in removal. Polyesters
are not as inflexible but can lock between periodontally involved teeth with recession and
interproximal spaces; hence they should not be used. Alginate should not be used in
conjunction with low viscosity silicones.

55-High-copper amalgams impart which of the following physical properties


compared with non-high-copper amalgams? Select one option only.

i
a) Increased corrosion

ad
b) Increased creep
c) Increased marginal breakdown
d) Increased tarnishing “ CORRECT ANSWER
af
e) Decreased speed to full compressive strength
-S
Al

• Comment on this Question


in

High-copper amalgams have a lower γ1 and γ2 phases. Therefore, they exhibit


decreased corrosion, marginal breakdown and creep. They exhibit increased
ss

compressive strength and speed at which they achieve full compressive strength.
However, they tarnish more.
Ya

56-Which one of the following is a component of mineral trioxide aggregate


(MTA)?

a) Bismuth oxide “ CORRECT ANSWER


b) Dicalcium aluminate
c) Tetracalcium silicate
d) Tricalcium aluminoferrate
e) Tristrontium silicate

• Comment on this Question

50

Yassin Al-Safadi (Safadi92@hotmail.com)


Composition of MTA includes: tricalcium silicate, dicalcium silicate, tricalcium aluminate,
tetracalcium aluminoferrate, gypsum and bismuth oxide. Bismuth oxide is added for
radio-opacity.

57-Which one of the following is correct regarding mineral trioxide aggregate


(MTA)?

a) Can only be used for retrograde root filling


b) It has the same composition as Portland cement
c) It is an ideal temporary restorative material
d) MTA bonds to gutta-percha
e) MTA is biocompatible with periradicular tissues “ CORRECT ANSWER

• Comment on this Question

MTA is very biocompatible. MTA has a similar composition to Portland cement but it

i
does not contain arsenic or lead. Bismuth oxide is added for radio-opacity. It is not an

ad
ideal temporary restorative material, as small quantities are very expensive.

af
MTA has a number of indications including: pulp capping, apexification, apexigenesis,
root resorption, perforation repair and retrograde root filling. MTA does not bond to gutta-
-S
percha.
Al

58-Which one of the following is a resorbable suture material?

a) Polybutester
in

b) Polyglactin “ CORRECT ANSWER


c) Polypropylene
ss

d) Silk
e) Staples
Ya

• Comment on this Question

Novafil™ is a non-absorbable monofilament formed of polybutester.

Prolene™ is a non-absorbable monofilament formed of polypropylene.

Staples and silk are non-absorbable.

Vicryl™ and Vicryl Rapide™ are absorbable, synthetic, braided sutures composed of
polyglactin.

59-Which one of the following statements regarding impression trays is correct?

a) Flexible impression trays improve impression accuracy


b) Metal special trays can be light cured for a command set

51

Yassin Al-Safadi (Safadi92@hotmail.com)


c) Perforations in impression trays improve mechanical retention of the impression
material “ CORRECT ANSWER
d) Spaced trays are required for zinc oxide eugenol impressions
e) Silicone impression materials do not require the use of fixative

• Comment on this Question

Zinc oxide eugenol impression must be taken in a close fitting tray.

Flexible impression trays lead to impression distortion and hence decreased accuracy.

Light cured composite can be used to make special trays.

Silicone impressions require the use of fixative, normally isopropylalcohol or


amylacetone.

i
ad
af
-S
Al
in
ss
Ya

52

Yassin Al-Safadi (Safadi92@hotmail.com)


[History and
Examination
i
ad
af
-S

MCQs]
Al
in
ss

[With answers]
Ya

53

Yassin Al-Safadi (Safadi92@hotmail.com)


1-Which one of the following signs is not associated with infective endocarditis?

a) Pulsus paradoxus « CORRECT ANSWER


b) Clubbing
c) Janeway lesions
d) Roth spots
e) Splinter haemorrhages

• Comment on this Question

Pulsus paradoxus describes the phenomenon of the heart rate and blood pressure falling during
inspiration. This usually occurs when there is a restriction of the ventricles, eg. Cardiac
tamponade or constrictive pericarditis. Roth spots are boat-shaped retinal haemorrhages with
pale centers seen on fundoscopy. Janeway lesions are painless palmar/plantar maculas.

2-Which one of the following statements regarding type-2 diabetes is true?

a) Cataract formation is uncommon in type-2 diabetes mellitus (DM)

i
ad
b) High Hb A1c readings correlate with good blood glucose monitoring (BM) control
c) Carpal tunnel syndrome is not associated with this disease
d) Insulin is the treatment of choice
af
e) Necrobiosis lipoidica may co-exist « CORRECT ANSWER
-S
• Comment on this Question
Al

Remember that diabetes is a multi-system disease and that patients require regular multi-
disciplinary team review - dieticians for dietary advice, ophthalmology for diabetic retinopathy
in

monitoring, endocrine/renal input for BM/HbA1c control, nephropathy review, peripheral pulse
ss

checks and autonomic/peripheral neuropathy screening. Peripheral neuropathy will place these
patients at increased risk of ulcers/infections and the difficult to treat ‘diabetic foot’. Good BM
Ya

control will also reduce the risk of cardiovascular disease. A balanced diet and oral
hypoglycaemics such as metformin/gliclazide are the treatment of choice. Necrobiosis lipoidica is
associated with diabetes and are yellow waxy plaques commonly found on the shins that are
surrounded by rusty margins.

3-Which one of the following symptoms is not associated with hyperthyroidism?

a) Palpitations
b) Weight loss
c) Tremor
d) Cold intolerance « CORRECT ANSWER
e) Anxiety

• Comment on this Question

54

Yassin Al-Safadi (Safadi92@hotmail.com)


Typically patients with hyperthyroidism complain of palpitations (atrial fibrillation), weight loss,
tremor, heat intolerance, diarrhoea and anxiety.

4-Which one of the following is true regarding neck swellings?

a) Sternocleidomastoid divides the neck into three triangular compartments


b) The presence of a thyroglossal cyst is best demonstrated by asking the patient to drink
and then observing
c) Cervical lymphadenopathy is pathognomonic for malignancy
d) Excision of parotid tumours may cause seventh cranial nerve palsy « CORRECT ANSWER
e) The presence of a goitre confirms the diagnosis of hyperthyroidism

• Comment on this Question

Sternocleidomastoid divides the neck into an anterior and posterior triangular compartment.
Thyroglossal cysts are embryonic remnants and are commonly found in the midline around the
hyoid bone. A thyroglossal cyst will move upwards from its resting position when the patient

i
protrudes their tongue. The cranial nerve (CN) VII may be damaged along its course if great care

ad
is not taken during excision of a parotid gland tumour. Goitres can be present in both hyper- and
hypothyroidism and further biochemical tests are required to confirm the diagnosis.
af
5-Which one of the following is not a 2ecognized cause of salivary gland enlargement?
-S
a) Bacterial infection
Al

b) Sarcoid
c) Sjögren’s syndrome
in

d) Diabetes mellitis
e) Sodium valproate « CORRECT ANSWER
ss

• Comment on this Question


Ya

Sodium valproate is not associated with salivary gland enlargement.

6-Which one of the following is the correct BLS sequence in assessing an unresponsive
patient?

a) Assess for danger, check airway, breathing, circulation, call for help, call for ambulance
b) Assess for danger, check responsiveness, call for help, assess airway, breathing,
circulation, call 999, start cardiopulmonary resuscitation (CPR)« CORRECT ANSWER
c) Assess for danger, check responsiveness, call 999, assess airway, breathing, circulation,
start CPR
d) Assess for danger, check responsiveness, call 999, assess airway, breathing, circulation,
and check patient notes
e) Assess for danger, check responsiveness, call 999, assess airway, breathing, circulation,
and give adrenaline

55

Yassin Al-Safadi (Safadi92@hotmail.com)


• Comment on this Question

Assess for danger, check responsiveness, call for help, assess airway, breathing, circulation, call
999, start cardiopulmonary resuscitation (CPR)

7-Which one of the following drugs is associated with gum hyperplasia?

a) Penicillamine
b) Nifedipine « CORRECT ANSWER
c) Levothyroxine
d) Cardivelol
e) Digoxin

8-Which one of the following conditions is a recognized cause of stomatitis?

a) Vitamin A deficiency
b) Vitamin D deficiency
c) Iron deficiency « CORRECT ANSWER

i
ad
d) Phenytoin
e) Polyarteritis nodosa
af
9-Which one of the following statements is true regarding temporal arteritis?
-S
a) Predominantly affects men
Al

b) May be managed conservatively


c) A normal erythrocyte sedimentation rate (ESR) excludes the diagnosis
d) A negative temporal artery biopsy excludes the diagnosis
in

e) High-dose steroids should be commenced before biopsy « CORRECT ANSWER


ss

• Comment on this Question


Ya

This disease predominantly affects elderly women (three times their male counterparts). The
patient may complain of fever, jaw pain, scalp pain on combing their hair, headache or visual
disturbance, if left untreated the patient may suffer permanent blindness. It is common to find
an erythrocyte sedimentation rate (ESR) > 60 mm/h but rarely may it be within the normal
range. Biopsy of the temporal artery is the investigation of choice but non-uniform infiltration of
giant cells may give a false negative biopsy on a small sample. Prednisolone 60 mg once daily
(OD) should be commenced immediately.

10-A patient presents with swan necking of their thumbs and ulnar deviation of their fourth
and fifth fingers. They have just been prescribed methotrexate. Which of the following
conditions do they have?

a) Dupuytren’s contracture
b) Osteoarthritis

56

Yassin Al-Safadi (Safadi92@hotmail.com)


c) Osteoporosis
d) Psoriatic arthritis
e) Rheumatoid arthritis « CORRECT ANSWER

11-A patient has a fixed flexion contracture of the palm, where the fingers bend towards the
palm and cannot be fully extended. What is the cause of this condition?

a) Dupuytren’s contracture « CORRECT ANSWER


b) Osteoarthritis
c) Osteoporosis
d) Psoriatic arthritis
e) Rheumatoid arthritis

• Comment on this Question

Dupuytren’s contracture is caused by underlying contractures of the palmar fascia. The ring
finger and little finger are the most commonly affected digits. The middle finger may be affected

i
in advanced cases, but the index finger and thumb are nearly always spared. Dupuytren’s

ad
contracture progresses slowly and is usually painless. In patients with this condition, the tissues
under the skin on the palm of the hand thicken and shorten so that the tendons connected to the
af
fingers cannot move freely.
-S
12-A patient presents with hearing loss and ill-fitting dentures, and says that his hat no longer
fits. Which of the following conditions is the most likely cause?
Al

a) Osteomyelitis
in

b) Osteopetrosis
c) Osteoporosis
ss

d) Osteosarcoma
e) Paget’s disease « CORRECT ANSWER
Ya

• Comment on this Question

Paget’s disease of bone is a condition in which the normal cycle of bone growth is disrupted. This
can cause bones to become weakened and deformed. Common symptoms of Paget’s disease
include bone pain and deformity.

13-Which disease is characterised by episodes of dyspnoea, cough and wheeze caused by


reversible airway obstruction?

a) Asthma « CORRECT ANSWER


b) Chronic obstructive pulmonary disease (COPD)
c) Fibrotic lung disease
d) Pulmonary embolism
e) Small-cell carcinoma of the lung

57

Yassin Al-Safadi (Safadi92@hotmail.com)


• Comment on this Question

Asthma affects all age groups, but often starts in childhood. The disease is characterised by
recurrent attacks of breathlessness and wheezing, which vary in severity and frequency from one
person to another. In an individual, they may occur from hour to hour and from day to day.

This condition is caused by inflammation of the air passages in the lungs, and it affects the
sensitivity of the nerve endings in the airways, which become easily irritated. During an asthma
attack the linings of the passages swell, causing the airways to narrow and reducing the flow of
air into and out of the lungs.

14-Ascites is normally caused by failure of which organ?

a) Heart
b) Kidney
c) Liver « CORRECT ANSWER
d) Lungs

i
e) Spleen

ad
• Comment on this Question
af
Rapidly developing (acute) ascites can occur as a complication of trauma, perforated ulcer,
-S
appendicitis, or inflammation of the colon or another tube-shaped organ (diverticulitis). This
condition can also develop when intestinal fluids, bile, pancreatic juices or bacteria invade or
Al

inflame the smooth transparent membrane (peritoneum) that lines the inside of the abdomen.
However, ascites is more often associated with liver disease and other long-lasting (chronic)
in

conditions.
ss

15-With which eponymous syndrome is koilonychia associated?


Ya

a) Gardner syndrome
b) Gaucher syndrome
c) Gorlin–Goltz syndrome
d) Plummer–Vinson syndrome « CORRECT ANSWER
e) Sturge–Weber syndrome

• Comment on this Question

Plummer–Vinson syndrome (PVS) presents as a triad of dysphagia (due to oesophageal webs),


glossitis and iron-deficiency anaemia. Symptoms include:

• dysphagia

• pain

• weakness

58

Yassin Al-Safadi (Safadi92@hotmail.com)


• odynophagia

• atrophic glossitis

• angular stomatitis.

16-In which condition are Roth spots, Osler’s nodes and Janeway’s lesions seen?

a) Infective endocarditis « CORRECT ANSWER


b) Osteoarthritis
c) Rheumatoid arthritis
d) Type 1 diabetes mellitus
e) Type 2 diabetes mellitus

• Comment on this Question

Infective endocarditis.

i
• Splinter haemorrhages are dark red linear lesions in the nail beds.

ad
• Osler’s nodes are tender subcutaneous nodules usually found on the distal pads of the
digits. af
-S
• Janeway’s lesions are non-tender maculae on the palms and soles.
Al

• Roth spots are retinal haemorrhages with small, clear centres. They are rare, being
observed in only 5% of patients.
in

17-You are examining a patient in the dental chair, and when they protrude their tongue it
deviates to the right. Which nerve is failing to function?
ss

a) Accessory nerve
Ya

b) Facial nerve
c) Glossopharyngeal nerve
d) Hypoglossal nerve « CORRECT ANSWER
e) Trigeminal nerve

• Comment on this Question

Disorders of the 12th cranial nerve (hypoglossal nerve), which is responsible for moving the
tongue, cause weakness and/or atrophy (wasting) of the tongue on the affected side. Causes of
loss of function of the hypoglossal nerve include a tumour or bone abnormality at the base of the
skull, a stroke, infection of the brainstem, or an injury to the neck. As a result of the weakness
and/or atrophy of the tongue, individuals with this disorder have difficulty speaking, chewing
and swallowing.

59

Yassin Al-Safadi (Safadi92@hotmail.com)


18-A patient with active tuberculosis who has been admitted under your care is concerned
about the fact that they are passing red-orange urine. Which of the following drugs is likely to
be the cause of this?

a) Ethambutol
b) Isoniazid
c) Pyrazinamide
d) Rifampicin « CORRECT ANSWER
e) The cause is unlikely to be drug related

• Comment on this Question

Rifampicin is a solid with an intense red colour, and the small fraction of this antibiotic that
reaches the body fluids is known to impart a harmless red-orange color to the urine (and to a
lesser extent to the sweat and tears) for a few hours after a dose has been taken. Maximal
concentrations in the blood are decreased by about a third when the antibiotic is taken with
food.

i
ad
19-Which muscle is one of the borders of the submandibular triangle?

a) Digastric « CORRECT ANSWER


af
b) Hyoglossus
-S
c) Sternocleidomastoid
d) Sternohyoid
Al

e) Thyrohyoid
in

• Comment on this Question


ss

Digastric.
Ya

• The submandibular triangle is bounded above by the lower border of the body of the
mandible, and a line drawn from its angle to the mastoid process.

• It is bounded below by the posterior belly of the digastric muscle, and in front by the
anterior belly of the digastric muscle.

20-A patient with chronic facial pain has been referred to you because they have developed
anaemia and their liver function tests are deranged. Which of the following drugs is the likely
cause?

a) Amitriptyline
b) Amoxicillin
c) Carbamazepine « CORRECT ANSWER
d) Dothiepin
e) Gabapentin

60

Yassin Al-Safadi (Safadi92@hotmail.com)


• Comment on this Question

It is important that carbamazepine is used carefully, and close clinical and frequent laboratory
supervision should be maintained throughout treatment.

Although reported infrequently, serious adverse effects have been observed during
carbamazepine use. Agranulocytosis and aplastic anaemia, with a fatal outcome, have been
documented in a few cases.

21-Which of the following muscles does not have a motor supply from the mandibular branch
of the trigeminal nerve?

a) Anterior belly of digastric


b) Masseter
c) Posterior belly of digastric « CORRECT ANSWER
d) Tensor veli palatini
e) Tensor tympani

i
ad
• Comment on this Question

The posterior belly, which is longer than the anterior belly, arises on the inferior surface of the
af
skull, from the mastoid notch on the medial surface of the mastoid process of the temporal bone
-S
and a deep groove between the mastoid process and the styloid process, called the digastric
groove.
Al

The posterior belly is supplied by the digastric branch of the facial nerve.
in

The digastric muscle extends from the mastoid process of the cranium to the mandible at the
chin, and part of the way between these it becomes a tendon, which passes through a tendon
ss

pulley attached to the hyoid bone.


Ya

22-A young child who has chickenpox (varicella) has a red rash and blisters/vesicles within
their ear and eardrum. What is the name of this syndrome?

a) Albright syndrome
b) Cushing syndrome
c) Frey syndrome
d) Patterson–Brown-Kelly syndrome
e) Ramsay Hunt syndrome « CORRECT ANSWER

• Comment on this Question

Ramsay Hunt syndrome is a herpes zoster virus infection of the geniculate ganglion of the facial
nerve. It is caused by reactivation of herpes zoster virus that has previously caused chickenpox in
the patient. Ramsay Hunt syndrome results in paralysis of the facial muscles on the same side of
the face as the infection, as the virus infects the facial nerve that normally controls the muscles

61

Yassin Al-Safadi (Safadi92@hotmail.com)


of the face. Ramsay Hunt syndrome is typically associated with a red rash and blisters in or
around the ear and eardrum, and sometimes on the roof of the mouth or the tongue.

23-A patient has had a neck dissection as part of their treatment for an oral malignancy. They
now have weakness of their trapezius and sternocleidomastoid muscles on that side, and are
unable to shrug their shoulders equally. Which nerve has been damaged?

a) Accessory nerve « CORRECT ANSWER


b) Facial nerve
c) Radial nerve
d) Ulnar nerve
e) Vagus nerve

• Comment on this Question

The trapezius muscle is tested by asking the patient to shrug their shoulders with and without
resistance. A one-sided weakness is indicative of an injury to the spinal accessory nerve on the

i
same side of the body as that being assessed.

ad
The sternocleidomastoid muscle is tested by asking the patient to turn their head to the left or
af
right against resistance. A weak leftward turn of the head is indicative of a weak right
sternocleidomastoid muscle (and thus right spinal accessory nerve injury), whereas a weak
-S
rightward turn of the head is indicative of a weak left sternocleidomastoid muscle (and thus left
spinal accessory nerve injury).
Al

24-A patient had a superficial parotidectomy 6 weeks ago. They report that an area of their
in

cheek on the same side becomes red and sweats at mealtimes. Which nerve has been
damaged?
ss

a) Auriculotemporal branch of the trigeminal nerve « CORRECT ANSWER


Ya

b) Buccal branch of the facial nerve


c) Mandibular branch of the trigeminal nerve
d) Temporal branch of the facial nerve
e) Zygomatic branch of the facial nerve

• Comment on this Question

Frey syndrome often occurs as a side-effect of parotid gland surgery. The auriculotemporal
branch of the trigeminal nerve carries sympathetic fibres to the sweat glands of the scalp and
parasympathetic fibres to the parotid gland. As a result of severance and inappropriate
regeneration, the fibres may switch courses, resulting in ‘gustatory sweating’ (sweating in
response to a food stimulus) instead of the normal salivatory response.

25-How many branches does the internal carotid artery give off as it travels from its
bifurcation to the carotid canal?

62

Yassin Al-Safadi (Safadi92@hotmail.com)


a) 0 « CORRECT ANSWER
b) 2
c) 3
d) 4
e) 7

• Comment on this Question

The internal carotid artery has no branches in this region.

26-Which of the following statements about the digastric muscle is INCORRECT?

a) The anterior belly of the digastric is innervated by the hypoglossal nerve. « CORRECT
ANSWER
b) The digastric depresses the mandible and raises the hyoid bone.
c) The posterior belly of the digastric is innervated by the facial nerve.
d) The superior attachment of the anterior belly of the digastric is the digastric fossa of the

i
mandible.

ad
e) The superior attachment of the posterior belly of the digastric is the mastoid notch of
the temporal bone. af
• Comment on this Question
-S
The digastric muscle consists of two fleshy bellies united by an intermediate rounded tendon. The
Al

two bellies are supplied by different cranial nerves.

• The posterior belly is supplied by the digastric branch of the facial nerve.
in

• The anterior belly is supplied by the trigeminal nerve via the mylohyoid nerve, a branch
ss

of the inferior alveolar nerve, which is itself a branch of the mandibular division of the trigeminal
Ya

nerve.

27-A patient has oral lichen planus, and some small, shiny, flat-topped, firm papules are
present on their wrists. The papules are purple in colour and are crossed by fine white lines.
What are these lesions called?

a) Candidiasis
b) Drug reaction
c) Psoriasis
d) Waldenström’s macroglobulinaemia
e) Wickham’s striae « CORRECT ANSWER

• Comment on this Question

These are characteristic fine white or grey lines or dots seen on the top of the pruritic papular
rash of lichen planus, and they also affect the oral mucosa in the same disease.

63

Yassin Al-Safadi (Safadi92@hotmail.com)


28-A patient presents to your clinic with oral ulceration. They have started five new drugs
recently. Which drug is most likely to have caused the oral ulceration?

a) Aspirin
b) Bendroflumethiazide
c) Diltiazem « CORRECT ANSWER
d) Fluoxetine
e) Propranolol

• Comment on this Question

The use of diltiazem commonly leads to oral ulceration

29-A patient presents to your practice. They tell you that they have recently had an operation
for oral cancer, which involved taking a free flap from their arm for reconstruction. They are
now experiencing some numbness of the skin over their thumb. Which nerve has been
affected?

i
ad
a) Digital nerve
b) Median nerve
c) Nerve to C4
af
d) Radial nerve « CORRECT ANSWER
-S
e) Ulnar nerve
Al

• Comment on this Question

The radial nerve and its branches supply the dorsal muscles, such as the triceps, the extrinsic
in

extensors of the wrist and hand, and the cutaneous nerve supply to most of the back of the hand.
ss

30-On examination you feel a click on opening of the left jaw joint. There is no associated pain
Ya

and the patient thinks it may have been present to for a long time. What is the likely
diagnosis?

a) Anterior displaced disc « CORRECT ANSWER


b) Atypical facial pain
c) Rheumatoid arthritis
d) Temporomandibular joint dysfunction
e) Trigeminal neuralgia

• Comment on this Question

Temporomandibular joint dysfunction is associated with pain in the muscles of mastication and
can be associated with bruxism or occlusal problems. Rheumatoid arthritis can cause breakdown
of the joint which can have a click but is usually associated with pain. Again, atypical facial pain
is associated with pain that has no known cause. Trigeminal neuralgia usually has a trigger point

64

Yassin Al-Safadi (Safadi92@hotmail.com)


causing extensive pain. Anterior displaced disc is common and the click occurs when the disc slips
past the joint on opening and translating forwards.

31-After receiving an inferior dental block, the patient finds her eye starts watering and
complains of difficulty blinking. Which branch of which nerve is most likely to have been
affected?

a) Cervical branch of facial nerve


b) Mandibular branch of facial nerve
c) Maxillary branch of trigeminal nerve
d) Ophthalmic branch of trigeminal nerve
e) Zygomatic branch of facial nerve « CORRECT ANSWER

• Comment on this Question

The facial nerve enters the parotid duct and splits into five main branches. These include the
marginal mandibular, cervical, temporal, zygomatic and buccal. The zygomatic branch

i
innervates the muscles of facial expression around the eye. In giving an inferior dental block, if it

ad
is given too high or there is diffusion to the facial nerve it can affect the facial nerve.
af
32-A patient complains of a beefy tongue, recurrent ulcers and soreness at the corners of her
mouth. What is the likely diagnosis?
-S
a) Angular chelitis
Al

b) Lack of intrinsic factor


c) Macrocytic anaemia
in

d) Microcytic hypochromic anaemia « CORRECT ANSWER


e) Minor recurrant aphthous stomatitis
ss

• Comment on this Question


Ya

Angular chelitis is a fungal infection that affects the corners of the mouth. This is a side effect of
microcytic anaemia. Minor recurrant aphthous stomatitis is multiple ulcers that can also be a
symptom of anaemia. The beefy tongue, recurrent ulcers and minor aphthae are all indicators of
microcytic anaemia from a lack of Iron.

33-A 55-year-old woman is taking alendronic acid. What is the most likely reason to be taking
it?

a) Breast cancer
b) Leukaemia
c) Multiple myeloma
d) Osteogenesis imperfecta
e) Osteoporosis « CORRECT ANSWER

65

Yassin Al-Safadi (Safadi92@hotmail.com)


• Comment on this Question

Alendronic acid is a bisphosphonate; these are used for many bone conditions as they affect
osteoclasts and bone turnover. Intravenous bisphosphonates are used for multiple myeloma and
breast cancer. They are used in children in osteogenesis imperfecta. They are most commonly
used in lower doses for women with osteoporosis and in patients with Paget’s disease. They are
not used in leukaemia.

34-During a new patient dental examination you find a large bony lump in the palate. The
mucosa is pale pink, non-ulcerated and keratinised. It is firm to touch and fixed. The patient is
unaware of how long it has been present and it is not causing any pain. What is the likely
diagnosis?

a) Kaposi’s sarcoma
b) Pleomorphic adenoma
c) Repair of a cleft palate
d) Squamous cell carcinoma

i
ad
e) Torus palatini « CORRECT ANSWER

• Comment on this Question af


Kaposi’s sarcoma is a tumour caused by human herpes virus 8. It often presents in the palate but
-S
is not a bony lump and often appears as a purplish bruises. A torus palatini is a bony growth in
the palate for which no treatment is necessary. A cleft palate repair is unlikely to show the
Al

appearance of a growth in the palate but more as a defect. A squamous cell carcinoma is rare to
present in the palate and usually has the appearance of a rolled unhealing ulcerated growth. A
in

pleomorphic adenoma is a benign salivary gland tumour that is most commonly found in the
ss

parotid, although it can present in the palate. It has the appearance of a firm singular nodular
mass but is not normally bony.
Ya

35-What nerve carries parasympathetic innervation to the submandibular salivary gland?

a) Chorda tympani carried by facial nerve


b) Chorda tympani carried by lingual nerve « CORRECT ANSWER
c) Chorda tympani carried by mandibular branch of trigeminal nerve
d) Hypoglossal nerve
e) Lingual nerve

• Comment on this Question

The parasympathetic innervation originates from the facial nerve but branches through the
petrotympanic fissure and enters the intratemporal fossa. It then joins the lingual nerve before
synapsing in the submandibular ganglion before passing to the submandibular gland itself.

66

Yassin Al-Safadi (Safadi92@hotmail.com)


36-A patient attends who is taking warfarin. She has a consistently high INR and multiple
carious lesions. On taking a diet sheet you discuss issues affecting the caries rate. Which one
of the following foodstuffs is most likely to be causing a high INR?

a) Coffee with three sugars


b) Chocolate
c) Cranberry juice « CORRECT ANSWER
d) Grapefruit juice
e) Strawberry jam on toast

• Comment on this Question

Cranberry juice has been known to increase the potency of warfarin. Ginger is also known to
affect warfarin. There have been some investigations into grapefruit juice but has not been
proven to increase the INR. Strawberry jam has a high titrated pH but does not affect warfarin.
Chocolate and coffee do not interfere with warfarin.

i
37-What spaces are most commonly affected with an infected maxillary molar tooth?

ad
a) Buccal space and maxillary sinus
b)
af
Buccal space, oral cavity and maxillary sinus « CORRECT ANSWER
c) Maxillary sinus, buccal space and parapharyngeal space
-S
d) Parapharyngeal space and buccal space
e) Pterygomandibular space, oral cavity and submasseteric space
Al

• Comment on this Question


in

A molar tooth when infected is most likely to spread through the thin buccal and palate plates
ss

into the oral mucosa. The roots are often close to the maxillary sinus and therefore can spread
into the sinus. If the infection passes behind the buccinator it can affect the buccal space as well.
Ya

It can affect the parapharyngeal space, but this is less common. The pterygomandibular space
and submasseteric space are more commonly affected with mandibular teeth.

38-You are looking after a patient on the ward who has been admitted for extraction of teeth
later today following an abscess. They have been given a course of penicillin for management
of the abscess. They get up to go and get changed and they collapse. On examination they
have a rash spreading across their chest and are finding it difficult to breathe. What drugs, and
in what correct order, are you going to administer within the hospital setting?

a) Call for help, 0.5 ml 1:1000 adrenaline im, high flow oxygen (15 l/min), chlorphenamine
10 mg iv, hydrocortisone 100 mg iv« CORRECT ANSWER
b) Call for help, 0.5 ml 1:1000 adrenaline im, high flow oxygen (15 l/min), hydrocortisone
100 mg iv, chlorphenamine 10 mg iv
c) Call for help, 0.5 ml 1:10,000 adrenaline im, high flow oxygen (15 l/min),
chlorphenamine 10 mg iv, hydrocortisone 100 mg iv

67

Yassin Al-Safadi (Safadi92@hotmail.com)


d) Call for help, chlorphenamine 10 mg iv, 0.5 ml 1:1000 adrenaline im, high flow oxygen
(15 l/min), hydrocortisone 100 mg iv
e) Call for help, high flow oxygen (15 l/min), 0.5 ml 1:1000 adrenaline im, chlorphenamine
10 mg iv, hydrocortisone 100 mg iv

• Comment on this Question

Diagnosis of anaphylactic attack. It is important to ensure help is on its way, as it can be fatal.
The adrenaline is important to administer first as it will reopen the airway and reduce the
oedema. High flow oxygen is then important to maintain the airway. Chlorphenamine will help
counteract the histamine that is being produced causing the oedema but takes longer to take
effect. The hydrocortisone will act as an anti-inflammatory and immunosuppressant to prevent
the swelling developing further but takes time to act. During this time another dose of
adrenaline may be necessary.

i
ad
af
-S
Al
in
ss
Ya

68

Yassin Al-Safadi (Safadi92@hotmail.com)


[ Human
Diseases
i
ad
af
-S

MCQs]
Al
in
ss

[With Answers]
Ya

69

Yassin Al-Safadi (Safadi92@hotmail.com)


1-Which one of the following is a disease associated with trisomy 18?

a) Down’s syndrome
b) Patau’s syndrome
c) Cri-du-chat syndrome
d) Edward’s syndrome « CORRECT ANSWER
e) CREST (calcinosis cutis, Raynaud’s phenomenon, oesophophageal dysfunction,
sclerodactyly and telangiectasia syndrome) syndrome

•Comment on this Question

Edward's syndrome is a genetic disorder caused by chromosome 18 in a foetus or infant's cells.


Incidence increases relative to the mother's age, and is estimated at 1 in 3,000 live births. It has
a very low rate of survival.

2-Which one of the following is not a radiographic finding of osteoarthritis?

a) Reduced joint space

i
ad
b) Periarticular cysts
c) Osteophytes
d) Osteopenia « CORRECT ANSWER
af
e) Articular sclerosis
-S
3-A young man presents with bloody diarrhoea, weight loss and a swollen lower lip. Which
Al

one of the following is the diagnosis?

a) Coeliac disease
in

b) Ulcerative colitis
ss

c) Crohn’s disease « CORRECT ANSWER


d) Gastroenteritis
Ya

e) IBS

•Comment on this Question

Crohn’s is a disease with skip lesions throughout out the gastrointestinal (GI) tract that has a
known history of lip swelling.

4-Which one of the following is a pneumonia-causing organism that is found in stagnant


water?

a) Staphylococcus aureus
b) Legionella « CORRECT ANSWER
c) Mycoplasma
d) Pneumocystis jiroveci
e) Chlamydia psittaci

70

Yassin Al-Safadi (Safadi92@hotmail.com)


•Comment on this Question

This bacterium is common in water tanks, which are kept at 60o C and is seen in travellers,
especially to Spain. It presents with a dry cough, bibasal consolidation and myalgia.

5-Which one of the following is a pneumonia-causing organism associated with AIDS?

a) Staphylococcus aureus
b) Legionella
c) Mycoplasma
d) Pneumocystis jiroveci « CORRECT ANSWER
e) Chlamydia psittaci

•Comment on this Question

The organism was previously called Pneumocystis carinii and is now Pneumocystis jiroveci.

6-A 70-year-old patient presents 17 days post-operatively for a major resection of oral cancer

i
ad
with fever, reduced level of consciousness and left basal crepitations. He has been
percutaneous endoscopic gastrostomy (PEG) fed post-operatively without complication on the
ward. Which one of the following is the likely diagnosis?
af
-S
a) Pulmonary embolus
b) Aspiration pneumonia
Al

c) Heart failure
d) Hospital-acquired pneumonia « CORRECT ANSWER
e) Pulmonary oedema
in
ss

7-Which one of the following is not a cause of atrial fibrillation?

a) Pulmonary embolus
Ya

b) Pregnancy
c) Alcoholism
d) Cushing’s disease « CORRECT ANSWER
e) Hyperthyroidism

8-A 65-year-old lady presents with a ‘burning sensation’ in her left hand in the thumb, index
and middle finger, that wakes her at night. Which one of the following nerves is affected?

a) Ulnar
b) Median « CORRECT ANSWER
c) Radial
d) Digital
e) Brachial plexus

71

Yassin Al-Safadi (Safadi92@hotmail.com)


•Comment on this Question

The lady has carpel tunnel syndrome due to compression of the median nerve therein.

9-A student presents with a headache that is the worst he has ever had on waking in the
morning after the night out. His neck feels stiff and he feels nauseous. He has a history of
polycystic kidney disease. Which one of the following is the likely diagnosis?

a) Hangover
b) Subarachnoid haemorrhage « CORRECT ANSWER
c) Meningitis
d) Migraine
e) Extra-dural haemorrhage

i
ad
•Comment on this Question af
Presents with worse headache ever and sudden onset with neck stiffness.
-S
10-You suspect a patient has a subarachnoid haemorrhage. You will make a diagnosis based
Al

on which of the following urgent tests?

a) Magnetic resonance imaging (MRI)


in

b) Clotting screen
ss

c) Computerised tomography (CT) scan « CORRECT ANSWER


d) Lumbar puncture
Ya

e) ECG

•Comment on this Question

A computerised tomography (CT) scan is used to assess for bleed.

11-A 21-year-old man presents to the Emergency Department. He has been punched in the
side of the head over the left temple. He was knocked out, but regained consciousness. After
some time in the Emergency Department he starts to become confused and irritable. Which
one of the following is the most likely diagnosis?

a) Subarachnoid haemorrhage
b) Extradural haemorrhage « CORRECT ANSWER
c) Concussion
d) Subdural bleed
e) Hypoglycaemia

72

Yassin Al-Safadi (Safadi92@hotmail.com)


•Comment on this Question

Lucid period after a knock-out blow is diagnostic.

12-A patient with recurrent oral cancer presents with brain metastasis, which one of the
following is part of the Cushing’s response to raised intra-cranial pressure?

a) Raised blood pressure « CORRECT ANSWER


b) Raised pulse
c) Down going plantar reflexes
d) Increased respiratory rate
e) Raised p(O2)

•Comment on this Question

Presents with hypertension and bradycardia.

13-Which one of the following options is not a recognised trigger of migraine?

i
ad
a) Chocolate
b) Alcohol af
c) Early mornings « CORRECT ANSWER
-S
d) Fear
e) Exercise
Al
in
ss

•Comment on this Question


Ya

Early mornings are not a recognised cause, but fatigue is.

14-Which one of the following medications does not cause bleeding?

a) Asprin
b) Clopidogrel
c) Ibuprofen
d) Alcohol
e) Prednisolone « CORRECT ANSWER

•Comment on this Question

73

Yassin Al-Safadi (Safadi92@hotmail.com)


Steroids lead to tissue fragility, so patients are more likely to bleed, but not bleed
spontaneously.

15-A known alcoholic is seen with confusion and hallucinations. On examination he is sweaty,
flushed, tachycardic and agitated. From the following options choose one that explains what
is occurring?

a) Alcohol intoxication
b) Subdural haemorrhage secondary to a fall
c) Wernicke’s encephalopathy
d) Delirium tremens « CORRECT ANSWER
e) Korsakoff’s psychosis

•Comment on this Question

Watch for hypotension.

16-Which one of the following options is not associated with alcohol abuse?

i
ad
a) Folate deficiency
b) Arrhythmias af
c) Cortical atrophy
-S
d) Microcytic anaemia « CORRECT ANSWER
e) Falls
Al
in
ss

•Comment on this Question


Ya

Macrocytic anaemia is due to folate deficiency.

17Which one of the following is not a sign and symptom of hypothyroidism?

a) Slow relaxing reflexes


b) Peaches and cream complexion
c) Depression
d) Lethargy
e) Lid lag « CORRECT ANSWER

•Comment on this Question

74

Yassin Al-Safadi (Safadi92@hotmail.com)


Lid lag is a sign of thyroid toxicosis.

18-Which one of the following is associated with McCune–Albright syndrome?

a) Syphilis
b) Fibrous dysplasia « CORRECT ANSWER
c) Ectodermal dysplasia
d) Delayed puberty
e) Oral cancer

•Comment on this Question

It is a syndrome associated with fibrous dysplasia, precocious puberty, skin pigmentation and
endocrine diseases.

i
ad
19-A man presents with persistent refractory hypertension, hypokalaemia and a unilateral
adrenal mass. Which one of the following diseases does he suffer?
af
a) Cushing’s disease
-S
b) Conn’s disease « CORRECT ANSWER
c) Phaeochromocytoma
Al

d) Addison’s
e) Congenital hyperaldosteronism
in
ss
Ya

•Comment on this Question

Conn’s disease is a unilateral adrenal mass that releases aldosterone and is normally amenable
to surgery

20-A 19-year-old man present to your practice. He has a history of recurrent chest infections,
wheeze and a positive sweat test. Which one of the following is the likely diagnosis?

a) Asthma
b) Bronchiectasis
c) AIDS
d) Cystic fibrosis « CORRECT ANSWER
e) Sarcoidosis

75

Yassin Al-Safadi (Safadi92@hotmail.com)


•Comment on this Question

A common disease affecting 1:2000 live births that affects Caucasians.

21-A 45-year-old female patient collapses in your surgery while waiting to have a tooth
extracted. She was extremely anxious for this procedure. She has a medical history of asthma
for which she frequently uses her brown inhaler (lately she has been using the inhaler more
frequently). On examination she is very hypotensive. Which one of the following best
describes what has occurred?

a) Vasovagal « CORRECT ANSWER


b) Addison’s crisis
c) Hypoglycaemic
d) Acute asthma attack
e) Anaphylaxis

i
ad
•Comment on this Question

Patients who suffer from anxiety are often prone to suffering from vasovagal attacks. During
af
such an attack, they are likely to be hypotensive until emergency intervention is carried out.
-S
22-A 16-year-old male patient presents with a dental abscess. He has a history of recent
Al

weight loss, abdominal pain, thirst and frequency of micturation. Which one of the following
is of concern?
in

a) Diabetes insipidus
b) HIV
ss

c) Diabetes mellitus « CORRECT ANSWER


Ya

d) Urinary tract infection


e) Acute renal failure

•Comment on this Question

This adolescent is presenting with classic signs of diabetes.

23-An unwell 16-year-old with diabetes has a blood glucose monitoring (BM) of 16. He needs
an important test. Which one of the following is the suitable firstline investigation?

a) Full blood count


b) Urea and electrolytes (U&Es)
c) Electrocardiogram (ECG)
d) Urine dipstick
e) Blood gas « CORRECT ANSWER

76

Yassin Al-Safadi (Safadi92@hotmail.com)


•Comment on this Question

This child could have diabetic ketoacidosis. A blood gas will show the pH, but urine will show
presence of ketones. The blood gas test is more important in the sick patient.

Revision point: Blood gases in the very young can be complicated.

24-Which one of the following is a syndrome of dysphagia and pain on mastication with
elongated styloid processes?

a) Eagle’s syndrome « CORRECT ANSWER


b) Frey’s syndrome
c) Gardner’s syndrome
d) Gorlin–Goltz syndrome
e) Crouzon’s syndrome

•Comment on this Question

i
ad
This syndrome also shows characteristic pain on head turning.

25-Which one of the following syndromes typically includes craniosynostosis?


af
a) Eagle’s syndrome
-S
b) Frey’s syndrome
c) Gardner’s syndrome
Al

d) Gorlin–Goltz syndrome
e) Crouzon’s syndrome « CORRECT ANSWER
in

•Comment on this Question


ss

Crouzon’s syndrome is an autosomal dominant condition consisting of premature cranial suture


Ya

fusion, mid-face hypoplasia with proptosis. The brain is trapped in the fused skull leading to
raised pressure.

26-Which one of the following is a disease typified by multiple osteomas, epithelial cysts and
intestinal polyps?

a) Eagle’s syndrome
b) Frey’s syndrome
c) Gardner’s syndrome « CORRECT ANSWER
d) Gorlin–Goltz syndrome
e) Crouzon’s syndrome

•Comment on this Question

The polyps require surveillance as they have a high risk of malignant chance in the large
intestine..

77

Yassin Al-Safadi (Safadi92@hotmail.com)


27-Which one of the following syndromes is not associated with lowered IQ?

a) Sturge–Weber syndrome
b) Treacher Collin’s syndrome « CORRECT ANSWER
c) Cri-du-chat syndrome
d) Crouzon’s syndrome
e) Edward’s syndrome

•Comment on this Question

Treacher Collin’s syndrome is a facial deformity secondary to a first brachial arch defect. It does
not affect intelligence!

28-Which one of the following can occur after superficial parotidectomy?

i
ad
a) Eagle’s syndrome
b) Frey’s syndrome « CORRECT ANSWER af
c) Gardner’s syndrome
-S
d) Gorlin–Goltz syndrome
e) Crouzon’s syndrome
Al

•Comment on this Question


in

Symptoms include gustatory sweating, erythaema and pain as parasympathetic fibres from the
parotid ‘rewire’ to innervate the skin.
ss

29-Which one of the following is a syndrome associated with odontogenic keratocysts and
Ya

basal cell carcinoma?

a) Eagle’s syndrome
b) Frey’s syndrome
c) Gardner’s syndrome
d) Gorlin–Goltz syndrome « CORRECT ANSWER
e) Crouzon’s syndrome

•Comment on this Question

Symptoms include basal cell carcinoma (BCC), multiple odontogenic keratocysts, calcified falx
cerebri and bifid ribs and cervical ribs.

78

Yassin Al-Safadi (Safadi92@hotmail.com)


30-Which one of the following is not associated with Horner’s syndrome?

a) Exophthalmus « CORRECT ANSWER


b) Miosis
c) Anhidrosis of the affected facial half
d) Pancoasts tumour
e) Sympathetic nerve supply

•Comment on this Question

Also associated with enophthalmos.

31-Which of the following medications is administered by injection in the treatment of


diabetes mellitus?

a) Acarbose
b) Glibenclamide
c) Insulin « CORRECT ANSWER

i
ad
d) Metformin
e) Rosiglitazone

•Comment on this Question


af
-S
Insulin is injected, whereas all the other drugs are oral medications.
Al

32-An elderly woman presents with pain, numbness and tingling over the lateral digits of the
hand. She has hypothyroidism and has previously had surgery to her left wrist. What is the
in

diagnosis?
ss

a) Carpal tunnel syndrome « CORRECT ANSWER


b) Fractured radius
Ya

c) Morton’s neuroma
d) Radial nerve compression
e) Trigger finger

•Comment on this Question

Carpal tunnel syndrome is a clinical diagnosis. The signs and symptoms are caused by
compression of the median nerve travelling through the carpal tunnel.

The symptoms of this condition include pain, paraesthesias and occasionally weakness in the
median nerve distribution. Patients with carpal tunnel syndrome often experience pain,
numbness and tingling sensations in the arm, and sometimes extending to the shoulder and
neck area.

79

Yassin Al-Safadi (Safadi92@hotmail.com)


33-A patient has a severe head injury and suffers damage to the foramen ovale. Which of the
following nerves may be damaged?

a) Facial nerve
b) Hypoglossal nerve
c) Mandibular division of the trigeminal nerve « CORRECT ANSWER
d) Maxillary division of the trigeminal nerve
e) Ophthalmic division of the trigeminal nerve

•Comment on this Question

The contents of the foramen ovale consist of the otic ganglion, the mandibular division of the
trigeminal nerve, the accessory meningeal artery, the lesser petrosal nerve and emissary veins.

34-A patient has a severe head injury and suffers damage to the foramen rotundum. Which of
the following nerves may be damaged?

a) Facial nerve

i
ad
b) Hypoglossal nerve
c) Mandibular division of the trigeminal nerve
d)
af
Maxillary division of the trigeminal nerve « CORRECT ANSWER
e) Ophthalmic division of the trigeminal nerve
-S
•Comment on this Question
Al

The foramen rotundum is one of several circular apertures (the foramina) located in the base of
the skull, in the anterior and medial part of the sphenoid bone, and the maxillary division of the
in

trigeminal nerve passes through it.


ss

35-Which of the following hormones is released by the posterior part of the pituitary gland?
Ya

a) ACTH
b) FSH
c) GH
d) Oxytocin « CORRECT ANSWER
e) TSH

•Comment on this Question

• The hormones released by the posterior pituitary are vasopressin and oxytocin.

• The hormones released by the anterior pituitary are follicle-stimulating hormone (FSH),
luteinising hormone (LH), adrenocorticotropic hormone (ACTH), thyroid-stimulating hormone
(TSH), melanocyte-stimulating hormone (MSH) and prolactin.

80

Yassin Al-Safadi (Safadi92@hotmail.com)


36-With deficiency of which vitamin are rickets (in children) and osteomalacia (in adults)
associated?

a) Vitamin A
b) Vitamin B12
c) Vitamin C
d) Vitamin D « CORRECT ANSWER
e) Vitamin E

•Comment on this Question

Rickets is a lack of mineralisation of bones in children, which potentially leads to fractures and
deformity. It is among the most frequent childhood diseases in many developing countries. The
most common cause is vitamin D deficiency. Although it can occur in adults, the majority of
cases occur in children from areas of the world where famine is prevalent.

37-When you examine a 6-year-old child, you observe Koplik spots on their buccal mucosa.

i
What childhood infection do they have?

ad
a) Diphtheria
b) Herpes simplex 1
af
c) Measles « CORRECT ANSWER
-S
d) Mumps
e) Varicella
Al

•Comment on this Question


in

The classical symptoms of measles are a four-day fever with cough, conjunctivitis and coryza.
ss

Koplik’s spots on the inside of the mouth are pathognomonic of measles.


Ya

38-A 60-year-old woman cut her arm while gardening 10 days ago. She presents to the
Emergency Department with neck stiffness, trismus, facial spasms, dysphagia and pyrexia.
What is the diagnosis?

a) Dental abscess
b) Meningitis
c) Rubella
d) Tetanus « CORRECT ANSWER
e) Tonsillitis

•Comment on this Question

The first signs of tetanus are trismus (lockjaw) and risus sardonicus (facial spasms that produce a
distorted grinning expression), followed by stiffness of the neck, difficulty in swallowing, and

81

Yassin Al-Safadi (Safadi92@hotmail.com)


rigidity of the pectoral and calf muscles. Other symptoms may include elevated temperature,
sweating, elevated blood pressure and rapid heart rate.

39-An elderly man who drinks 50 units of alcohol a week has a red, painful, stiff big toe. He is
afebrile. He has previously taken colchicine for this condition. What is the likely cause?

a) Cellulitis
b) Gout « CORRECT ANSWER
c) Osteoarthritis
d) Septic arthritis
e) Vascular insufficiency

•Comment on this Question

Gout is a medical condition that is usually characterised by a red, tender, hot, swollen joint. The
big toe is most commonly affected, accounting for around 50% of all cases. Gout is caused by
elevated blood levels of uric acid, which crystallises and is deposited in joints, tendons and the

i
surrounding tissues.

ad
The diagnosis is confirmed clinically by microscopic examination of the characteristic negatively
af
birefringent crystals in joint fluid. Treatment with non-steroidal anti-inflammatory drugs
(NSAIDs), steroids or colchicine reduces the symptoms.
-S
40-A 74-year-old man who used to work with asbestos has a lung mass. What is the likely
Al

cause?

a) Adenocarcinoma
in

b) Basal-cell carcinoma
ss

c) Mesothelioma « CORRECT ANSWER


d) Squamous-cell carcinoma
Ya

e) Transitional-cell carcinoma

•Comment on this Question

The main risk factor for mesothelioma is exposure to asbestos dust. In contrast to lung cancer,
mesothelioma is not associated with smoking.

41-Which is the best drug treatment for Parkinson’s disease?

a) Clonidine
b) Chlorpromazine
c) Dopamine
d) L-DOPA « CORRECT ANSWER
e) Vasopressin

•Comment on this Question

82

Yassin Al-Safadi (Safadi92@hotmail.com)


The symptoms of Parkinson’s disease are caused by a lack of dopamine in the substantia nigra.
The administration of L-DOPA (levodopa) temporarily reduces the symptoms. Dopamine cannot
cross the blood–brain barrier, so L-DOPA is used to treat the disease.

42-A 19-year-old man has breakfast consisting of scrambled eggs and toast. He later develops
headache, fever, abdominal pain and diarrhoea that last for 4 days. What is the likely cause?

a) Campylobacter
b) Clostridium difficile
c) E. coli 0157:H7
d) Salmonella « CORRECT ANSWER
e) Shigella

•Comment on this Question

A salmonella infection is a food-borne illness caused by the Salmonella bacterium which is


carried by some animals. It can be transmitted via kitchen surfaces, and may be present in

i
water, soil, animal faeces, raw meats and eggs. Salmonella infections typically affect the

ad
intestines, causing diarrhoea, fever, abdominal cramps and nausea and/or vomiting that usually
resolve without medical treatment. af
43-A patient has immunity to hepatitis B. What is the serological marker that proves this?
-S
a) HBsAg (surface antigen)
Al

b) HBsAb (surface antibody) « CORRECT ANSWER


c) HBcAg (core antigen)
in

d) HBcAb (core antibody)


e) HBeAb (antibody to e antigen)
ss

•Comment on this Question


Ya

• HBsAg (surface antigen) indicates carrier state.

• HBsAb (surface antibody) indicates immunity.

• HBcAg (core antigen) indicates that the person is infectious.

• HBcAb (core antibody), which is positive during a window period, is an indicator of


recent disease.

• HBeAb (antibody to e antigen) indicates low transmissibility.

44-A 30-year-old man has a sudden-onset headache, neck stiffness and nausea, and collapses
with a fluctuating level of consciousness. What is the likely cause?

a) Cerebrovascular accident (stroke)


b) Extradural haematoma

83

Yassin Al-Safadi (Safadi92@hotmail.com)


c) Meningitis
d) Subarachnoid haemorrhage « CORRECT ANSWER
e) Subdural haematoma

•Comment on this Question

A subarachnoid haemorrhage is bleeding that occurs in the subarachnoid space, which is the
area between the arachnoid membrane and the pia mater surrounding the brain. Such bleeding
may occur spontaneously, usually from a ruptured cerebral aneurysm.

Symptoms of subarachnoid haemorrhage include a severe headache with a rapid onset,


vomiting, confusion or a lowered level of consciousness, and sometimes seizures. The diagnosis
is confirmed by a CT scan of the head and a lumbar puncture.

45-Which is the best chemical marker for determining whether a patient has had a myocardial
infarction?

a) Alkaline phosphatase

i
ad
b) Creatinine kinase
c) C-reactive protein
d) D-dimer
af
e) Troponin « CORRECT ANSWER
-S
•Comment on this Question
Al

When heart muscle is damaged, as occurs during a myocardial infarction (MI), troponins leak
out of cells and into the bloodstream. Therefore increased troponin levels indicate myocardial
in

infarction or injury in a person who is experiencing chest pain or pressure.


ss

46-With which genetic abnormality is Klinefelter syndrome associated?


Ya

a) Trisomy 13
b) Trisomy 18
c) XO
d) XXY « CORRECT ANSWER
e) XYY

•Comment on this Question

The presence of an inactivated X chromosome is one of the most common causes of


hypogonadism in males. The main features of the syndrome are testicular atrophy,
gynaecomastia, female hair distribution and increased height.

47-With which genetic abnormality is Turner syndrome associated?

a) Trisomy 13

84

Yassin Al-Safadi (Safadi92@hotmail.com)


b) Trisomy 18
c) XO « CORRECT ANSWER
d) XXY
e) XYY

•Comment on this Question

This condition occurs only in females, and is the most common cause of primary amenorrhoea.
The main features of the syndrome are ovarian dysgenesis, webbing of the neck and coarctation
of the aorta.

48-With which abdominal condition is Rovsing’s sign associated?

a) Appendicitis « CORRECT ANSWER


b) Bowel cancer
c) Crohn’s disease
d) Diverticulitis

i
e) Ulcerative colitis

ad
•Comment on this Question
af
If palpation of the left lower quadrant of a person’s abdomen results in more pain in the right
-S
lower quadrant, the patient is considered to have a positive Rovsing’s sign, and may have
appendicitis.
Al

49-When you tap repeatedly along the course of the facial nerve of a patient, the nerve goes
into spasm. Then, to further your investigation, you inflate the cuff of a sphygmomanometer
in

and the patient has a spasm at the wrist. Which of the following electrolyte disturbances does
ss

the patient have?


Ya

a) Hypercalcaemia
b) Hyperkalaemia
c) Hypernatraemia
d) Hypocalcaemia « CORRECT ANSWER
e) Hypokalaemia

•Comment on this Question

The facial spasms are known as Chvostek’s sign, and the carpal spasms, which occur as a result
of arterial occlusion, are known as Trousseau’s sign. Both are classic signs of hypocalcaemia.

50-A 24-year-old woman has a raised beta-hCG level. She has not had a period for 6 weeks.
What is the most likely diagnosis?

a) Ovarian cancer
b) Vaginal cancer

85

Yassin Al-Safadi (Safadi92@hotmail.com)


c) Normal physiology
d) Pregnancy « CORRECT ANSWER
e) Early menopause

•Comment on this Question

A raised beta-human chorionic gonadotropin (beta-hCG) level is the blood test used to confirm
pregnancy.

51-A 28-year-old man who has recently immigrated to the UK from Somalia presents with a
cough of 3 months duration, fever, haemoptysis and nights sweats. Which one of the
following is the most likely diagnosis?

a) Cholera
b) Community-acquired pneumonia
c) Malaria
d) Pulmonary malignancy

i
e) Tuberculosis « CORRECT ANSWER

ad
•Comment on this Question
af
This patient has presented with the classical signs and symptoms of tuberculosis. This has a high
-S
prevalence in Africa and Asia. The incidence is increasing in the UK, especially among
immigrants.
Al

52-A patient has been diagnosed with trigeminal neuralgia. What is the most common vessel
implicated in trigeminal nerve compression and demyelination in trigeminal neuralgia? Select
in

one option only.


ss

a) Basilar artery
Ya

b) Internal carotid artery


c) Middle cerebral artery
d) Superior cerebellar artery « CORRECT ANSWER
e) Vertebral artery

•Comment on this Question

The most common vessel implicated in trigeminal nerve compression and demyelination in
trigeminal neuralgia is the superior cerebellar artery.

53-Which one of the following asthma medications is a long-acting ß2 adrenoreceptor


agonist?

a) Betamethasone
b) Salbutamol
c) Salmeterol « CORRECT ANSWER

86

Yassin Al-Safadi (Safadi92@hotmail.com)


d) Theophylline
e) Tiotropium bromide

•Comment on this Question

Salmeterol is a long-acting β2 adrenoceptor agonist.

Salbutamol is a short-acting β2 adrenoceptor agonist.

Betamethasone is a glucocorticoid.

Tiotropium bromide is an anticholinergic bronchodilator.

Theophylline is a methylxanthine.

54-Which one of the following is the anticoagulant medication of choice when a patient is
identified pre-operatively as being high risk for venous thromboembolism?

a) Aspirin

i
ad
b) Clopidogrel
c) Compression stocking
d)
e)
Enoxaparin « CORRECT ANSWER
Warfarin
af
-S
•Comment on this Question
Al

Prophylactic enoxaparin, a low molecular weight heparin, is the anticoagulant of choice when a
patient is identified as being high risk for venous thromboembolism, as dictated by NICE
in

guidelines. Venous thromboembolism prophylaxis guidelines are essential reading for all of the
ss

oral and maxillofacial surgery team. Compression stockings can be used in combination.
Ya

55-A patient presents with sudden onset, severe epigastric abdominal pain. There is bilateral
free air under the diaphragm on erect chest X-ray. Which of the following is the most likely
diagnosis?

a) Flatus
b) Gastric bubble
c) Perforated peptic ulcer « CORRECT ANSWER
d) Pneumothorax
e) Small bowel obstruction

•Comment on this Question

Chest X-rays must be taken erect when diagnosing abdominal pain. Free air in the abdomen is
unlikely to be visualised if supine. Bilateral free air under the diaphragm associated with sudden
onset abdominal pain may indicate bowel perforation, in this case from a perforated peptic

87

Yassin Al-Safadi (Safadi92@hotmail.com)


ulcer. The gastric bubble can be differentiated from free air under the diaphragm as this is on
the left side and a fluid level in the stomach is normally visible.

56-Which of the following is the most accurate method of confirming correct nasogastric tube
position?

a) Chest X-ray « CORRECT ANSWER


b) Lack of cough on insertion
c) pH testing of stomach aspirate
d) Syringe aspiration of stomach contents
e) Whoosh test

•Comment on this Question

A chest X-ray is the most accurate method of confirming correct nasogastric (NG) tube position.
This is shown when the tip of the NG tube lies below the level of the diaphragm. Tube position
must be confirmed, normally by more than one method, before commencing feeding. Patients

i
have died when feeding has been commenced and the NG tube lies in the right main bronchus.

ad
Local guidelines should be consulted.
af
57-Which one of the following is a common cause for right upper quadrant abdominal pain?
-S
a) Acute appendicitis
Al

b) Acute cholecystitis « CORRECT ANSWER


c) Gastrointestinal malignancy
d) Pelvic inflammatory disease
in

e) Rupture aortic aneurysm


ss

•Comment on this Question


Ya

Acute cholecystitis is commonly caused by choleliths (gallstones). This initially presents as biliary
colic, an intermittent pain, but develops into a constant severe pain.

58-Melaena is a sign of what?

a) Acute kidney injury


b) Alcoholic liver disease
c) Chronic obstructive pulmonary disease (COPD)
d) Lower GI bleeding
e) Upper GI bleeding « CORRECT ANSWER

•Comment on this Question

Melaena is the name given to black, tar-like stool. It obtains its colour from altered blood. This is
a sign of upper GI haemorrhage.

88

Yassin Al-Safadi (Safadi92@hotmail.com)


[ Oral and
Maxillofacial
i
ad
af
-S

MCQs ]
Al
in
ss

[With Answers]
Ya

89

Yassin Al-Safadi (Safadi92@hotmail.com)


1. Which one of the following drugs in hospital cannot be used in a patient with
anaphylactic shock?
a) Oxygen
b) Hartman’s solution
c) Adrenaline
d) Prednisolone « CORRECT ANSWER
e) Chlorphenamine malate

• Comment on this Question

Anaphylaxis is an emergency and as such in the hospital and intravenous drug such a
hydrocortisone would be used.

2. On histology for a salivary gland fine needle aspiration it is reported to show a


characteristic mix of polygonal and spinal shaped myo-epithelium on a back group of
stroma that is mucoid. The epithelial elements are arranged in duct-like structures.
Which one of the following diagnosis matches the histology results?

i
ad
a) Mucoepidermoid carcinoma
b) Pleomorphic adenoma « CORRECT ANSWER
c) MALToma
d) Adenoid cystic carcinoma
af
-S
e) Warthin tumour
Al
in

• Comment on this Question


ss

This is the histological-defining feature of a pleomorphic adenoma, that if true, biopsy would
Ya

also show to have a pseudocapsule.

3. A 55-year-old lady presents with a unilateral parotid swelling. Of note in her medical
history is rheumatoid arthritis. She also has a complaint of dry eyes and mouth.
a) Mucoepidermoid carcinoma
b) Pleomorphic adenoma
c) MALToma « CORRECT ANSWER
d) Adenoid cystic carcinoma
e) Warthin tumour

• Comment on this Question

90

Yassin Al-Safadi (Safadi92@hotmail.com)


Mucosa-associated lymphoid tissue lymphoma, classically is associated with autoimmune
disease, such as Sjögren’s syndrome.

4. A fine needle aspiration biopsy (FNAB) reported for a parotid tumour shows
perineural spread and Swiss-cheese appearance. Which one of the following is the
most likely diagnosis?
a) Mucoepidermoid carcinoma
b) Pleomorphic adenoma
c) MALToma
d) Adenoid cystic carcinoma « CORRECT ANSWER
e) Warthin tumour

• Comment on this Question

i
ad
This is the classical histological appearance of a tumour which has a predilection for perineural
spread and often presents with pain or cranial nerve dysfunction.
af
5. A 78-year-old lady, who smokes presents with a left parotid gland swelling. On
-S
examination she has a scar in the right parotid region, which she says was from an
operation to remove a gland and the result was a benign tumour.
Al

Which one of the following is the most likely diagnosis?


in

a) Mucoepidermoid carcinoma
b) Pleomorphic adenoma
ss

c) MALToma
Ya

d) Adenoid cystic carcinoma


e) Warthin’s tumour « CORRECT ANSWER

• Comment on this Question

Warthin’s tumour (adenolymphoma) is a benign tumour that is more common in men and
smokers. They are bilateral in 15% of cases, however the masses normally present at different
times.

6. A 21-year-old man presents with a 3-month history of a bluish sessile soft swelling in
the hard palate near the upper left 1st molar, which is pain free. Which one of the
following is the most correct lesion to differential first?

91

Yassin Al-Safadi (Safadi92@hotmail.com)


a) Dentigerous cyst
b) Melanoma
c) Kaposi’s sarcoma
d) Mucous retention cyst
e) Mucoepidermoid carcinoma « CORRECT ANSWER

• Comment on this Question

You must always assume that a pigmented swelling in the palate is a malignant minor salivary
gland tumour until proven otherwise.

7. A patient presents with malignant change in his long-standing ameloblastoma. Which


one of the following organs could it have metastasised to?
a) Liver

i
ad
b) Bone marrow
c) Brain
d) Lung « CORRECT ANSWER
af
e) Spleen
-S
Al
in

• Comment on this Question


ss

Classically these rare malignant tumours have pulmonary metastasis.


Ya

8. Which one of the following is not a sign histologically of dysplasia?


a) Anisocytosis
b) Poikilocytosis
c) Hyperchromatism
d) Loss of cell adhesion
e) Absence of miotic figures « CORRECT ANSWER

• Comment on this Question

The cells are in uncontrolled activity so have increase in mitosis.

92

Yassin Al-Safadi (Safadi92@hotmail.com)


9. A 23-year-old woman is seen in the one stop head and neck clinic with thyroid
swelling, which one of the following is the most likely diagnosis?
a) Lymphoma
b) Papillary cell carcinoma « CORRECT ANSWER
c) Follicular cell carcinoma
d) Secondary deposits
e) Anaplastic malignancy

• Comment on this Question

Young women tend to have papillary cell malignancy.

10. Which one of the following is not an indication to remove or reduce the size of a
thyroid?

i
ad
a) Obstructive symptoms
b) Refractory hypothyroidism « CORRECT ANSWER
c) Aesthetics
af
d) Malignancy
-S
e) Retrosternal goitre
Al
in
ss

• Comment on this Question

Thyroids are removed in the presence of refractory HYPERthyroidism.


Ya

11. Which one of the following does not cause a unilateral lower motor neurone palsy of
the facial nerve?
a) Guillain–Barré syndrome « CORRECT ANSWER
b) Diabetes
c) Ramsey Hunt Syndrome
d) Bell’s palsy
e) Acoustic neuroma

• Comment on this Question

93

Yassin Al-Safadi (Safadi92@hotmail.com)


All the options listed give lower motor neurone lesions and all are unilateral except Guillaine-
Barré.

12. Which one of the following is the correct incidence of cleft lip in the UK?
a) 1 in 200
b) 1 in 500
c) 1 in 900 « CORRECT ANSWER
d) 1 in 1800
e) 1 in 2500

• Comment on this Question

The incidence of a live birth with a cleft lip.

i
ad
The incidence can vary depending on your point of reference, but is usually recorded between
1:700 and 1:1000.
af
13. Which one of the following is the correct term for a fracture of bilateral condylar
-S
necks and mandibular symphysis?
a) Bucket handle fracture
Al

b) Guardsman’s fracture « CORRECT ANSWER


c) Pillion fracture
d) Le Fort fracture
in

e) Guerin’s fracture
ss
Ya

• Comment on this Question

This is a guardsman’s fracture, so named after household guards fainting and falling flat on their
chin, which produces this fracture pattern.

14. Which one of the following anatomical structures is not contained in the cavernous
sinus?
a) Optic nerve « CORRECT ANSWER
b) Abducent nerve
c) Ophthalmic nerve
d) Maxillary nerve
e) Internal carotid

94

Yassin Al-Safadi (Safadi92@hotmail.com)


• Comment on this Question

It does not contain the optic nerve.

15. Which one of the following muscles does the facial nerve supply?
a) Masseter
b) Sternocleidomastoid
c) Stapedius « CORRECT ANSWER
d) Intrinsic muscles of the tongue
e) Lateral rectus

i
ad
• Comment on this Question
af
Stapedius muscle, which helps to dampen loud noises in the inner ear.
-S
16. Which one the following features of basal cell carcinoma (BCC) makes for a good
prognosis?
Al

a) Early detection
b) Rarity
in

c) Ease of removal
d) Sensitivity to chemotherapy
ss

e) Rarity of metastasis « CORRECT ANSWER


Ya

• Comment on this Question

BCC, also called rodent ulcers, are common but only locally aggressive and very rarely
metastasise.

17. Which one of the following diseases is associated with hypodontia?


a) Ectodermal dysplasia « CORRECT ANSWER
b) Congenital syphilis
c) Apert’s syndrome
d) Edward’s syndrome
e) Patau’s syndrome

95

Yassin Al-Safadi (Safadi92@hotmail.com)


• Comment on this Question

Ectodermal dysplasia is a group of syndromes all deriving from abnormalities of the ectodermal
structures. Patients present with missing or peg-shaped or pointed teeth.

18. Which one of the following is not associated with dentinogenesis imperfecta (DI)?
a) Blue sclera
b) Hearing loss
c) Early death
d) Amelogenesis imperfecta « CORRECT ANSWER
e) Osteogenesis imperfecta

i
ad
• Comment on this Question af
The enamel is lost early in DI, but only because the dentine is too weak to support it.
-S
19. Which one of the following immunoglobulins is found in saliva?
Al

a) IgA « CORRECT ANSWER


b) IgD
in

c) IgE
d) IgG
ss

e) IgM
20. Which one of the following is not a muscle of mastication?
Ya

a) Temporalis
b) Lateral pterygoid
c) Masseter
d) Buccinator « CORRECT ANSWER
e) Medial pterygoid

• Comment on this Question

All muscles of mastication are supplied by the trigeminal nerve. The buccinator muscle is very
important in mastication, but it is not supplied by the trigeminal nerve. The buccinator muscle

96

Yassin Al-Safadi (Safadi92@hotmail.com)


pulls back the angle of the mouth and flattens the cheek area, which aids in holding the cheek to
the teeth during mastication. It is also used for facial expression.

21. Which one of the following is not a periosteal elevator?


a) Ward’s
b) Mitchel’s
c) Howarth's
d) Faraboeuf
e) Black’s « CORRECT ANSWER

• Comment on this Question

Black’s is an enamel chisel designed by G.V. Black, who classified cavity design.

i
ad
22. Which one of the following tumours does not metastasis to bone?
a) Prostate
b) Breast af
c) Parathyroid « CORRECT ANSWER
-S
d) Thyroid
e) Bronchus
Al
in
ss

• Comment on this Question


Ya

Parathyroid malignancy is rare and does not go to bone.

23. Which one of the following is a famous maxillofacial surgeon?


a) McGill
b) McIndoe « CORRECT ANSWER
c) Black
d) Lindhe
e) Ivor-Lewis

• Comment on this Question

97

Yassin Al-Safadi (Safadi92@hotmail.com)


Archibald McIndoe (1900–1960) is the cousin of Sir Harold Gillies and treated the patients who
went on to form the ‘Guinea Pig Club’ during World War 2 in East Grinstead.

24. Which one of the following has the highest change of malignant change?
a) Lichen planus
b) Paget’s disease of the bone
c) Syphylitic leucoplakia « CORRECT ANSWER
d) Solar keratosis
e) Hairy leukoplakia

• Comment on this Question

Although almost historical has a near 100% conversion to oral cancer.

i
ad
25. Which one of the following spreading odontogenic infections can present with
minimal swelling and absolute trismus?
a) Sublingual af
b) Parapharyngeal
-S
c) Submandibular
d) Submasseteric « CORRECT ANSWER
Al

e) Buccal
in
ss

• Comment on this Question


Ya

The masseter is an extremely powerful muscle and irritation due to underlying abscess leads to
irritation, spasm and therefore trismus.

Submasseteric abscess is the best answer, but parapharyngeal can also give minimal external
swelling and trismus.

26. Which one of the following is not a long-term complication of spreading odontogenic
infections?
a) Death « CORRECT ANSWER
b) Scarring
c) Halitosis
d) Chronic abscess formation
e) Oro-cutaneous fistula

98

Yassin Al-Safadi (Safadi92@hotmail.com)


• Comment on this Question

Death is a short-term and final complication.

27. Which one of the following is the correct incidence of oral cancer in the UK (2006)?
a) 5 per 100,000
b) 7 per 100,000
c) 9 per 100,000 « CORRECT ANSWER
d) 11 per 100,000
e) 13 per 100,000

i
ad
• Comment on this Question

This represents 5325 new cases per year in the UK. af


-S
28. A patient presents with a 3-cm lesion that is shown to be squamous cell carcinoma
(SCC), clinically he has a unilateral ipsilateral lymph node of 2 cm diameter. Which one
Al

of the following is the correct TNM classification?


a) T1 N2 M0
in

b) T2 N1 Mx « CORRECT ANSWER
c) T3 N1 Mx
ss

d) T3 N2 Mx
e) Tis N1 M1
Ya

• Comment on this Question

Please refer to the World Health Organization TNM classification of head and neck tumours.

29. Which one of the following is the correct percentage of submandibular gland stones
present on radiographs?
a) 20%
b) 30%
c) 50%
d) 80% « CORRECT ANSWER
e) 100%

99

Yassin Al-Safadi (Safadi92@hotmail.com)


• Comment on this Question

Most but not all submandibular stone are radio-opaque; however the reverse is true with
parotid stones.

30. Which one of the following structures is not fractured in a zygomatic complex
fracture?
a) Orbital
b) Zygoma
c) Butress
d) Infra-orbital rim
e) Pterygoid plate « CORRECT ANSWER

i
ad
• Comment on this Question
af
-S
If the pterygoid plates are fractured bilaterally this is a Le Fort-defining injury.
Al

31. How many origins and insertions does each muscle of mastication have?
a) It varies with each muscle
in

b) One insertion and one origin


c) One insertion and two origins
ss

d) Two insertions and one origin


e) Two insertions and two origins « CORRECT ANSWER
Ya

• Comment on this Question

For each muscle of mastication there are two insertions and two origins.

32. Which of the following muscles is supplied by the abducent nerve?


a) Inferior rectus
b) Lateral rectus « CORRECT ANSWER
c) Medial rectus
d) Superior oblique
e) Superior rectus

100

Yassin Al-Safadi (Safadi92@hotmail.com)


• Comment on this Question

The lateral rectus muscle is supplied by the abducent nerve. This can be remembered using the
mnemonic ‘LR6 SO4’ (Lateral Rectus supplied by VI and Superior Oblique supplied by IV).

33. Which one of the following would you request if you suspected a mandibular fracture?
a) OM facial bones
b) Orthopantomogram
c) PA mandible
d) Towne’s view
e) None of the above « CORRECT ANSWER

i
ad
• Comment on this Question af
If there is a suspected fracture, two views should always be requested, namely OPG and PA
-S
mandible in this case.
Al

34. Double vision following trauma is a common sign when which of the following injuries
is sustained?
in

a) Frontal bone fracture


b) High condyle fracture
ss

c) Isolated zygomatic arch fracture


d) Nasal bone fracture
Ya

e) Orbital floor fracture « CORRECT ANSWER

• Comment on this Question

Orbital floor fractures will disrupt the anatomy of the floor, causing entrapment of the inferior
rectus muscle, which results in diplopia.

35. According to 'ATLS' principles, which one of the following is a priority when first
assessing a trauma patient?
a) Airway and cervical spine immobilisation « CORRECT ANSWER
b) Airway control
c) Arrest of haemorrhage

101

Yassin Al-Safadi (Safadi92@hotmail.com)


d) Cervical spine immobilisation
e) None of the above

• Comment on this Question

Airway and cervical spine immobilisation is the first-line management when assessing a trauma
patient.

36. To which of the following injuries does Battle’s sign refer?


a) Base of skull fracture « CORRECT ANSWER
b) Bilateral orbital floor fractures
c) Le Fort II fracture
d) Mandibular fracture
e) Naso-ethmoidal complex fracture

i
ad
af
-S
• Comment on this Question
Al

It is ecchymosis posterior to the ear lying over the mastoid. It generally indicates a base of skull
fracture involving the middle cranial fossa. Although mentioned regularly, it is a relatively late
in

sign.
ss

37. Which fascial space lies inferior to the mylohyoid muscle, and has as its inferior
boundary anteriorly the anterior and posterior bellies of the digastric muscle, and as
Ya

its medial boundary the mylohyoid, hyoglossus and styloglossus muscles?


a) Buccal space
b) Masseteric space
c) Pterygomandibular space
d) Submandibular space « CORRECT ANSWER
e) Submental space

• Comment on this Question

These are the boundaries of the submandibular space, which is also bounded laterally by the
skin, the superficial fascia, the platysma muscle, the superficial layer of deep cervical fascia and
the lateral border of the mandible.

102

Yassin Al-Safadi (Safadi92@hotmail.com)


38.Which anatomical structures are derived from the fourth pharyngeal arch?
a) All of the muscles of mastication
b) The laryngeal muscles and the recurrent laryngeal branch of the vagus nerve
c) The muscles of facial expression, the posterior digastric, stylohyoid and stapedius
muscles, and the facial nerve.
d) The pharyngeal constrictors, levator veli palatini, cricothyroid, larynx and vagus nerve «
CORRECT ANSWER
e) None of the above

• Comment on this Question

All of the muscles of mastication are derived from the first pharyngeal arch. The second arch
(also known as the ‘hyoid’ arch) gives rise to the muscles of facial expression, the posterior

i
digastric, stylohyoid and stapedius muscles, and the facial nerve. The third arch gives rise to the

ad
stylopharyngeus muscle and the glossopharyngeal nerves. The fourth arch gives rise to the
pharyngeal constrictors, levator veli palatini, cricothyroid, larynx and vagus nerve, and the fifth
af
arch gives rise to the laryngeal muscles and the recurrent laryngeal branch of the vagus nerve.
-S
39. Which one of the following would suggest a malignant neoplasm in a salivary gland?
a) Bilateral salivary gland swelling
Al

b) Pyrexia with acute onset of swelling


c) Rapidly growing lesion with involvement of the facial nerve « CORRECT ANSWER
in

d) Slow-growing lesion
ss

e) None of the above


Ya

• Comment on this Question

A rapidly growing lesion could be a sign of acute infection, but the fact that the facial nerve is
also involved would suggest that it is invasive and has malignant properties.

40. Following a biopsy of an intra-oral lesion, the pathology report states the following:
‘Large irregularly shaped nuclei which are darkly stained with abnormal mitoses and
loss of architecture, with local invasion into the lymphatic and blood vessels, with
poor differentiation.’ Which one of the following conditions is this likely to be?
a) Carcinoma in situ
b) Lichen planus
c) Squamous-cell carcinoma « CORRECT ANSWER

103

Yassin Al-Safadi (Safadi92@hotmail.com)


d) Squamous papilloma
e) Traumatic ulceration

• Comment on this Question

This is a classical histological picture of squamous-cell carcinoma, and it is not in situ as it has
invaded. Carcinoma in situ is the most severe stage of epithelial dysplasia, involving the entire
thickness of the epithelium, with the epithelial basement membrane remaining intact.

41. What is the most frequent sequela of an inadequately treated orbital floor fracture?
a) Blindness
b) Diplopia and enopthalmos « CORRECT ANSWER
c) Paraesthesia of the distribution of the infra-orbital nerve
d) Ptosis

i
ad
e) Superior orbital fissure syndrome

af
-S
• Comment on this Question
Al

Diplopia (as the entrapment may not be fully released) and enopthalamos (due to the incorrect
in

volume of the orbit).


ss

42. Which of the following is the most appropriate treatment for a patient in ventricular
fibrillation?
Ya

a) 1 mg IV adrenaline 1:10 000


b) 3 mg atropine IV
c) CPR
d) DC shock « CORRECT ANSWER
e) Intubation via an endotracheal tube

• Comment on this Question

Ventricular fibrillation is a ‘shockable’ cardiac arrest rhythm. Adrenaline, cardiopulmonary


resuscitation (CPR) and intubation are indicated, but if a defibrillator is available, DC shock is
likely to give the best outcome.

104

Yassin Al-Safadi (Safadi92@hotmail.com)


43. Which one of the following is associated with Bell’s palsy?
a) Frey sydrome
b) Parotid tumour
c) Pseudobulbar palsy
d) Ramsay Hunt syndrome « CORRECT ANSWER
e) None of the above

• Comment on this Question

Ramsay Hunt syndrome (cephalic herpes zoster) is a peripheral facial nerve palsy, usually
accompanied by pain and an erythematous vesicular rash on the ear (zoster oticus) or in the
mouth. Ramsay Hunt syndrome can occur with an absence of rash (zoster sine herpete) and is
thought to be the cause of about a fifth of clinically diagnosed cases of Bell palsy.

i
ad
44. Which one of the following is the treatment of choice for a bilaterally displaced
mandibular fracture?
af
a) Conservative management (ie soft diet and analgesics)
b) Extra-oral reduction and fixation
-S
c) Inter-maxillary fixation (IMF) with screws and elastic
d) Inter-maxillary fixation (IMF) with wires and arch-bar reduction
Al

e) Open reduction and internal fixation (ORIF) « CORRECT ANSWER


in
ss
Ya

• Comment on this Question

ORIF is the treatment of choice.

45. What is one of the most common causes of xerostomia in the general population
today?
a) Alcoholism
b) Drug induced « CORRECT ANSWER
c) Malignancy
d) Radiotherapy
e) Sjögren syndrome

105

Yassin Al-Safadi (Safadi92@hotmail.com)


• Comment on this Question

A high proportion of the population today are on a variety of medications, many of which cause
xerostomia as a side-effect. Other causes must also be excluded.

46. Which one of the following is at risk during excision of a submandibular salivary
gland?
a) Mandibular branch of the facial nerve and the hypoglossal nerve
b) Mandibular branch of the facial nerve and the inferior alveolar nerve
c) Mandibular branch of the facial nerve and the lingual nerve
d) Mandibular branch of the facial nerve, the hypoglossal nerve and the mental nerve
e) Mandibular branch of the facial nerve, the lingual nerve and the hypoglossal nerve «
CORRECT ANSWER

i
ad
• Comment on this Question

A neck incision at least 3 cm below the lower border of the mandible and careful surgical
af
technique will avoid damage to the facial nerve, and meticulous dissection with haemostasis is
-S
required throughout the procedure.
Al

47. Which one of the following solutions can be used to treat a keratocyst?
a) Betadine solution
b) Carnegie solution
in

c) Carnoy’s solution « CORRECT ANSWER


ss

d) Chlorhexidine solution
e) None of the above
Ya

• Comment on this Question

Carnoy’s solution is a fixative consisting of 60% ethanol, 30% chloroform and 10% glacial acetic
acid.

48. Which one of the following is a primary branch of the external carotid artery?
a) Ascending pharyngeal branch « CORRECT ANSWER
b) Buccal branch
c) Labial branch
d) Mandibular branch
e) Supratrochlear branch

106

Yassin Al-Safadi (Safadi92@hotmail.com)


• Comment on this Question

The ascending pharyngeal branch is the first branch of the external carotid artery.

49. When placing a nasogastric tube following severe midface trauma, which of the
following should one be cautious about?
a) Causing further haemorrhage
b) Infection
c) Intubation of the trachea
d) Perforation into the anterior cranial fossa « CORRECT ANSWER
e) None of the above

i
ad
• Comment on this Question af
In cases of midface trauma there is a potential risk of a facial dysjunction of the midface from
-S
the cranium. Therefore when passing a nasogastric tube through the nostril there is a small risk
that it will not pass easily through the floor of the nose and inferiorly, but instead will travel
Al

superiorly or posteriorly into the anterior cranial fossa.


in

50. Which one of the following can cause lichen planus as an extra-hepatic sign?
a) Hepatitis B virus (HBV)
ss

b) Hepatitis C virus (HCV) « CORRECT ANSWER


c) Herpes simplex virus
Ya

d) Human immunodeficiency virus (HIV)


e) Varicella zoster virus (VZV)

• Comment on this Question

Lichen planus may be associated with many systemic diseases, few of which are confirmed.
However, infection with HCV can cause extra-hepatic signs, one of which is lichen planus.

51. Which of the following images would you request for a patient with a suspected
mandible fracture, having already excluded a head injury and cleared the cervical
spine?
a) DPT

107

Yassin Al-Safadi (Safadi92@hotmail.com)


b) DPT and PA mandible « CORRECT ANSWER
c) Lateral oblique and PA mandible
d) OM view
e) PA mandible

• Comment on this Question

Two views are always required to ensure that a fracture is visualised.

52. Which one of the following is an indication, according to NICE, for third molar
removal?
a) Anterior crowding in the mandible
b) Distoangular impaction
c) Mesioangular impaction

i
ad
d) Unerupted third molar
e) Unrestorable caries in the third molar « CORRECT ANSWER
af
-S
Al

• Comment on this Question


in

Unrestorable caries is an indication according to the NICE guidelines. None of the other options
is an indication due to the risk of inferior alveolar nerve or lingual nerve damage.
ss

53. A patient who is 6 hours post-operative from the reconstruction of his orbital floor is
Ya

complaining of pain behind his eye with a decrease in vision. What is the most
appropriate first-line management for this case?
a) Administer high dose analgesia
b) Administer steroid eye drops
c) Call the ophthalmologist on call
d) Remove the sutures from the operative site to allow drainage of blood « CORRECT
ANSWER
e) Review the patient on the next morning ward round

• Comment on this Question

108

Yassin Al-Safadi (Safadi92@hotmail.com)


This patient is likely to have a retrobulbar haemorrhage, which is sight threatening. Following
removal of the sutures, the patient will require high-dose steroids and is likely to be taken to
theatre to save the globe.

54. As the maxillofacial junior on call you are called to the Emergency Department to see
a patient with an acute facial swelling that is causing him to drool his saliva. His
tongue is elevated and he has suddenly become short of breath. What is your
immediate management?
a) Admit for intravenous antibiotics and book on the urgent theatre list
b) Admit for intravenous antibiotics and review with the team on the ward round
c) Call non-resident senior
d) Get senior help in the form of anaesthetic and surgical team « CORRECT ANSWER
e) Admit and prescribe oral antibiotics

i
ad
• Comment on this Question
af
This patient has an ensuing airway obstruction. They will potentially need a surgical airway and
incision, and drainage of the swelling. Once you have sought help on site then it is pertinent to
-S
call your senior colleague who is most likely non-resident, or ask someone to do that while you
seek an airway specialist. Do not delay in managing this patient.
Al

55. Which one of the following is likely to alter early in response to systemic infection?
in

a) Blood glucose
b) Blood pressure
ss

c) C-reactive protein « CORRECT ANSWER


d) Haemoglobin
Ya

e) Liver function tests

• Comment on this Question

C-reactive protein is elevated early in response to infection. Blood pressure may alter later.
Other early responses are increase in respiratory rate and heart rate.

56. With regard to dentigerous cysts, which one of the following is correct?
a) Are likely to recur
b) Are invasive and rapidly growing
c) Develop from residual periapical infection

109

Yassin Al-Safadi (Safadi92@hotmail.com)


d) Metastasise to regional lymph nodes
e) Present as asymptomatic swelling/incidental finding « CORRECT ANSWER

• Comment on this Question

Dentigerous cysts are most likely to present as an asymptomatic swelling. They are well
circumscribed on imaging, and unilocular and radiolucent.

57. Which one of the following is the most common maxillofacial fracture sustained?
a) Le Fort I
b) Le Fort II
c) Mandible
d) Nasal bones « CORRECT ANSWER
e) Zygoma

i
ad
af
-S
• Comment on this Question
Al

Nasal bones are the most common bones fractured in the facial skeleton. However they are not
usually managed acutely and mostly treated by ENT colleagues.
in

58. A patient who presents following an alleged assault to the face and complains of
ss

double vision is likely to have sustained which of the following injuries?


a) Fractured condyle
Ya

b) Fractured parasymphysis
c) Nasal bone fracture
d) Palatal fracture
e) Zygomatic complex fracture « CORRECT ANSWER

• Comment on this Question

Fractured zygomatic complex can lead to a depression in the orbital floor causing a ‘trap door’
effect and this can lead to restriction of the inferior rectus muscle.

110

Yassin Al-Safadi (Safadi92@hotmail.com)


[Oral
Medicine
i
ad
af
-S

MCQs]
Al
in
ss

[With Answers]
Ya

111

Yassin Al-Safadi (Safadi92@hotmail.com)


1. 5-year-old school boy presents with a golden crusting sore at the angle of his mouth.
Which one of the following is the diagnosis?
a) Angular cheilitis
b) Scarlet fever
c) Herpes labialis
d) Chickenpox
e) Impetigo « CORRECT ANSWER

• Comment on this Question

Impetigo is a very common local Staphlococcus aureus infection in children with a classic golden
crust.

2. A 13-year-old presents with a sore throat with Koplik’s spots. Which one of the

i
ad
following is the correct diagnosis?
a) Glandular fever
b) Scarlet fever
af
c) Gonorrhoea
-S
d) Chickenpox
e) Measles « CORRECT ANSWER
Al
in
ss

• Comment on this Question


Ya

The prodromal phase is of small white spots with surrounding erythaema. This is before the
maculopapular rash occurs on the trunk.

3. Which one of the following diseases is associated with a high infant mortality rate that
used to kill children and also associated with a strawberry tongue.
a) Glandular fever
b) Scarlet fever « CORRECT ANSWER
c) Gonorrhoea
d) Chickenpox
e) Measles

112

Yassin Al-Safadi (Safadi92@hotmail.com)


• Comment on this Question

Scarlet fever is an infection with β-haemolytic streptococci. It is associated with malaise, fever
and skin rash.

4. A 25-year-old woman presents with multiple superficial ulcers and pharyngitis. Her
swab is reported as Gram –ve intracellular diplococci. Which one of the following is
the most likely diagnosis?
a) Glandular fever
b) Scarlet fever
c) Gonorrhoea « CORRECT ANSWER
d) Chickenpox
e) Measles

i
ad
• Comment on this Question

Gonorrhoea is spread by oro-genital contact and can present with a milder stomatitis.
af
Treatment is with high-dose penicillin.
-S
5. Which one of the following is not associated with Reiter’s syndrome?
Al

a) Oral ulcers
b) Young male
c) Proctitis « CORRECT ANSWER
in

d) Arthritis
ss

e) HLA-B27
Ya

• Comment on this Question

The patient gets urethritis in this disease of unknown aetiology

6. A 25-year-old man present with fever, fatigue and a sore throat. On examination he is
noted to have cervical lymphadenopathy, hepatosplenomegaly and macular–papular
rash. Intra-orally there is a petechial rash on the palate and multiple small ulcers.
Which one of the following is the correct diagnosis?
a) HIV
b) Herpes simplex
c) Coxsackie virus infection
d) Glandular fever « CORRECT ANSWER

113

Yassin Al-Safadi (Safadi92@hotmail.com)


e) Toxoplasmosis

• Comment on this Question

Glandular fever or infectious mononucleosis is a common disease of the young, which maybe
asymptomatic or cause acute illness. It is spread by saliva, so is the ‘kissing’ disease. It is caused
by Epstein–Barr virus.

7. Which one of the following fungal infections has a potential for malignant change?
a) Histoplasmosis
b) Chronic atrophic candidosis
c) Atrophic erythaematous candidosis
d) Chronic mucocutaneous candidosis
e) Chronic hyperplastic candidosis « CORRECT ANSWER

i
ad
af
-S
• Comment on this Question
Al

Presents a dense white, adherent, keratotic patch, with the chance for malignant change. The
lesion is investigated with biopsy and microbiology swabs, if there is no underlying dysplasia it
in

can be treated with systemic antifungals and reduction in oncogenic risk factor, ie stop smoking.
ss

8. Which one of the following does not lead to macroglossi?


a) Acromegaly
Ya

b) Amyloidosis
c) Down’s syndrome
d) Hurler’s syndrome « CORRECT ANSWER
e) Sarcoidosis

• Comment on this Question

Hurler's Syndrome is not typically associated with macroglossia.

9. Which one of the following is associated with an elongation of the filiform papillae?
a) Ankyloglossia
b) Fissured tongue

114

Yassin Al-Safadi (Safadi92@hotmail.com)


c) Hairy tongue « CORRECT ANSWER
d) Erythaema migrans
e) Glossodynia

• Comment on this Question

The hairy tongue, if pigmented the black hairy tongue.

10. Which one of the following is an oral manifestation of Melkerson-Rosenthal


syndrome?
a) Ankyloglossia
b) Fissured tongue « CORRECT ANSWER
c) Hairy tongue
d) Erythaema migrans

i
ad
e) Glossodynia

af
-S
• Comment on this Question
Al

In itself this is not pathological with about 3% of the population have a deeply fissure tongue.
in

Melkerson-Rosenthal syndrome is a deeply fissured tongue, recurrent facial nerve palsy and
swelling.
ss

11. Which one of the following is associated with microglossi?


Ya

a) Ankyloglossia « CORRECT ANSWER


b) Fissured tongue
c) Hairy tongue
d) Erythaema migrans
e) Glossodynia

• Comment on this Question

Tongue tie is the most common developmental abnormality intra-orally.

12. Which one of the following gives atrophic areas of filiform papillae appearing red next
to the normal white areas, which give strickling patterns?

115

Yassin Al-Safadi (Safadi92@hotmail.com)


a) Ankyloglossia
b) Fissured tongue
c) Hairy tongue
d) Erythaema migrans « CORRECT ANSWER
e) Odontoglossum

• Comment on this Question

Also called geographic tongue, which is self-limiting and benign.

13. A 7-year-old deaf child has Hutchinson’s incisors and Moon’s molars. Which one of the
following is his likely diagnosis?
a) Osteogenesis imperfect
b) Syphilis « CORRECT ANSWER

i
ad
c) Romberg syndrome
d) Papillion-Lefevre syndrome
e) Amelogenesis imperfect
af
-S
Al

• Comment on this Question


in

These are signs of congential syphilis along with fontal bossing and saddle nose.
ss

14. Which one of the following statements about malignant melanomas intra-orally is
Ya

false?
a) Rare and accounts for about 1% of intra-oral malignancy
b) Maybe amelanotic
c) Spread haematologically only « CORRECT ANSWER
d) Tend to be maxillary
e) May be preceded by melanosis

• Comment on this Question

They spread both lymphatically and haematologically.

116

Yassin Al-Safadi (Safadi92@hotmail.com)


15. A 65-year-old man has severe throbbing pain in his head. You eliminate the possibe
cause as toothache. It transpires that the pain is made worse when putting on his
glasses and mastication increases the temporal pain. What investigation is diagnostic?
a) Full blood count (FBC)
b) Computerised tomography (CT) brain
c) Orthopantomogram (OPG)
d) Erythrocyte sedimentation rate (ESR) « CORRECT ANSWER
e) X-ray om

• Comment on this Question

ESR for this man who is suffering from temporal arteritis, which produces painful pulseless
temporal arteries. It can affect the lingual arteries leading to tongue necrosis. Retinal artery

i
involvement leads to sudden ocular signs.

ad
16. Which one of the following best describes congenital endocrine disease, which
af
presents with macroglossia, puffy lips and checks with a peaches and cream
complexion?
-S
a) Cushing’s disease
b) Addisons’ disease
Al

c) Hypothyroidism « CORRECT ANSWER


d) Hyperparathyroidism
in

e) Acromegaly
ss
Ya

• Comment on this Question

Congenital hypothyroidism is rare and produces the listed effects. In adults there are no real
intra-oral signs.

17. Which one of the following presents with melanotic lesion on the buccal mucosa,
palmar creases and an old scar from parotid surgery?
a) Cushing’s disease
b) Addison’s disease « CORRECT ANSWER
c) Hypothyroidism
d) Hyperparathyroidism
e) Acromegaly

117

Yassin Al-Safadi (Safadi92@hotmail.com)


• Comment on this Question

Addison’s adrenal insufficiency, which leads to excess adrenocorticotrophic hormone (ACTH)


release to stimulate the adrenal gland’s to release cortisol, but also stimulate melanocytes.

18. A round faced lady with friable gingival mucosa sees her dentist with thrush under
denture. Which one of the following is indicated?
a) Cushing’s syndrome « CORRECT ANSWER
b) Addisons’ disease
c) Hypothyroidism
d) Hyperparathyroidism
e) Acromegaly

i
ad
• Comment on this Question
af
-S
This lady has Cushing’s syndrome, which is excess blood cortisol (steroid) leading to increased
infection risk due to immunosuppression, ‘moon’ face and weaker tissues.
Al

19. A 21-year-old man sees you for a routine check-up. He tells you he has been
in

investigated in the hospital for kidney stones, which has depressed him and still gives
abdominal pain. The orthopantomogram (OPG) you take shows a multi-locular
ss

radiolucency at the angle of the mandible. Which one of the following is a possible
aetiology/diagnosis?
Ya

a) Cushing’s disease
b) Addisons’ disease
c) Hypothyroidism
d) Hyperparathyroidism « CORRECT ANSWER
e) Acromegaly

• Comment on this Question

Hyperparathyroidism with a Brown’s tumour. This endocrine disease gives the classic symptoms
of ‘stones, groans and psychic moans’.

118

Yassin Al-Safadi (Safadi92@hotmail.com)


20. Of the following conditions which one does not have altered facial sensations as a
possible neurological sequelae?
a) Multiple sclerosis (M)
b) Cerebrovascular accident (CVA)
c) Diabetes mellitus (DM)
d) Paget’s disease
e) Bell's palsy « CORRECT ANSWER

• Comment on this Question

Bell's palsy is not facial sensation, ie trigeminal nerve.

21. Which one of the following is not associated with minor recurrent aphthous ulcers
(RAU)?

i
ad
a) Size of ulcer under 1 cm
b) Lack of scarring
c) Age of onset 20–29 years « CORRECT ANSWER
af
d) Usually 1 to 5 lesions
-S
e) Duration 7–14 days
Al
in
ss

• Comment on this Question

Minor RAU start at 10–19 years. Herpetiform RAU starts at 20–29 years.
Ya

22. Which one of the following is not asociated with major recurrent aphthous ulcers
(RAU)?
a) Affects palate, lips, cheeks tongue and pharynx
b) Usually last 14 to 21 days « CORRECT ANSWER
c) Scarring
d) Number of ulcers 1 to 10
e) Age of onset 10–19

• Comment on this Question

119

Yassin Al-Safadi (Safadi92@hotmail.com)


Major RAU last more than 30 days and are very slow to heal, which is unusual in the mouth
where defects normally heal rapidly.

23. Which one of the following is not true of herpetiform recurrent aphthous ulcers
(RAU)?
a) Age of onset 20–29 years
b) Number 10 to 100
c) Size 1 to 2 mm
d) Duration 10 to 30 days
e) Affects a more select region of the oral cavity than minor aphthous ulcer « CORRECT
ANSWER

• Comment on this Question

i
ad
Herpetiform ulcers can occur anywhere in the oral cavity.

24. Which one of the following is not a trigger for recurrent aphthous ulcers (RAU)?
af
a) Menopause « CORRECT ANSWER
-S
b) Streptococcus sanguis
c) HLA-B51
Al

d) Vitamin B12 deficiency


e) Phenytoin
in
ss
Ya

• Comment on this Question

A recent large retrospective literature review shows no association between RAU and hormonal
changes, such as premenstrual tension (PMT), pregnancy or menopause.

25. Hailey-Hailey disease is associated with which one of the following vesicular bullous
diseases?
a) Linear IgA
b) Pemphigus « CORRECT ANSWER
c) Dermatitis herpetiformis
d) Angina bullosa haemorrhagica
e) Erythaema multiforme

120

Yassin Al-Safadi (Safadi92@hotmail.com)


• Comment on this Question

Hailey-Hailey disease is familial benign chronic pemphigus.

26. Which one of the following vesicular bullous diseases would produce a positive
Nikolsky’s sign?
a) Linear IgA
b) Pemphigus « CORRECT ANSWER
c) Dermatitis herpetiformis
d) Angina bullosa haemorrhagica
e) Erythaema multiforme

i
ad
• Comment on this Question

This is the intraepithelial vesiculobullous lesion, which lead to a positive Nikolsky’s sign, in which
af
the epithelium can be slid over the superficial layer.
-S
27. Which one of the following vesiculobullous diseases would produce a basement
membrane zone of immunofluoresence (indirect)?
Al

a) Bullous pemphigoid « CORRECT ANSWER


b) Pemphigus
in

c) Dermatitis herpetiformis
d) Linear IgA
ss

e) Erythaema multiforme
Ya

• Comment on this Question

This produces a band of IgG on INDIRECT immunological testing.

28. Which one of the following lesions is not pre-malignant?


a) Syphylitic leucoplakia
b) Solar keratosis
c) Dyskeratosis congenital
d) Patterson-Brown-Kelly syndrome
e) Systemic lupus erythaematosus (SLE) « CORRECT ANSWER

121

Yassin Al-Safadi (Safadi92@hotmail.com)


• Comment on this Question

There is no risk of malignant change in SLE.

29. Which one of the following oncogenes has been shown to be important in squamous
cell carcinoma (SCC)?
a) p41
b) p53 « CORRECT ANSWER
c) RAS
d) BRAC
e) p-ANCA

i
ad
• Comment on this Question

Mutations of the tumour suppressor such as p53 are associated with SCC.
af
30. Which one of the following is associated with syphilis?
-S
a) Snail tract ulcer
b) Chancre
Al

c) Gumas
d) Argyll Robertson pupils
in

e) All the above « CORRECT ANSWER


ss
Ya

• Comment on this Question

Syphilis is a disease that is caused by Treponema pallidum . It has been a disease studied
throught the ages and is now on the increase again!

31. Which one of the following is the most common cause of recurrent oral ulceration?
a) Aphthae « CORRECT ANSWER
b) Carcinoma/malignancy
c) Infection
d) Radiotherapy
e) Trauma

122

Yassin Al-Safadi (Safadi92@hotmail.com)


• Comment on this Question

Aphthae account for 85% of recurrent oral ulceration, which is most commonly minor. Ulcers
are less than 1 cm in diameter, and typically 2–3 mm. They occur in crops of up to ten ulcers,
and last between 3 days and 2 weeks. The other causes listed are important and should always
be included in your differential diagnosis.

32. Behçet syndrome is a rare condition originally described by a triad of oral ulceration,
genital ulceration and which other symptom?
a) Anaemia
b) Anterior uveitis « CORRECT ANSWER
c) Hypertension
d) Neutropenia
e) Xerostomia

i
ad
• Comment on this Question
af
-S
Anterior uveitis is inflammation of the anterior chamber of the eye. Anaemia and neutropenia
Al

can be associated with oral ulceration, and xerostomia has many causes, but is associated with
Sjögren syndrome.
in

33. What condition is a patient who presents with hyperplastic gingiva, pallor,
lymphadenopathy and gingival haemorrhage most likely to have?
ss

a) Anaemia
Ya

b) Epstein–Barr virus
c) Leukaemia « CORRECT ANSWER
d) Pemphigus
e) Sjögren syndrome

• Comment on this Question

Leukaemia is an uncommon but important cause of oral signs. Ulceration is common, but usually
presents with the above signs. It is an important diagnosis to exclude.

123

Yassin Al-Safadi (Safadi92@hotmail.com)


34. In which condition are antibodies directed against various proteins in the basement
membrane zone, causing damage that results in the full thickness of the epithelium
lifting off the submucosa?
a) Angina bullosa haemorrhagica
b) Dermatitis herpetiformis
c) Linear IgA disease
d) Pemphigoid « CORRECT ANSWER
e) Pemphigus

• Comment on this Question

• The class of antibody is usually IgG or IgM, and C3 is involved. It is an autoimmune


disease that is tissue specific. Remember that it has two distinct forms – mucous membrane

i
pemphigoid (cicatricial) and bullous pemphigoid.

ad
• Dermatitis herpetiformis is characterised by the deposition of IgA in a granular pattern
along the basement membrane zone.
af
-S
• Linear IgA disease is difficult to distinguish clinically from pemphigoid or dermatitis
herpetiformis. It displays IgA as a linear band along the basement membrane zone on
Al

immunofluorescence, and may well be part of a spectrum of diseases, including the above.

• Angina bullosa haemorrhagica has no known aetiology and no identifiable immunology.


in

• Pemphigus is an organ-specific autoimmune disease in which the patient has antibodies


ss

to proteins in the desmosomes which bind the prickle cells together. The antibodies involved
Ya

here are usually IgG or IgM and the C3 component of complement.

35. Which one of the following is associated with gluten-sensitive enteropathy (coeliac
disease)?
a) Chronic hyperplastic candidiasis
b) Dermatitis herpetiformis « CORRECT ANSWER
c) Gonorrhoea
d) Lichen planus
e) Pemphigus

• Comment on this Question

124

Yassin Al-Safadi (Safadi92@hotmail.com)


This is characterised by an itchy vesicular rash which affects the skin, and most patients also
have oral lesions. All patients with this disease have gluten-sensitive enteropathy, which may or
may not be symptomatic. An integral part of treatment is therefore a gluten-free diet, as there is
an allergy to α-gliadin.

36. A patient presents with a palate that has a diffuse grey/white multi-nodular
appearance, with red dots in the centre of the lesions. Which one of the following
diagnoses is most likely?
a) Candida
b) Leukoplakia
c) Lichen planus
d) Lupus erythematosus
e) Nicotinic stomatitis « CORRECT ANSWER

i
ad
• Comment on this Question
af
This is a specific lesion that affects the palate, and which occurs mostly among pipe smokers.
The small red dots represent dilated minor salivary gland duct orifices. In extreme examples the
-S
palate may become fissured or cracked. It is a benign lesion which usually resolves once the
patient has stopped smoking.
Al

37. Which one of the following conditions is defined by the World Health Organization as
in

‘a bright red velvety plaque that cannot be characterised clinically or pathologically as


being due to any other condition’?
ss

a) Erythroplakia « CORRECT ANSWER


b) Leukoderma
Ya

c) Leukoplakia
d) Lichen planus
e) Malignant melanoma

• Comment on this Question

• Leukoplakia is defined as ‘a white patch or plaque that cannot be characterised clinically


or pathologically as any other disease and is not associated with any physical or chemical agent
except the use of tobacco.’

• Leukoderma is a condition characterised by a faint whiteness of the mucosa.

125

Yassin Al-Safadi (Safadi92@hotmail.com)


• Lichen planus has an unknown aetiology.

38. Which one of the following viruses can lie dormant in the trigeminal ganglion or basal
ganglia of the brain before becoming reactivated following a secondary infection?
a) Coxsackie virus
b) Cytomegalovirus (CMV)
c) Herpes simplex virus (HSV) « CORRECT ANSWER
d) Human immunodeficiency virus (HIV)
e) Human papilloma virus (HPV)

• Comment on this Question

Coxsackie viruses are RNA viruses. There are a number of different types of human papilloma
virus, the commonest of which is the wart or verruca virus.

i
ad
39. Which one of the following diagnoses is most likely in a patient who presents with
‘snail-track’ ulcers? af
a) Acute ulcerative gingivitis
-S
b) Behçet syndrome
c) Primary syphilis
Al

d) Recurrent aphthous ulceration


e) Secondary syphilis « CORRECT ANSWER
in
ss
Ya

• Comment on this Question

Primary syphilis presents in the mouth as a primary chancre, whereas mucous patches and
‘snail-track’ ulcers are secondary lesions.

40. Which one of the following oral conditions is a patient with HIV infection most likely
to develop?
a) Acute dental abscess/bacterial infection
b) Human papilloma virus/oral wart-like lesions
c) Kaposi’s sarcoma
d) Oral candidiasis « CORRECT ANSWER
e) Periodontal disease

126

Yassin Al-Safadi (Safadi92@hotmail.com)


• Comment on this Question

All of the other conditions can develop with HIV infection, but oral candidiasis is most commonly
reported.

41. A 47-year-old woman has been referred to you with a lump in her parotid gland. She
does not think that its size has changed much, although it has slowly increased.
Clinical examination reveals that it is firm and painless, but rubbery in texture. She has
no other medical conditions and is otherwise asymptomatic. Which tumour is this
most likely to be?
a) Adenoid cystic carcinoma
b) Mucoepidermoid carcinoma
c) Oncocytoma
d) Pleomorphic adenoma « CORRECT ANSWER
e) Warthin’s tumour

i
ad
af
-S
• Comment on this Question
Al

Pleomorphic adenoma is the commonest salivary gland neoplasm, most often arising in the
parotid gland. The symptoms and clinical findings described above are classical. Warthin’s
tumours are also common, but tend to be soft or cystic in nature, with some fluctuation in size.
in

42. Which condition is characterised by the classic description of a ‘pepper-pot skull’


ss

appearance on skull radiographs, and associated amyloidosis?


Ya

a) Acute leukaemia
b) Hodgkin’s disease
c) Myeloma (plasma cell tumours) « CORRECT ANSWER
d) Paget’s disease
e) Porphyria

• Comment on this Question

These tumours are derived from B lymphyoctes and produce immunoglobulin, which can be
detected in the urine and serum. Gingival bleeding is the commonest sign, but this can occur
following an extraction.

127

Yassin Al-Safadi (Safadi92@hotmail.com)


43. Which investigation is almost diagnostic for giant cell arteritis?
a) CRP (C-reactive protein)
b) ESR (erythrocyte sedimentation rate) « CORRECT ANSWER
c) FBC (full blood count)
d) LFT (liver function test)
e) WCC (white cell count)

• Comment on this Question

This is usually elevated significantly in this condition, although very rarely it may be normal.

44. What is the initial treatment for acute viral sialadenitis?


a) Antivirals
b) CT scanning of head and neck

i
ad
c) Incision and drainage
d) Rehydration and anti-pyretics « CORRECT ANSWER
e) Sialogram
af
-S
Al

• Comment on this Question


in

Initially symptomatic treatment and rehydration are the priority. Antibiotics may be indicated if
ss

there are signs of systemic infection. The diagnosis is made on clinical grounds, although it can
be confirmed via antibody titres between acute and convalescent serum. Vaccination is now
Ya

available for children, but there are several viruses which can cause this condition.

45. What test can be used in the diagnosis of Sjögren syndrome?


a) Allen’s test
b) INR
c) Nikolsky’s sign
d) Oral swab
e) Schirmer’s test « CORRECT ANSWER

• Comment on this Question

This test is not used regularly, but can be utilised to assess the activity of the lacrimal glands.

128

Yassin Al-Safadi (Safadi92@hotmail.com)


46. Which condition is diagnosed with a positive Paul-Bunnell test result?
a) Candida
b) Herpangina
c) HIV
d) Infectious mononucleosis « CORRECT ANSWER
e) Histoplasmosis

• Comment on this Question

Infectious mononucleosis (caused by the Epstein–Barr virus), also known as glandular fever, is
characterised by a sore throat, lymphadenopathy and malaise. The Paul-Bunnell test is
performed on a venous blood sample and is a heterophil antibody test (Monospot®).

47. For the management of which condition are systemic corticosteroids often indicated?

i
ad
a) Geographic tongue
b) Lingual tori
c) Mucocoele
af
d) Pemphigus « CORRECT ANSWER
-S
e) Recurrent aphthous ulceration
Al
in
ss

• Comment on this Question

Systemic corticosteroids are often indicated for the management of pemphigus, and should be
Ya

used with caution and prescribed only by a specialist.

48. In which age group is median rhomboid glossitis most common?


a) 0–15 years
b) 25–35 years
c) 45–60 years « CORRECT ANSWER
d) ≥ 60 years
e) None of the above

• Comment on this Question

129

Yassin Al-Safadi (Safadi92@hotmail.com)


It is most common between the ages of 45 and 60 years. A sufferer of this condition will
commonly present with a sore and beefy red tongue.

49. A 34-year-old woman presents complaining of a sharp ‘electric’-like pain on the left
side of her face over her top jaw and the side of her nose. What is the most likely
diagnosis?
a) Acute sinusitis
b) Burning mouth syndrome
c) Dental pain
d) Temporomandibular joint dysfunction
e) Trigeminal neuralgia « CORRECT ANSWER

• Comment on this Question

i
ad
A sharp or electric shock-like pain is a classic symptom of neuralgia. It is frequently treated with
carbemazepine, however monitoring of liver function is also required.

50.
af
Which one of the following commonly causes macrocytic anaemia?
-S
a) Anaemia of chronic infection
b) Chronic blood loss
Al

c) Sideroblastic anaemia
d) Thalassaemia
in

e) Vitamin B12 deficiency « CORRECT ANSWER


ss
Ya

• Comment on this Question

Vitamin B12 deficiency causes megaloblastic anaemia, which is the most common cause of
macrocytic anaemia.

51. Osteoporosis is associated with which of the following:


a) Addision’s disease
b) Hypergonadism
c) Hypothyroidism
d) Late-onset menopause
e) Smoking « CORRECT ANSWER

130

Yassin Al-Safadi (Safadi92@hotmail.com)


• Comment on this Question

This is a lifestyle risk factor for osteoporosis along with lack of exercise, diet low in calcium or
vitamin D, and excess alcohol intake.

52. Which of the following is a recognised sign of oral squamous cell carcinoma?
a) Angular cheilitis
b) Apthous ulcers
c) Persistent ulcer lasting over 2 weeks « CORRECT ANSWER
d) Smooth erythematous tongue
e) Wickman’s striae

i
ad
• Comment on this Question

This is a classic sign of a malignancy/early invading lesion.


af
53. A patient of yours has a coarse tremor. You recognise this as one of the symptoms of
-S
Parkinsonism. What other symptom would increase your suspicions?
a) Labile mood
Al

b) Lid lag
c) Panic attacks
in

d) Rigidity of the limbs « CORRECT ANSWER


ss

e) Uncontrolled fasiculations of muscles of facial muscles


Ya

• Comment on this Question

Parkinsonism is a common serious brain disorder causing trembling, muscle rigidity, difficult in
walking and problems with balance and coordination. Prevalence increases with age. It affects
the basal ganglia with degeneration of the substantia nigra pigmented cells that release
dopamine.

54. Which one of the following is a recognised complication of phenytoin administration?


a) Deafness
b) Gingival hyperplasia « CORRECT ANSWER
c) Sedation
d) Tremor

131

Yassin Al-Safadi (Safadi92@hotmail.com)


e) Weight gain

• Comment on this Question

Gingival hyperplasia is a common side-effect of phenytoin. Close liaison with the physician
involved in the patient’s care and potentially surgical reduction are important.

55. Which one of the following results would lead to a diagnosis of concerning
hyperglycaemia?
a) Glycosuria
b) Haematuria
c) Ketonuria « CORRECT ANSWER
d) Proteinuria
e) Urobilinogen

i
ad
af
-S
• Comment on this Question
Al

Ketones in the urine reflect a sign of ketoacidosis or starvation. This should be treated urgently
before diabetic ketoacidosis ensues.
in

56. With regard to left-sided heart failure, which of the following is a classic clinical
ss

symptom?
a) Chest pain
Ya

b) Finger clubbing
c) Palpitations
d) Shortness of breath when climbing a flight of stairs
e) Shortness of breath when lying flat/in the night « CORRECT ANSWER

• Comment on this Question

Paraoxysmal nocturnal dyspnea is due to pulmonary oedema, and patients often wake at night
with a feeling of choking or suffocation. An additional pillow or two may help relieve this
symptom.

132

Yassin Al-Safadi (Safadi92@hotmail.com)


57. With regard to a normochromic macrocytic anaemia, which of the following results
would be likely?
a) Decreased mean cell volume (MCV)
b) High haemoglobin
c) Increased MCV « CORRECT ANSWER
d) Normal haemoglobin
e) Normal MCV

• Comment on this Question

The mean cell volume will increase with a normochromic macrocytic anaemia as the cell size
increase, but the degree of haemoglobinisation will remain the same.

58. A patient presenting with ‘snail track’ ulcers in their oral mucosa is likely to have

i
ad
which of the following:
a) Actinomycosis
b) Gonorrhoea
af
c) Human papilloma virus
-S
d) Syphilis « CORRECT ANSWER
e) Tuberculosis
Al
in
ss

• Comment on this Question


Ya

This rarely presents intra-orally nowadays, but this is a classic appearance. A biopsy should be
taken, and the patient referred to the genito-urinary medicine clinic for consultation, or to their
physician.

133

Yassin Al-Safadi (Safadi92@hotmail.com)


[Oral
Pathology
i
ad
af
-S

MCQs]
Al
in
ss

[With Answers]
Ya

134

Yassin Al-Safadi (Safadi92@hotmail.com)


Oral Pathology MCQs
1. A 37-year-old man who has recently returned from a developing world county
presents with pain on the side of his face. Clinical examination reveals multiple
draining sinuses and firm indurated area. The material draining from his sinus appears
to contain yellow granules. This lesion is most likely caused by which one of the
following infections?
a) Tuberculosis
b) Streptococcus pyogenes
c) Actinomyces israelii « CORRECT ANSWER
d) Borrelia vicentii
e) Candida albicans

i
ad
• Comment on this Question

Actinomyces israelii is a normal bacterium within the oral cavity and may produce an indurated
af
appearance with multiple draining fistulas or sinuses. It is a Gram-positive bacterium.
-S
2. With regard to acute inflammation, integrins are involved in which one of the
following phases?
Al

a) Transmigration
b) Rolling
in

c) Adhesion « CORRECT ANSWER


ss

d) Margination
e) Chemotaxis
Ya

• Comment on this Question

Adhesion is the process whereby there is firm adhesion to the endothelium and the integrins are
the molecules involved in this. Selectins are involved in rolling and platelet endothelial cell
adhesion molecules are involved in transmigration.

3. Which one of the following is the most commonly associated condition with
hypodontia?
a) Apert’s syndrome
b) Crouzon’s syndrome
c) Down’s syndrome « CORRECT ANSWER

135

Yassin Al-Safadi (Safadi92@hotmail.com)


d) Osteogenesis imperfecta
e) Paget’s disease

• Comment on this Question

There are many rare syndromes in which hypodontia is a feature, but Down’s is the most
common. Palatal clefts are also common and anodontia is rare.

4. Which one of the following statements is false?


a) Usually with amelogenesis imperfecta all of the teeth are affected and defects involve
the whole enamel
b) Radiographically in dentinogenesis imperfecta the main features are obliterated pulp
chambers and stunted roots
c) Gardner’s syndrome presents only with dental developmental pathology in the form of

i
ad
osteomas « CORRECT ANSWER
d) Congenital syphilis causes fetal infections, small incisors and an anterior open bite. It
may also affect the molar teeth
af
e) Pitting defects may be present in the teeth in a patient with epidermolysis bullosa
-S
Al
in

• Comment on this Question


ss

Gardner’s syndrome is characterised by multiple osteomas (commonly in the jaws), colonic


polyps and skin tumours. The colonic polyps have virtually 100% malignant change with a high
Ya

mortality. Treatment is usually via colectomy and if recognised early by the dental pathology can
be successful.

5. Which one of the following is not a factor in the aetiology of dental caries?
a) Diet
b) Oral flora
c) Age « CORRECT ANSWER
d) Time
e) Susceptible surface

• Comment on this Question

136

Yassin Al-Safadi (Safadi92@hotmail.com)


Age is not an aetiological factor.

6. Which one of the following is likely to be detectable clinically?


a) Dead tracts
b) Regular reactionary dentine
c) Tubular sclerosis (translucent zone) « CORRECT ANSWER
d) Irregular reactionary dentine
e) Live tracts

• Comment on this Question

Tubular sclerosis forms a translucent zone that may be visible radiographically and is detectable
with hand instruments when excavating caries.

i
ad
7. Which one of the following relates to saliva and dental caries?
a) The buffering power of saliva depends mainly on its bicarbonate content and is
increased at high rates of flow« CORRECT ANSWER af
b) The buffering power of saliva is independent of the bicarbonate content
-S
c) The buffering power of saliva depends on mainly its bicarbonate content and is
decreased at low rates of flow
Al

d) The buffering power of saliva depends on its IgA content and is increased at high rates
of flow
in

e) The buffering power of saliva depends on its IgA content and is decreased at high rates
of flow
ss
Ya

• Comment on this Question

The buffering power of saliva depends mainly on its bicarbonate content and is increased at high
rates of flow. IgA is not a component of the buffering power of saliva.

8. After demineralisation which one of the following causes destruction of the dentine
matrix?
a) Liquefaction
b) Proteolysis « CORRECT ANSWER
c) Cleft formation
d) Hydrogen ions permeating the matrix
e) Slow remineralisation and repair

137

Yassin Al-Safadi (Safadi92@hotmail.com)


• Comment on this Question

Remember that hydrogen ions permeate the organic matric on enamel in the initial attack.
Liquefaction foci are involved in the destruction and collection of bacteria in dentinal tubules.

9. Which one of the following is the most common cause of dental pain and loss of teeth,
especially in the younger population:
a) Fracture of a crown/cusp
b) Thermal or chemical irritation
c) Dental caries « CORRECT ANSWER
d) Traumatic pulpal exposure
e) ‘Cracked tooth syndrome’

i
ad
• Comment on this Question
af
-S
Dental caries penetrating the dentine is the most common cause of dental pain and loss of teeth
in the younger population. This is usually obvious unless it is underneath a restoration.
Al

10. he histology of chronic closed pulpitis is most likely to resemble which one of the
in

following?
a) Initial hyperaemia and infiltration by inflammatory cells
ss

b) Predominantly mononuclear cell infiltrate with a vigorous connective tissue reaction «


CORRECT ANSWER
Ya

c) Predominantly mononuclear cell infiltrate without a connective tissue reaction


d) Presence of granulation tissue with a few odontoblasts present
e) Formation of a calcific barrier beneath the lesion

• Comment on this Question

Initial hyperaemia and inflammatory cell infiltrate is found in acute pulpitis, granulation tissue
and a few odontoblasts resembles a pulp polyp following open pulpitis and a calcific barrier
formation is rarely seen, but when it is present is related to acute pulpitis.

11. All of the following are a systemic cause of premature periodontal tissue destruction
except

138

Yassin Al-Safadi (Safadi92@hotmail.com)


a) Hypophosphatasia
b) Down’s syndrome
c) Actinomycosis « CORRECT ANSWER
d) Ehlers–Danlos syndrome
e) Eosinophilic granuloma

• Comment on this Question

Actinomycosis is a chronic suppurative infection caused by Actinomyces israelii usually. It affects


mainly the soft tissues, although now rare.

12. Drug-induced hyperplasia is most commonly associated with which one of the
following?
a) Beta-blockers

i
ad
b) Phenytoin « CORRECT ANSWER
c) Nicorandil
d) Simvastatin
af
e) Benrofluramethazide
-S
Al
in

• Comment on this Question


ss

Phenytoin (EpanutinÔ) is used in the treatment of epilepsy and long-term use can cause gingival
hyperplasia. Other drugs eg calcium-channel blockers (diltiazem and nifedipine) also have similar
Ya

side-effects. Nicorandil (potassium channel activator) is known for its oral ulceration.

13. Which one of the following is the correct definition of a cyst?


a) A pathological fluid-filled cavity lined by epithelium « CORRECT ANSWER
b) A cavity filled with exudate
c) A pathological pus filled cavity lined by epithelium
d) A cavity filled with oedema due to inflammation
e) A cavity with islands and trabeculae of epithelial cells in a connective tissue stroma

• Comment on this Question

139

Yassin Al-Safadi (Safadi92@hotmail.com)


A cavity with islands and trabeculae of epithelial cells in a connective tissue stroma is classic of a
follicular ameloblastoma.

14. Which one of the following conditions is uncommon with acute osteomyelitis?
a) Paget’s disease
b) Osteopetrosis
c) HIV infection « CORRECT ANSWER
d) Acute leukaemia
e) Poorly controlled diabetes mellitus

• Comment on this Question

Acute osteomyelitis of the jaws is uncommon in HIV infection. The effect of immunodeficiencies
is variable. All of the other conditions are important predisposing factors for acute

i
ad
osteomyelitis.

15. Which one of the following is a common cause of swelling of the edentulous jaw in
af
older patients?
-S
a) Lateral radicular cyst
b) Residual cyst « CORRECT ANSWER
Al

c) Odontogenic keratocyst
d) Lateral periodontal cyst
in

e) Eruption cyst
ss
Ya

• Comment on this Question

Residual cysts may persist after extraction of the causative tooth but are also a common cause
of swelling of edentulous jaw in older patients. They may interfere with the fitting of dentures
and sometimes regress spontaneously.

16. Which one of the conditions below is associated with the calcification of the falx
cerebri?
a) Gorlin–Goltz syndrome « CORRECT ANSWER
b) Crouzon’s syndrome
c) Apert’s syndrome
d) Paget’s disease
e) Gardner’s syndrome

140

Yassin Al-Safadi (Safadi92@hotmail.com)


• Comment on this Question

Gorlin–Goltz syndrome has intra-cranial anomalies that include calcification of the falx cerebri
and an abnormally shaped sella turcia

17. Abnormality of the development of the branchial arches or pharyngeal pouches is


associated with which of the following:
a) Thryoid swelling
b) Dermoid cyst « CORRECT ANSWER
c) Sublingual ranula
d) Giant cell lesion
e) Hamartoma

i
ad
• Comment on this Question
af
-S
Dermoid cysts are due to the abnormality of the branchial arches or pharyngeal pouches.
Al

18. Which one of the following condition is hyperplastic and not neoplastic?
a) Giant cell granuloma « CORRECT ANSWER
in

b) Giant cell tumour (osteoclastoma)


c) Ameloblastoma
ss

d) Osteochondroma
e) Cemento-ossifying fibroma
Ya

• Comment on this Question

Giant cell granulomas are hyperplastic rather than neoplastic. However, the giant cell tumour
(osteoclastoma) is an aggressive neoplasm – commonly affecting the limbs but rarely the jaws.

19. Which one of the following conditions is a neoplasm of plasma cells?


a) Acute leukaemia
b) Amyloidosis
c) Multiple myeloma « CORRECT ANSWER
d) Ewing’s sarcoma
e) Langerhans’ cell histiocytosis

141

Yassin Al-Safadi (Safadi92@hotmail.com)


• Comment on this Question

Multiple myeloma produces a monoclonal immunoglobulin causing multiple foci of bone


destruction, bone pain and tenderness. Systemically is also causes anaemia and
thrombocytopenia. Classically a punched out appearance on a skull radiograph is described.

20. Which one of the following conditions stains wth Congo red?
a) Plasmacytoma
b) Candida spp.
c) Amyloidosis « CORRECT ANSWER
d) Burkitt’s lymphoma
e) Warthin’s tumour

i
ad
• Comment on this Question
af
-S
Amyloidosis is deposition of protein within the tissues that is typically perivascular and stains
with Congo red, which also shows a characteristic apple-green birefringence under polarised
Al

light.
in

21. A classic mulberry appearance is seen with which one of the following conditions on
smear?
ss

a) Herpetic vesicle « CORRECT ANSWER


b) Cytomegalovirus
Ya

c) Syphilis
d) Candida spp.
e) Tuberculosis

• Comment on this Question

Herpetic vesicles on a smear have a typical mulberry appearance. Candida shows fungal hyphae
on gram stain. Syphilis is diagnosed typically via dark-ground illumination of a smear from the
chancre. Cytomegalovirus microscopically the ulcers are non-specific, although is commonly
confirmed by immunocytochemistry.

142

Yassin Al-Safadi (Safadi92@hotmail.com)


22. Parakeratosis or hyperkeratosis, saw-tooth-like profile of the rete ridges and
liquefaction degeneration of the basal cell layer are histological appearances of which
one of the following conditions?
a) Behçet’s disease
b) HIV-associated oral ulceration
c) Lichen planus « CORRECT ANSWER
d) Pemphigus
e) Pemphigoid

• Comment on this Question

Lichen planus is a common chronic inflammatory disease of skin and mucous membrane. Other
classical histological features are a pre-dominantly T-cell infiltrate and CD8 lymphocytes.

i
ad
23. A male patient in his mid-30s presents with a firm rubbery neck lump in the cervical
chain and the presence of Reed–Sternberg cells on biopsy. Which one of the following
conditions is he likely to have?
af
a) Non-Hodgkin’s lymphoma
-S
b) Hodgkin’s lymphoma « CORRECT ANSWER
c) Leukaemia
Al

d) Infectious mononucleosis
e) Agranulocytosis
in
ss
Ya

• Comment on this Question

Non-Hodgkin’s is characterised by the absence of Reed–Sternberg cells but the presence of


diffuse or nodular sheets of lymphocytes or lymphoblasts. It is staged in a similar fashion to
Hodgkin’s. Leukaemias are classified according to the specific leucocyte stem cell of origin.
Infectious mononucleosis or Epstein–Barr virus (EBV) causes a mononuclear leucocytosis.
Agranulocytosis is due to a marked decrease in the circulating granulocytes, especially
neutrophils and can be due to a variety of causes. Hodgkin’s lymphoma is based on the
classification of the Reed–Sternberg cell and is more common in men in their third generation.
Firm rubbery cervical chain nodes are an early presentation. Axillary and inguinal are also
common sites.

24. Palatal petechiae are common in which one of the following conditions?
a) Scurvy (vitamin C deficiency)

143

Yassin Al-Safadi (Safadi92@hotmail.com)


b) Hereditary haemorrhagic telangiectasia
c) Thrombocytopenia « CORRECT ANSWER
d) Haemophilia
e) Sickle cell disease

• Comment on this Question

Scurvy is associated with purpura and is due to vascular vessel wall fragility. Hereditary
haemorrhagic telangiectasia is associated with multiple small raised reddish papules on the
dorsum of the tongue as opposed to the palate. Haemophilia is more likely to present with
‘bruises’ or ecchymosis either on the skin or in the oral mucosa.

25. Which one of the following is not found in epithelial dysplasia?


a) Abnormal mitotic figures

i
ad
b) Hyperchromatic nuclei
c) Decreased mitotic activity « CORRECT ANSWER
d) Prominent nucleoli
af
e) Nuclear pleomorphism
-S
Al
in

• Comment on this Question


ss

There is actual increased mitotic activity within epithelial dysplasia.


Ya

26. Which one of the following conditions is likely not to metastasise?


a) Squamous cell carcinoma
b) Basal cell carcinoma « CORRECT ANSWER
c) Adenosquamous carcinoma
d) Nasopharyngeal carcinoma
e) Spindle cell carcinoma

• Comment on this Question

All of the other carcinomas are well known for their ability to metastasizes. Basal cell carcinoma
is locally destructive but non-metastasising.

144

Yassin Al-Safadi (Safadi92@hotmail.com)


27. Which one of the following is not a member of the herpesvirus family?
a) Varicella zoster virus
b) Cytomegalovirus
c) Epstein–Barr virus
d) Herpangina « CORRECT ANSWER
e) Human herpesvirus 6

• Comment on this Question

Herpangina is an infection caused by a mixture of coxsackie A viruses.

28. Which one of the following can be mistaken for a poorly differentiated carcinoma
microscopically?
a) Calcifying odontogenic cyst

i
ad
b) Calcifying epithelial odontogenic tumour « CORRECT ANSWER
c) Adenomatoid odontogenic tumour
d) Squamous odontogenic tumour
af
e) Odontogenic myxoma
-S
Al
in

• Comment on this Question


ss

Calcifying epithelial odontogenic tumour, otherwise known as a ‘Pindborg’ tumour, although


rare, can often be mistaken for a carcinoma as it has bizarre microscopic features such as
Ya

pleomorphic and hyperchromatic nuclei sometimes associated with amyloid like material. All of
the other lesions above are benign.

29. Which one of the following rarely metastasise to the jaw?


a) Breast
b) Bronchus
c) Bladder « CORRECT ANSWER
d) Prostate
e) Thyroid

• Comment on this Question

145

Yassin Al-Safadi (Safadi92@hotmail.com)


Bladder tumours rarely metastasise to the jaws, all of the other primary sites are important ones
to consider.

30. A concerned mother brings her 2-year-old child to your practice. The child has
inflamed gingivae, and ulcerative lesions on the oral mucosa, tongue and gingivae.
What is the likely cause of these symptoms?
a) Candida albicans
b) Cytomegalovirus
c) Epstein–Barr virus
d) Herpes simplex virus 1 « CORRECT ANSWER
e) Herpes simplex virus 2

• Comment on this Question

i
ad
Initial infection with primary herpetic gingivostomatitis most commonly affects children under
10 years of age, with a peak incidence at 2–4 years of age. The incubation period for herpes
af
simplex virus infection is approximately 2–10 days. Patients usually present with an acute
gingivostomatitis characterised by the following:
-S
• pain when eating and drinking
Al

• high fever
in

• extensive ulceration of the tongue, palate and buccal mucosa.


ss

31. A 40-year-old woman presents with a bluish transparent cystic lesion, 5 mm in


diameter, on her lower lip. It has increased and decreased in size repeatedly over the
Ya

last 3 months, and is not pedunculated. What is the likely cause?


a) Fibro-epithelial polyp
b) Granulo cell myeloblasta
c) Mucocoele « CORRECT ANSWER
d) Pyogenic granuloma
e) Squamous-cell carcinoma

• Comment on this Question

‘Mucocoele’ is a clinical term that refers to two related phenomena, namely mucus
extravasation phenomenon and mucus retention cyst. The former is a swelling of connective

146

Yassin Al-Safadi (Safadi92@hotmail.com)


tissue, consisting of mucin that has accumulated as a result of rupture of a salivary gland duct,
usually caused by local trauma. Mucus retention cysts are caused by a collection of mucin
resulting from an obstructed salivary duct. The mucocoele is bluish in colour and translucent,
and is more commonly found in children and young adults.

32. At which of the following sites does squamous-cell carcinoma most commonly
present?
a) Floor of mouth
b) Hard palate
c) Lip
d) Retromolar pad
e) Tongue « CORRECT ANSWER

i
ad
• Comment on this Question
af
The tongue is the most common site for oral squamous-cell carcinoma. It presents on the
tongue in 30% of cases. The next most common sites are the lower lip (17%) and the floor of the
-S
mouth (17%).
Al

33. Which of the following sites of presentation of oral squamous-cell carcinoma has the
best prognosis?
in

a) Floor of mouth
b) Hard palate
ss

c) Lip « CORRECT ANSWER


d) Retromolar pad
Ya

e) Tongue

• Comment on this Question

Squamous-cell carcinoma of the lip has the best prognosis. One of the main reasons is that, as it
is visible on the lip, patients notice the lesion earlier and therefore present to hospital at an
early stage.

34. Which one of the following viruses is implicated in the development of squamous-cell
carcinoma?
a) Cytomegalovirus

147

Yassin Al-Safadi (Safadi92@hotmail.com)


b) Epstein–Barr virus
c) Herpes simplex virus 1
d) Herpes simplex virus 2
e) Human papilloma virus « CORRECT ANSWER

• Comment on this Question

Human papilloma virus is implicated in the production of squamous-cell carcinoma, and has
been implicated in the rise in squamous-cell carcinomas especially in the young. It is also a risk
factor for cervical cancer and rectal cancer.

35. The pathology report for a lesion that has been found in a right superficial
parotidectomy states the following: ‘Mixed tumour (epithelial and mesenchymal).
Circumscribed, partially encapsulated, firm, yellow-white tumour composed of cords

i
ad
and nests of epithelial cells in a hyaline-like mucoid stroma.’ What is the diagnosis?
a) Adenoid cystic carcinoma
b) Basal-cell carcinoma
af
c) Monomorphic adenoma
-S
d) Pleomorphic adenoma « CORRECT ANSWER
e) Squamous-cell carcinoma
Al
in
ss

• Comment on this Question


Ya

Pleomorphic adenoma is a benign neoplastic tumour of the salivary glands. It is the most
common type of salivary gland tumour, and the most common tumour of the parotid gland.

36. In which oral disease are Wickham’s striae the cutaneous component?
a) Bullous pemphigoid
b) Bullous pemphigus
c) Lichen planus « CORRECT ANSWER
d) Lichenoid reaction
e) Lupus erythematosus

• Comment on this Question

148

Yassin Al-Safadi (Safadi92@hotmail.com)


Classical lichen planus is characterised by the presence of firm, shiny, flat-topped papules that
range from pinpoint size to more than 1 cm in diameter. They may be close together or
widespread, or grouped in lines (linear lichen planus) or rings (annular lichen planus). Although
sometimes there are no symptoms, the condition is often very itchy.

37. Which organism causes syphilis?


a) Clostridium perfringens
b) Mycobacterium leprae
c) Staphylococcus aureus
d) Streptococcus pyogenes
e) Treponema pallidum « CORRECT ANSWER

• Comment on this Question

i
ad
Treponema pallidum is a species of spirochaete bacterium. The organism is too thin to be seen
on a Gram-stained smear.

38. Which organism causes gas gangrene?


af
-S
a) Clostridium perfringens « CORRECT ANSWER
b) Mycobacterium leprae
Al

c) Staphylococcus aureus
d) Streptococcus pyogenes
in

e) Treponema pallidum
ss
Ya

• Comment on this Question

Gas gangrene is a life-threatening bacterial infection that results in the production of gas
bubbles in gangrenous tissues. It is usually caused by infection with the bacterium Clostridium
perfringens.

39. An elderly woman presents with a headache and jaw pain during mastication. Her
blood tests show a raised erythrocyte sedimentation rate (ESR). Which of the
following is the most likely cause?
a) Dental abscess
b) Migraine headache
c) Temporal arteritis « CORRECT ANSWER
d) Temporomandibular joint dysfunction

149

Yassin Al-Safadi (Safadi92@hotmail.com)


e) Trigeminal neuralgia

• Comment on this Question

Temporal arteritis affects patients over 50 years of age. Although it typically affects the
superficial temporal arteries, it can involve medium-sized and large vessels, including the aorta
and the carotid, subclavian, vertebral and iliac arteries.

Headache is the most common symptom, and is present in over two-thirds of patients with
temporal arteritis. It tends to be new or different in character to previous headaches, and is
typically sudden in onset, localising to the temporal region. Therefore the possibility of temporal
arteritis should be considered when patients over 50 years of age present with any new
headache.

i
ad
The involvement of the superficial temporal artery can lead to severe scalp tenderness when,
for example, resting the head on a pillow, combing the hair, or wearing a hat or spectacles.
Patients may also present with visible areas of scalp necrosis. Similarly, jaw claudication when
af
speaking or chewing is observed when the maxillary artery is involved (around 50% of patients
-S
with temporal arteritis).
Al

Visual loss may also be a presenting symptom, and can be sudden and painless.

40. Which of the following genetic combinations is responsible for Turner syndrome?
in

a) Trisomy 18
b) Trisomy 21
ss

c) XO « CORRECT ANSWER
Ya

d) XX
e) XXY

• Comment on this Question

Turner syndrome affects about 1 in every 2500 girls.

Most girls are born with two X chromosomes, but those with Turner syndrome have only one of
these chromosomes. The effects of the condition vary widely.

Girls with Turner syndrome are usually of short stature. Those who are not treated for this reach
an average height of about 1.4 metres (4 feet 7 inches). Treatment with growth hormones helps
girls with this condition to reach a height that is closer to normal.

150

Yassin Al-Safadi (Safadi92@hotmail.com)


Turner syndrome also prevents the ovaries from developing normally, which affects girls’ sexual
development and their ability to have children.

Additional physical features that are commonly seen in girls with Turner syndrome include the
following:

• a ‘webbed’ neck (extra folds of skin extending from the tops of the shoulders to the
sides of the neck)

• a low hairline at the back of the neck

• drooping of the eyelids

• differently shaped ears that are set lower than normal on the sides of the head.

41. A patient has a factor IX deficiency. What is the common name of this condition?
a) Christmas disease « CORRECT ANSWER
b) Disseminated intravascular coagulopathy

i
ad
c) Easter disease
d) Haemophilia
e) Von Willebrand’s disease af
-S
Al

• Comment on this Question


in

Haemophilia B is a blood clotting disorder caused by a mutation of the factor IX gene, which
ss

leads to a deficiency of factor IX. It is rarer than haemophilia A (the more common form of
haemophilia). Haemophilia B is often called Christmas disease, after Stephen Christmas, the first
Ya

patient to be described with this disease. In addition, the first report of its identification was
published in the Christmas edition of the British Medical Journal.

42. With which tumour are Reed–Sternberg cells associated?


a) Adenocarcinoma
b) Hodgkin’s lymphoma « CORRECT ANSWER
c) Melanoma
d) Non-Hodgkin’s lymphoma
e) Squamous-cell carcinoma

• Comment on this Question

151

Yassin Al-Safadi (Safadi92@hotmail.com)


Reed–Sternberg cells are a type of giant cell that is observed on light microscopy of biopsies
from individuals with Hodgkin’s lymphoma.

43. Which of the following substances is the major causative agent for mesothelioma?
a) Aflatoxins
b) Arsenic
c) Asbestos « CORRECT ANSWER
d) Naphthalene
e) Nitrosamines

• Comment on this Question

Most mesotheliomas are caused by exposure to asbestos. They are usually diagnosed 30 years
or more after the first exposure to asbestos. Mesothelioma occurs more often in men than in

i
ad
women, and the risk of developing the disease increases with age.

44. Which one of the following statements about salivary calculi is true?
af
a) They cause marked xerostomia
-S
b) They are always seen radiographically
c) They are never asymptomatic
Al

d) They most commonly occur in the parotid gland


e) They most commonly occur in the submandibular gland « CORRECT ANSWER
in
ss
Ya

• Comment on this Question

Calculi are most commonly found in the submandibular gland, where Wharton’s duct can
become obstructed by stones. They are frequently associated with chronic infection of the
glands, dehydration and/or increased calcium levels locally, but in many cases they can arise
idiopathically. If pain occurs, it usually originates from the floor of the mouth, although in many
cases the stones cause only intermittent swelling. Since chewing promotes the release of saliva,
symptoms tend to increase when the patient is eating. A palpable lump or visible swelling in the
area of the gland is often observed.

Complications include persistent obstruction of the duct, leading to bacterial invasion,


overgrowth and infection.

45. Which of the following conditions is associated with anti-basement membrane auto-
antibodies?

152

Yassin Al-Safadi (Safadi92@hotmail.com)


a) Erythema multiforme
b) Herpes zoster
c) Pemphigoid « CORRECT ANSWER
d) Pemphigus
e) Stevens–Johnson syndrome

• Comment on this Question

Pemphigoid is a group of uncommon autoimmune diseases that cause blistering of the skin. It is
similar to pemphigus, but in contrast to the latter it does not feature acantholysis.

Pemphigoid is more common than pemphigus. It is slightly more common in women than in
men, and in people over 60 years of age than in younger individuals.

i
ad
46. Which of the following is not a feature of dysplasia?
a) Altered cytoplasmic/nuclear ratio
b) Atypical mitosis af
c) Hyperkeratinisation « CORRECT ANSWER
-S
d) Loss of cellular polarity
e) Loss of or decrease in intercellular adherence
Al
in
ss

• Comment on this Question


Ya

Dysplasia is a term used in pathology to refer to an abnormality of development. It is often


indicative of an early neoplastic process, and typically refers to a cellular abnormality that is
restricted to the tissue of origin.

47. Which of the following is a non-odontogenic cyst?


a) Dentigerous cyst
b) Eruption cyst
c) Nasopalatine cyst « CORRECT ANSWER
d) Odontogenic keratocyst
e) Radicular cyst

• Comment on this Question

153

Yassin Al-Safadi (Safadi92@hotmail.com)


This type of cyst, also known as a nasopalatine duct cyst, occurs in the median of the palate. It is
usually anterior to the first molars. It is the commonest type of oral non-odontogenic cyst, and is
usually asymptomatic, but sometimes produces an elevation in the anterior portion of the
palate. Radiographically, it appears as a heart-shaped radiolucency.

48. Which of the following is an inflammatory odontogenic cyst?


a) Dentigerous cyst
b) Eruption cyst
c) Gingival cyst
d) Odontogenic keratocyst
e) Radicular cyst « CORRECT ANSWER

• Comment on this Question

i
ad
This type of cyst, also known as a periapical cyst, is the most common type of odontogenic cyst.
It is caused by inflammation or death of the pulp as a result of trauma or dental caries. These
af
cysts are usually asymptomatic, but pain can be caused by secondary inflammation.
Radiographically, a radicular cyst appears as radiolucency around the apex of the root of a
-S
tooth.
Al

49. Which of the following cysts is derived from the epithelial cell rests of Malassez?
a) Dentigerous cyst
in

b) Eruption cyst
c) Gingival cyst
ss

d) Odontogenic keratocyst
e) Radicular cyst « CORRECT ANSWER
Ya

• Comment on this Question

This type of cyst, also known as a periapical cyst, is the most common type of odontogenic cyst.
The lining of the cyst is derived from the epithelial cell rests of Malassez.

50. A lady attends an oral medicine clinic after an urgent referral from her GDP. She
presented with an asymptomatic, bilateral white lesion with poorly defined margins.
The lesion was soft on palpation but irregularly thick. Incisional biopsy revealed
hyperplastic epithelium and rete ridges with a smooth border. What is this lesion most
likely to be?

154

Yassin Al-Safadi (Safadi92@hotmail.com)


a) Frictional keratosis
b) Leukoplakia
c) Lichen planus
d) White hairy leukoplakia
e) White sponge naevus « CORRECT ANSWER

• Comment on this Question

The diagnosis is made as a result of the histological report. Lichen planus could display several
similar characteristics, being bilateral in appearance, and when in plaque-like form shows similar
clinical features; however, histologically lichen planus displays saw tooth rete ridges with
hyperkeratosis or parakeratosis. White sponge naevus shows rete ridges with smooth borders
and hyperplastic epithelium. Frictional keratosis has a bilateral presentation adjacent to the

i
occlusal plane, whereas the other two lesions are generally unilateral.

ad
51. Regarding pemphigus vulgaris, which one of the following is correct?
a) Affects men more than women
af
b) Has a high titre of antibodies to desmoglein 3 « CORRECT ANSWER
-S
c) Presents with subepithelial bullae
d) Should be confirmed by Nikolsky’s sign
Al

e) Skin involvement is absent


in
ss
Ya

• Comment on this Question

Pemphigus predominantly affects women more than men. It consists of intraepithelial bullae,
and while Nikolsky’s sign would prove the diagnosis it should be avoided as it will produce more
bullae. Pemphigus also generally spreads widely on the skin.

52. Which one of the following diseases has a diagnostic autoantibody associated with
secondary Sjögren syndrome?
a) Diabetes mellitus
b) Osteoarthritis
c) Psoriasis
d) Rheumatoid arthritis « CORRECT ANSWER
e) Sarcoidosis

155

Yassin Al-Safadi (Safadi92@hotmail.com)


• Comment on this Question

Rh factor is present in up to 90% of patients with secondary Sjögren syndrome, while it is


present in only 50% of those with primary Sjögren syndrome.

53. Which one of the following syndromes has been known to predispose to oral
squamous cell carcinoma?
a) Crouzon syndrome
b) Gorlin–Goltz syndrome
c) Paterson–Kelly syndrome « CORRECT ANSWER
d) Ramsay Hunt syndrome
e) Sjögren syndrome

i
ad
• Comment on this Question
af
Paterson–Kelly syndrome has been linked to oral and oesophageal carcinoma, but is reducing as
-S
a risk factor. Ramsay Hunt syndrome is related to varicella zoster infection around the ear.
Gorlin–Goltz is related to multiple basal cell carcinomas and odontogenic keratocysts.
Al

54. A patient attends the oral medicine clinic following a referral from their GDP. She has
been experiencing recurrent aphthae for several months. The GDP has prescribed
in

benzydamine (Difflam®) mouthwash, which does manage to control her symptoms,


but the aphthae are still recurring. What is your first stage of treatment?
ss

a) Blood tests and diet analysis « CORRECT ANSWER


Ya

b) B12 supplements
c) No treatment
d) Systemic corticosteroids
e) Topical corticosteroids

• Comment on this Question

Minor recurrent aphthous stomatitis (MiRAS) has been linked to benzoates in the diet, and iron
or B12 deficiency, and these must be ruled out prior to adopting any further treatment.

55. Orthopantomogram (OPG) radiograph shows a unilocular swelling associated with the
enamocemental junction. What is this lesion most likely to be?

156

Yassin Al-Safadi (Safadi92@hotmail.com)


a) Dentigerous cyst « CORRECT ANSWER
b) Eruption cyst
c) Keratocystic odontogenic tumour
d) Radicular cyst
e) Unicysytic ameloblastoma

• Comment on this Question

Dentigerous cysts are developmental and are associated with the crowns of unerupted (or
partially erupted) teeth, most commonly third molars. The cyst cavity is lined with epithelial cells
derived from the reduced enamel epithelium.

56. A swelling related to a dentigerous cyst is biopsied. The histological report states
islands of epithelium with a peripheral layer of cells that have nuclei at the opposite

i
ad
to the basement membrane (reversed polarity). What is the lesion most likely to be?
a) Dentigerous cyst
b) Eruption cyst
af
c) Keratocystic odontogenic tumour
-S
d) Radicular cyst
e) Unicysytic ameloblastoma « CORRECT ANSWER
Al
in
ss

• Comment on this Question


Ya

This pathology suggests an ameloblastoma. Unicystic ameloblastoma can envelop the crowns of
third molars, and radiographically could be mistaken for a dentigerous cyst.

57. A young lady attends your surgery embarrassed as she has developed marked
halitosis. Which one of these organisms could you attribute to the sulphur production
underlying this problem?
a) Aggregatibacter actinomycetemcomitans
b) Candida albicans
c) Centipedia periodontii « CORRECT ANSWER
d) Streptococcus mutans
e) Treponema denticola

• Comment on this Question

157

Yassin Al-Safadi (Safadi92@hotmail.com)


Centipedia periodontii is an organism responsible for the release of sulphur, which is the basis of
halitosis.

i
ad
af
-S
Al
in
ss
Ya

158

Yassin Al-Safadi (Safadi92@hotmail.com)


[Oral Surgery
i
MCQs] ad
af
-S
Al

[With Answers]
in
ss
Ya

159

Yassin Al-Safadi (Safadi92@hotmail.com)


1. Who has described lines for deciding radiographically the difficulty in wisdom tooth
extraction?
a) Winter « CORRECT ANSWER
b) Summer
c) Spring
d) Autumn
e) Falls

• Comment on this Question

Winter’s lines look at the internal oblique ridge, occlusal plane and a line from the point of
application to the occlusal line to express difficulty of extraction.

2. Which one of the following scalpel blades is commonly used in intra-oral surgery?

i
ad
a) No. 10
b) No. 11
c) No. 12
af
d) No. 15 « CORRECT ANSWER
-S
e) No. 22
Al
in
ss

• Comment on this Question

No. 15
Ya

3. Which teeth are Warwick-James elevators designed to remove?


a) Upper impacted canines
b) Lower canines
c) Lower first molars’ roots
d) Lower eights
e) Upper eights « CORRECT ANSWER

• Comment on this Question

Upper eights

160

Yassin Al-Safadi (Safadi92@hotmail.com)


This is a root elevator, and should be used to remove upper wisdom teeth.

4. Upper wisdom teeth are especially associated with an infection leading to trismus.
Extract with a rotation and pull:
a) Upper 1, 2, 3 « CORRECT ANSWER
b) Upper 4, 5
c) Upper 6, 7
d) Lower 4, 5
e) Lower 6, 7

• Comment on this Question

Upper 1, 2, 3

i
ad
This question examines whether you have extracted teeth before. In oral surgery, this is the
motion required to extract these teeth.

5.
af
Extract with a bucco-palatal movement until they give, then pulled down and buccally:
-S
a) Upper 1, 2, 3
b) Upper 4, 5 « CORRECT ANSWER
Al

c) Upper 6, 7
d) Lower 4, 5
e) Lower 6, 7
in
ss
Ya

• Comment on this Question

Upper 4, 5

6. Moved buccally while pushing coronally, but frequently need a varied rocking
movement to extract:
a) Upper 1, 2, 3
b) Upper 4, 5
c) Upper 6, 7 « CORRECT ANSWER
d) Lower 4, 5
e) Lower 6, 7

161

Yassin Al-Safadi (Safadi92@hotmail.com)


• Comment on this Question

Upper 6, 7

7. These teeth are rotated and lifted to extract:


a) Upper 1, 2, 3
b) Upper 4, 5
c) Upper 6, 7
d) Lower 4, 5 « CORRECT ANSWER
e) Lower 6, 7
8. Extracted by controlled buccal movements:
a) Upper 1, 2, 3
b) Upper 4, 5
c) Upper 6, 7
d) Lower 4, 5
e) Lower 6, 7 « CORRECT ANSWER
9. What percentage of the population has ectopic maxillary canines?

i
ad
a) 0.5%
b) 2% « CORRECT ANSWER
c) 5%
af
d) 10%
-S
e) 20%
Al
in
ss

• Comment on this Question

2% in the Caucasian population.


Ya

10. What percentage of the population has a missing wisdom tooth?


a) 0.5%
b) 2%
c) 5%
d) 10%
e) 20% « CORRECT ANSWER

• Comment on this Question

20% in the Caucasian population.

162

Yassin Al-Safadi (Safadi92@hotmail.com)


11. Which one of the following odontogenic pathologies is associated with an unerupted
tooth?
a) Ameloblastoma
b) Odontomes
c) Solitary bone cyst
d) Dentigerous cyst « CORRECT ANSWER
e) Keratocyst

• Comment on this Question

Dentigerous cysts form around the crown of impacted permanent teeth and arise from the
reduced enamel epithelium.

12. Which odontogenic pathology can be a sac of multiple ‘teeth’?

i
ad
a) Ameloblastoma
b) Odontomes « CORRECT ANSWER
c) Solitary bone cyst
af
d) Dentigerous cyst
-S
e) Keratocyst
Al
in
ss

• Comment on this Question

Odontomes are either compound (ie looking like teeth) or complex (ie a mass of malformed
Ya

dental hard tissues).

13. Which odontogenic pathology is associated with Afro-Caribbean men?


a) Ameloblastoma « CORRECT ANSWER
b) Odontomes
c) Solitary bone cyst
d) Dentigerous cyst
e) Keratocyst

• Comment on this Question

A benign odontogenic tumour.

163

Yassin Al-Safadi (Safadi92@hotmail.com)


14. Which odontogenic pathology is associated with daughter cysts?
a) Ameloblastoma
b) Odontomes
c) Solitary bone cyst
d) Dentigerous cyst
e) Keratocyst « CORRECT ANSWER

• Comment on this Question

Odontogenic keratocyst.

15. Which odontogenic pathology would contain clear straw-coloured fluid?


a) Ameloblastoma
b) Odontomes

i
ad
c) Solitary bone cyst « CORRECT ANSWER
d) Dentigerous cyst
e) Keratocyst
af
-S
Al

• Comment on this Question


in

Solitary bone cyst, which is often an incidental finding devoid of lining and containing straw-
ss

coloured fluid.
Ya

16. What does a black gas cylinder with a white valve contain?
a) Air
b) Oxygen « CORRECT ANSWER
c) Nitrogen
d) Nitrous oxide
e) Entonox

• Comment on this Question

Oxygen

17. What is Vicryl made from?

164

Yassin Al-Safadi (Safadi92@hotmail.com)


a) Polyglycolic acid
b) Polydioxanone
c) Polyglactin « CORRECT ANSWER
d) Polyglecaprone
e) Polyglyconate

• Comment on this Question

Polyglactin

18. Which one of the following is not a soft laser?


a) Carbon dioxide laser
b) Argon laser
c) Helium-neon laser « CORRECT ANSWER

i
ad
d) KTP laser
e) Nd-Yag laser
af
-S
Al

• Comment on this Question


in

Hard lasers produce a cutting affect, as in the James Bond film Goldfinger. Soft lasers work by
stabilising cell membranes by a non-thermal photochemical process, increasing cellular
ss

metabolism. Helium-neon lasers are used for laser pointers!


Ya

19. Which one of these is not an NHS justification for implant placement?
a) Prosthetics tolerance issues
b) Trauma
c) Hypodontia
d) Teaching « CORRECT ANSWER
e) Oncology

• Comment on this Question

Teaching need is not a justification for the NHS to pay for implants.

20. Which one of the following syndromes is associated with multiple tori?

165

Yassin Al-Safadi (Safadi92@hotmail.com)


a) Eagle’s syndrome
b) Frey’s syndrome
c) Gardner’s syndrome « CORRECT ANSWER
d) Golin–Goltz syndrome
e) Crouzon’s syndrome

• Comment on this Question

Gardner’s syndrome is associated with multiple osteomas, epithelial cysts and intestinal polyps.

21. A 78-year-old woman presents for the extraction of her lower left 6. She is deaf with a
‘cotton wool’ bone appearance on OPG. Which one of the following is the diagnosis?
a) Paget’s disease « CORRECT ANSWER
b) Giant cell granuloma

i
ad
c) Cherubism
d) Fibrous dysplasia
e) Osteogenesis imperfecta
af
-S
Al

• Comment on this Question


in

Paget’s affects 3% of over 55-year-olds and is associated with the above feature and cranial
ss

nerve defects.
Ya

22. Which one of these biochemical changes is associated with Paget’s disease?
a) Anaemia
b) Vitamin increase
c) Raised alkaline phosphate « CORRECT ANSWER
d) Low serum calcium
e) High serum phosphate

• Comment on this Question

Paget’s disease is a disease of increased turnover and chaotic bone remodelling, which leads to
enlarged but weak bone morphology. It has normal calcium and phosphate, but increased
alkaline phosphate.

166

Yassin Al-Safadi (Safadi92@hotmail.com)


23. Which one of the following is not a complications of Pagets’ disease?
a) Osteosarcoma
b) Low output congestive cardiac failure « CORRECT ANSWER
c) Cranial nerve defect
d) Pathological fractures
e) Bone pain

• Comment on this Question

Paget’s produce HIGH output heart failure, as the heart is asked to perform an abnormally high
function and not that it cannot cope with normal base-like function.

24. A bone biopsy shows irregular Chinese characters. Which one of the following is the
diagnosis?

i
ad
a) Paget’s disease
b) Giant cell granuloma
c) Cherubism
af
d) Fibrous dysplasia « CORRECT ANSWER
-S
e) Osteogenesis imperfect
Al
in
ss

• Comment on this Question

Areas of bone are replaced by fibrous tissue with a ‘ground-glass’ appearance on radiographs
Ya

and a histologically characteristic appearance due to fibrous replacement of bone with osseous
trabeculae.

25. Which retrograde filling is recommended when performing an apicectomy?


a) Gutta percha
b) Amalgam
c) Mineral trioxide aggregate (MTA) « CORRECT ANSWER
d) Zinc oxide–eugenol cement (IRM)
e) Glass ionomer

• Comment on this Question

167

Yassin Al-Safadi (Safadi92@hotmail.com)


MTA, which is a derivative of Portland cement, is recommended as the material of choice, but
remains very expensive and technique sensitive.

Glass ionomer could also be used, but that would be a less effective choice than MTA.

26. Which one of these sutures could be used to close a face skin biopsy?
a) Silk
b) Polypropylene « CORRECT ANSWER
c) Steel
d) Polytetrafluoroethylene
e) Vicryl

• Comment on this Question

i
ad
Polypropylene is prolene.

27. How close is the implant to bone in osseointegration?


af
a) 5 nm
-S
b) 10 nm
c) 20 nm « CORRECT ANSWER
Al

d) 30 nm
e) 50 nm
in
ss
Ya

• Comment on this Question

20 nm is the distance from the bone edge to the implant surface in osseointegration.

28. Tracheostomy:
a) has to be formally closed after use
b) helps the cough reflex
c) is a recognised cause of hypothyroidism
d) is best placed at the first tracheal cartilage
e) may be complicated by tracheal stenosis « CORRECT ANSWER

• Comment on this Question

168

Yassin Al-Safadi (Safadi92@hotmail.com)


Tracheal stenosis following tracheostomy may occur at three possible sites – the level of the
stoma, the level of the cuff and at the tip of the tube. The incidence is approximately 10%. The
standard approach is a 2 cm transverse incision 2 cm above the sternal notch. The thyroid
isthmus may need to be tied as it lies over the second and third tracheal rings. This has no effect
on the thyroid status of the patient. In adults the tracheostomy is placed between the second
and fourth tracheal rings, and in children at the second and third tracheal rings. The cough reflex
is lost in someone with a tracheostomy and the patient is unable to clear secretions from the
tracheobronchial tree – frequent suction is therefore necessary.

29. Which one of the following is true regarding local anaesthetics?


a) Classically, Moffett’s solution contains cocaine « CORRECT ANSWER
b) Prilocaine lasts longer than bupivacaine
c) The maximum safe dose of lidocaine is 7 mg/kg
d) They are a good choice for infected tissue
e) They work better at low pH

i
ad
• Comment on this Question
af
-S
Local anaesthetics work better at higher pH, when they are less ionised. Infected tissue has an
increased blood supply, which could lead to rapid absorption and hazardous effects – hence a
Al

general anaesthetic may be a better choice. Bupivacaine lasts longer than prilocaine, which in
turn lasts longer than lidocaine; the safe dosages of lidocaine are 3 mg/kg, or 7 mg/kg with
in

adrenaline.
ss

Moffett’s solution is a cocktail of anaesthetic agents, used commonly in the pre-operative


anaesthesia of nasal polyps before removal. There are many variations but the classic recipe is 2
Ya

ml of 8–10% cocaine, 1 ml of 1:1000 adrenaline and 2 ml of 1% sodium bicarbonate, sprayed on


both sides to cover the anterior compartment mucous membranes of the nose.

30. Which one of the following is correct regarding radical neck dissection?
a) it has frozen shoulder as a late complication « CORRECT ANSWER
b) it may preserve the sternocleidomastoid muscle
c) it results in a 30-fold increase in intracranial pressure
d) it results in a 50-fold increase in intracranial pressure
e) it should not be undertaken bilaterally

• Comment on this Question

169

Yassin Al-Safadi (Safadi92@hotmail.com)


A radical neck dissection refers to the removal of lymph nodes in the anterior and posterior
triangles, along with the submandibular gland, the spinal accessory nerve, and internal jugular
vein and the sternocleidomastoid muscle. To omit at least one of these removals makes it a
modified radical neck dissection. Intracranial pressure rises threefold in a unilateral dissection,
and rises tenfold in a bilateral one – so bilateral dissections are not undertaken unless they are
completely necessary. Frozen shoulder is a late complication, although removal of the spinal
accessory nerve results in an immediate loss of shoulder shrugging.

31. Which one of the following is true concerning salivary gland neoplasms?
a) Adenocarcinoma is the most common malignant parotid tumour
b) They are 90% benign in the minor salivary glands
c) They are 20% benign in the parotid
d) They mostly originate in the parotid « CORRECT ANSWER
e) They occur bilaterally in 50% of cases of Warthin’s tumour

i
ad
• Comment on this Question af
Eighty per cent of all salivary gland tumours originate in the parotid and, of these, 80% are
-S
benign. The peak incidence of pleomorphic adenoma is in the fifth decade. In the minor salivary
glands only 50% are benign; 10% are bilateral in the case of Warthin’s tumours.
Al

Mucoepidermoid carcinoma is the most common malignant parotid tumour, followed by


adenoid cystic carcinoma.
in

32. Which of the following is correct with regard to tracheostomy?


ss

a) increases the ventilation–perfusion mismatch


b) is straightforward in people with a short neck
Ya

c) is uncomplicated by thyroid disease


d) may be needed for bronchial toilet « CORRECT ANSWER
e) will increase the anatomical dead space

• Comment on this Question

Emergency tracheostomy is a formal operation and should be carried out under controlled
circumstances under general anaesthetic. However, percutaneous tracheostomy is performed
under local anaesthetic in some centres. It is useful in enabling adequate toilet of the lungs, and
reduction in dead space may aid weaning from ventilation. Anatomical dead space of the
respiratory tract refers to all areas not involved in gas exchange, eg the oropharynx.

170

Yassin Al-Safadi (Safadi92@hotmail.com)


33. Which is the most appropriate image to use when assessing third molars?
a) Bitewing radiograph
b) Long cone CT
c) Long cone periapical radiograph
d) Orthopantomogram (OPG) « CORRECT ANSWER
e) PA mandible

• Comment on this Question

Long cone CT may be indicated following an initial assessment of the third molars on the OPG,
but OPG should be the image of choice initially.

34. Which one of the following is an absolute contraindication to the administration of 2%


lignocaine with 1:80 000 adrenaline as local anaesthesia for an oral surgery

i
ad
procedure?
a) Hypertension
b) Myocardial infarct 3 years ago
af
c) Previous coronary artery bypass graft (CABG)
-S
d) Previous ‘faint’ following administration
e) Proven allergy to the local anaesthetic « CORRECT ANSWER
Al
in
ss

• Comment on this Question


Ya

A documented and proven allergy is the only absolute contraindication. Patients with a history
of cardiac disease should be treated with care, and advice should be sought if there are any
concerns. However, if the self-aspirating technique is used, cardiac patients usually tolerate it
well.

35. Which factors are inhibited by warfarin?


a) Factors II, VII and IX
b) Factors II, VII, IX and X « CORRECT ANSWER
c) Factors II, VII and X
d) Factors II, VIII and X
e) Factors II, IX and X

171

Yassin Al-Safadi (Safadi92@hotmail.com)


• Comment on this Question

Warfarin inhibits factors II, VII, IX and X. It is a coumarin-derived drug which takes 48–72 hours
to work, as it produces altered clotting factors which lead to an increased International
Normalised Ratio.

36. Which of the following is supplied by the mandibular branch of the trigeminal nerve?
a) Motor component to the muscles of the face, cheek and scalp
b) Motor component to the muscles of mastication, the mylohyoid, the anterior belly of
the digastric muscle and the tensor veli palatini
c) Motor component to the muscles of mastication, the mylohyoid, the anterior belly of
the digastric muscle, the tensor veli palatini and the tensor typmani« CORRECT ANSWER
d) Sensory innervation to the mandibular dentition
e) Sensory innervation to the skin of the cheek, over the mandible, the lower lip, the side
of the head, the mandibular teeth and the posterior third of the tongue

i
ad
• Comment on this Question
af
-S
The mandibular branch of the trigeminal nerve supplies sensory innervation to the skin of the
cheek, over the mandible, the lower lip, the side of the head, the mandibular teeth and the
Al

anterior two-thirds of the tongue, not the posterior third.

37. n which anatomical area would it be acceptable to make a relieving incision?


in

a) Distal on sound bone to the mandibular third molar « CORRECT ANSWER


ss

b) Distal to the lower first premolar and mesial to the lower second premolar
c) In the lingual aspect of the third molar
Ya

d) In the palate for removal of an impacted canine


e) Over the canine eminence in either the mandible or the maxilla

• Comment on this Question

A distal relieving incision is commonly made to provide access for the removal of an impacted
third molar. All of the other options involve anatomical considerations and risks.

38. Which muscle is passed through when performing an inferior alveolar nerve block?
a) Buccinator « CORRECT ANSWER
b) Lateral pterygoid
c) Masseter

172

Yassin Al-Safadi (Safadi92@hotmail.com)


d) Medial pterygoid
e) Temporalis

• Comment on this Question

In performing an inferior alveolar nerve block, the buccinator muscle is pierced when aiming for
the inferior alveolar nerve.

39. What is the most likely complication during removal of a lone-standing maxillary
molar?
a) Acute facial swelling
b) Fracture of the alveolar/maxillary tuberosity « CORRECT ANSWER
c) Infection
d) Oro-antral communication

i
ad
e) Paraesthesia in the distribution of the infra-orbital nerve

af
-S
• Comment on this Question
Al

The most likely complication is a fracture. Following this there is also a risk of oro-antral
in

communication, but this is dependent upon the anatomy of the sinus floor and the amount of
bone present.
ss

40. Which of the following is a cardinal sign of alveolar osteitis (dry socket)?
Ya

a) Acute swelling
b) Bleeding
c) Pyrexia
d) Throbbing pain « CORRECT ANSWER
e) None of the above

• Comment on this Question

Throbbing pain is usually diagnostic of dry socket. All of the other signs listed are rarely, if ever,
associated with a dry socket.

41. Which of the following is a common cause of dry socket?

173

Yassin Al-Safadi (Safadi92@hotmail.com)


a) Elderly patient
b) Forceps extraction
c) Medically compromised patient
d) Oral contraceptives « CORRECT ANSWER
e) Previous treatment with bisphosphonates

• Comment on this Question

The oral contraceptive pill has been implicated in the aetiology of dry socket, but otherwise the
aetiology is unclear.

42. What is the first-line treatment for suspected anaphylactic shock?


a) Cardiopulmonary resuscitation
b) Intramuscular administration of adrenaline 1 mg/1 ml 1:1000 « CORRECT ANSWER

i
ad
c) Intramuscular administration of adrenaline 1 mg/1 ml 1:10 000
d) Intravenous administration of adrenaline 1 mg/1 ml 1:1000
e)
af
Intravenous administration of adrenaline 1 mg/1 ml 1:10 000
-S
Al

• Comment on this Question


in

Adrenaline is rarely administered intravenously, and if this treatment is required it should only
ss

be administered by an adequately trained doctor (eg an anaesthetist).


Ya

43. Which cranial nerves pass through the cavernous sinus?


a) III, IV and V
b) III, IV, V and VI « CORRECT ANSWER
c) IV, V and VI
d) IV and VI
e) VII

• Comment on this Question

The Oculomotor, Trochlear, Trigeminal and Abducens cranial nerves pass through the Cavernous
sinus.

174

Yassin Al-Safadi (Safadi92@hotmail.com)


44. Which of the following nerves is at risk during the extra-oral drainage of a
submandibular abscess?
a) Cervical branch of the facial nerve
b) Lingual nerve
c) Mandibular branch of the trigeminal nerve
d) Marginal mandibular branch of the facial nerve « CORRECT ANSWER
e) Mental nerve

• Comment on this Question

The incision should lie two finger-breadths below the inferior border of the mandible.

45. What is the mode of action of penicillin?


a) It breaks down cell walls

i
ad
b) It inhibits β-lactamases
c) It inhibits cell wall synthesis « CORRECT ANSWER
d) It inhibits transcription
af
e) It inhibits translation
-S
Al
in

• Comment on this Question


ss

Penicillin is an inhibitor of cell wall synthesis. Penicillin has an interesting mode of action: it
prevents the cross-linking of small peptide chains in peptidoglycans, the main wall polymer of
Ya

bacteria. Pre-existing cells are unaffected, but all newly-produced cells grow abnormally, unable
to maintain their wall rigidity, and they are susceptible to osmotic lysis.

46. Which of the following statements about fluconazole is correct?


a) It can be administered intravenously, as an oral tablet or as an oral suspension «
CORRECT ANSWER
b) It can only be administered intravenously
c) It can only be administered orally
d) It can only be administered orally, in either tablet or suspension form
e) It is only available as an oral suspension

175

Yassin Al-Safadi (Safadi92@hotmail.com)


• Comment on this Question

Fluconazole is a crystalline solid which is slightly soluble, and as such, can be administered
intravenously, as an oral tablet or as an oral suspension.

47. Which of the following drugs has a known effect on simvastatin?


a) Erythromycin « CORRECT ANSWER
b) Ibuprofen
c) Metronidazole
d) Paracetamol
e) None of the above

• Comment on this Question

i
ad
Erythromycin increases the plasma concentration of simvastatin, leading to an increased risk of
myalgias, rhabdomyolysis and renal failure. Avoid prescribing it if possible. Alternatively, request
the physician to stop the simvastatin temporarily if necessary.
af
-S
48. In which group of patients should inferior alveolar nerve blocks be avoided if possible?
a) Anticoagulated patients « CORRECT ANSWER
Al

b) Anxious patients
c) Immunocompromised patients
d) Patients on long-term treatment with non-steroidal anti-inflammatory drugs (NSAIDs)
in

e) Patients undergoing treatment under intravenous sedation


ss
Ya

• Comment on this Question

There is a higher risk of haematoma formation in patients who are anticoagulated. When
performing an inferior alveolar nerve block, local infiltration should be used wherever possible.

49. Which of the following investigations is required before performing surgery on a


patient who is taking clopidogrel?
a) Clotting screen
b) Full blood count
c) INR
d) Liver function tests
e) None of the above « CORRECT ANSWER

176

Yassin Al-Safadi (Safadi92@hotmail.com)


• Comment on this Question

None of the above investigations are required. An INR (international normalised ratio) is
required for patients who are warfarinised but not anticoagulated with clopidogrel, as
clopidogrel specifically and irreversibly inhibits the P2Y12 subtype of ADP receptor, which has an
important role in aggregation of platelets and cross-linking by the protein fibrin.

50. Which of the following drugs is an absolute contraindication for a patient who gives a
history of allergy to penicillin?
a) Cefalexin
b) Clindamycin
c) Co-amoxiclav « CORRECT ANSWER
d) Erythromycin

i
e) Metronidazole

ad
af
-S
• Comment on this Question
Al

Co-amoxiclav is a combination of amoxicillin and clavulanic acid. It is important to be aware that


some patients are also allergic to penicillin and cefalexin.
in

51. Which of the following cases requires antibiotic prophylaxis when undergoing a
ss

surgical dental procedure?


a) Patient on long-term steroid treatment
Ya

b) Previous history of infective endocarditis


c) Prosthetic cardiac valve replacement
d) Total knee replacement
e) None of the above « CORRECT ANSWER

• Comment on this Question

None of the above cases require antibiotic prophylaxis according to guidelines published by the
National Institute for Health and Clinical Excellence (NICE) in March 2008, Prophylaxis against
infective endocarditis: Antimicrobial prophylaxis against infective endocarditis in adults and
children undergoing interventional procedures.

177

Yassin Al-Safadi (Safadi92@hotmail.com)


52. For which of the following are excisional biopsies usually indicated?
a) All lesions
b) Lesions greater than 1 cm in diameter, where the clinical appearance suggests that the
lesion is benign
c) Lesions less than 1 cm in diameter, where the clinical appearance suggests that the
lesion is benign« CORRECT ANSWER
d) Lesions where the clinical appearance suggests that it is malignant
e) None of the above

• Comment on this Question

Lesions less than 1 cm in diameter, whose clinical appearance is considered (by an experienced
clinician) to suggest that the lesion is benign, can be excised. If there is any doubt about the

i
clinical diagnosis, an incisional biopsy should be performed.

ad
53. Which one of the following local anaesthetics, when given as a local nerve block, is
likely to cause permanent paraesthesia?
af
a) Articaine « CORRECT ANSWER
-S
b) Bupivacaine
c) Lidocaine
Al

d) Mepicavine
e) Prilocaine
in
ss
Ya

• Comment on this Question

Articaine is unique in that its initial metabolism occurs in plasma and therefore has a shorter
half-life that is safer systemically than other amides. It can therefore be used in higher
concentrations, but the risk of this is that in high concentrations and delivered as a block it
might increase the chance of permanent paraesthesia.

54. Loss of sensation in the lower lip is most likely caused by:
a) Bell’s palsy
b) Fracture of the mandible in the parasymphysis area « CORRECT ANSWER
c) Ludwig’s angina
d) Traumatic bone cyst
e) Trigeminal neuralgia

178

Yassin Al-Safadi (Safadi92@hotmail.com)


• Comment on this Question

A fracture of the mandible in the parasymphysis region may compromise the mental nerve,
thereby causing altered sensation or indeed loss of sensation in the lower lip area.

55. An oro-antral fistula (OAF) that persists for several weeks is best treated by which one
of the following?
a) Broad spectrum antibiotic therapy
b) Broad spectrum antibiotic therapy with combination ephedrine nasal drops
c) Debridement and closure with a buccal advancement flap « CORRECT ANSWER
d) Debridement and washout of the antrum via a caldwell luc
e) Ephedrine nasal drops

i
ad
• Comment on this Question
af
-S
Persistent OAFs should be debrided and closed surgically. Initial management of a suspected
OAF may include medical management such as antibiotics combined with nasal decongestants,
Al

but a persistent one should be treated surgically and medically.


in

56. Which of the following is a principle of flap surgery during surgical exodontia?
a) Broad base « CORRECT ANSWER
ss

b) Ensure closure is over the defect


c) Ensure the papillae are not in the flap
Ya

d) Limit the size of flap


e) Vertical incision

• Comment on this Question

An essential element of flap design is to ensure that there is a broad base, otherwise the blood
supply to the flap is compromised. Papillae should always be included in the flap design to
ensure good apposition; flaps should never, where possible, be closed over a defect – only over
sound bone - and they should be the size required to carry out the surgery required. Vertical
incisions can compromise the blood supply and also compromise the base size.

57. Which one of the following is an indication for surgical endodontics?

179

Yassin Al-Safadi (Safadi92@hotmail.com)


a) Inaccessible root end
b) Extruded material with clinical or radiological findings or apical periodontitis and/or
symptoms over a prolonged period« CORRECT ANSWER
c) Patient with severely compromised medical history
d) Tooth with inadequate bone and periodontal support
e) Perforation of the root or pulp chamber floor that can be treated from within the pulp
cavity

• Comment on this Question

Extruded material with clinical or radiological findings or apical periodonitis and/or symptoms
over a prolonged period of time is an indication according to the ESE guidelines in 2006 for
surgical endodontics.

i
ad
Perforation of the root or pulp chamber floor that cannot be treated through the pulp would
also be an indication, but if accessible through the pulp then an orthograde approach should be
taken.
af
-S
58. Implants are generally considered a contraindication in which group of patients?
a) Smokers
Al

b) Patients taking oral bisphosphonates


c) Patients treated with intravenous bisphosphonates « CORRECT ANSWER
in

d) Patients who suffer from diabetes mellitus


e) Patients who have previous had radiotherapy to the maxillofacial skeleton
ss
Ya

• Comment on this Question

It is thought that implants are achievable in many patients with co-morbidities providing there is
a thorough assessment and consent is obtained from the patient; some co-morbidities lead to a
higher likelihood of failure; whereas patients who are currently being treated with intravenous
bisphonates are at the highest risk of failure of osseointegration and osteonecrosis.

59. Which one of the following retractors is especially useful in the removal of maxillary
third molars?
a) Kilner retractor
b) Lasters retractor « CORRECT ANSWER
c) Minnesota retractor

180

Yassin Al-Safadi (Safadi92@hotmail.com)


d) Rake retractor
e) McKesson

• Comment on this Question

A lasters retractor is the retractor of choice for maxillary third molar teeth, allowing
displacement of the buccal mucosa and cheek, and also retraction around the maxillary
tuberosity.

60. A 33-year-old woman currently taking the oral contraceptive pill, who recently had a
difficult mandibular surgical extraction carried out, presents with pain that is
excruciating and not relieved with analgesics is most likely to be suffering from:
a) Actinomycosis
b) Alveolar osteitis « CORRECT ANSWER

i
ad
c) Bisphosphonate-induced necrosis
d) Osteoradionecrosis
e) Post-operative infection
af
-S
Al

• Comment on this Question


in

Alveolar osteitis or ‘dry socket’ is more common in patients who: are female, are on the oral
ss

contraceptive pill, have had lower extractions, have had difficult extractions, or are smokers. It is
commonly not relieved with analgesics.
Ya

181

Yassin Al-Safadi (Safadi92@hotmail.com)


[Orthodontics
and
i
ad
Pedodontics
af
-S
Al

MCQS]
in
ss
Ya

[With Answers]

182

Yassin Al-Safadi (Safadi92@hotmail.com)


1. Which one of the following is the classical extraction pattern for the serial extraction
technique?
a) Deciduous canines, first premolars then first deciduous molars
b) Deciduous canines, first permanent molars then first premolars
c) Deciduous canines, second deciduous molars then second premolars
d) Deciduous canines, second molars then second deciduous molars
e) Deciduous canines, first deciduous molars then first premolars « CORRECT ANSWER

Serial extraction was described by Kjellgren in 1948 for spontaneous improvement in incisor
alignment through planned extraction of deciduous teeth and the first permanent premolars. It
is carried out in three stages:

• All deciduous canines are extracted just as the upper lateral incisors are erupting at the age of
8.5 to 9.5 years. This process allows spontaneous incisor alignment.

• All first deciduous molars are extracted 1 year later, ideally when the roots of the first
premolar are half formed. This is to encourage eruption of the first premolars before the

i
ad
canines.

• All four first premolars are extracted as the permanent canines are erupting to provide space
af
for them to erupt.
-S
2. In facial growth, at the age of 5 years, what percentage of growth has been
completed?
Al

a) Cranium 96%, maxilla 45%, mandible 45%


b) Cranium 85%, maxilla 4%, mandible 65%
in

c) Cranium 85%, maxilla 45%, mandible 45% « CORRECT ANSWER


ss

d) Cranium 4%, maxilla 85%, mandible 45%


e) Cranium 85%, maxilla 65%, mandible 4%
Ya

Neural development is completed fairly early and therefore by the age of 5, the majority of
cranial development has been completed. Between the ages of 8–10 years 96% of cranial
development is completed and only 4% of further cranial development occurs between the ages
of 10–20 years. In contrast, the majority of maxillary and mandibular development occurs later
so that at the age of 5 years on 45% of maxillary and mandibular growth has occurred. At age 10
years 65% of maxillary and mandibular growth has been completed with a further 35% of
growth occurring between the ages of 10–20 years.

3. What is the ideal age for extraction of a lower first permanent molar to get favourable
spontaneous space closure and development of the dentition?
a) 6–8 years
b) 8–10 years « CORRECT ANSWER
c) 10–12 years
d) 12–14 years

183

Yassin Al-Safadi (Safadi92@hotmail.com)


e) 14–16 years

Ideally, the lower first molar should be extracted when there is radiographic evidence of early
dentine calcification within the second molar root bifurcation. This usually occurs within a
chronological age range of 8 to 10 years. If the molar is extracted before the age of 8 years, the
third molars may not have begun development and the second premolars may drift distally in to
the space. In addition the incisor tends to retrocline and cause an increase in the overbite. In
contrast if they are extracted at a later age then the erupting second molar tends to tip mesially
and rotate mesio-lingually.

4. Which appliance would you use to treat an 8-year-old child with an anterior crossbite
associated with an upper central incisor?
a) URA with anterior bite plane and a midline screw
b) URA with a midline screw and a T spring
c) URA with posterior bite planes and a midline screw
d) URA with an anterior bite plane and a T spring
e) URA with posterior bite planes and a T spring « CORRECT ANSWER

i
ad
The posterior bites planes disclude the occlusion anteriorly and allow the T spring to push the in-
af
standing upper central incisor labially to correct the cross bite. An anterior bite plane would
disclude the buccal segments whereas a midline screw is used to correct buccal cross bites.
-S
5. In the slide below, when a force is applied to a tooth as shown, what happens on the
Al

side labelled ‘A’?


in
ss
Ya

a) Osteoblasts making new bone


b) Osteocytes resorbing the bone
c) Osteoblasts resorbing the bone
d) Osteoclasts resorbing the bone « CORRECT ANSWER
e) Osteocytes making new bone

184

Yassin Al-Safadi (Safadi92@hotmail.com)


The bone has to resorb to allow tooth movement in the direction of the force. Osteoclasts are
the cells responsible for bone resorption.

6. For extraction of permanent first molars, which one of the following statements is
incorrect:
a) Compensating extraction of an upper first molar is often recommended when extraction
of the lower is required to prevent over-eruption of an unopposed upper first molar
b) In crowded cases with bilateral buccal segment crowding, consider a balancing
extraction to provide space and maintain the centreline
c) Balancing the extraction of healthy first molars is generally recommended in either arch
as a unilateral extraction will adversely affect the dental centreline« CORRECT ANSWER
d) Compensating extraction of an upper first molar is often recommended when extraction
of the lower is required to allow mesial movement of the lower second molar
e) In uncrowded cases, if the upper first molar is to be lost, do not compensate with
extraction of the lower first molar if it is healthy.

There is no evidence to suggest that unilateral extraction of a first permanent molar has a

i
ad
detrimental affect on the dental centreline.

af
7. In the slide below, when a force is applied to a tooth as shown, what happens on the
side labelled ‘B’?
-S
Al
in
ss
Ya

a) Osteoblasts making new bone « CORRECT ANSWER


b) Osteocytes resorbing the bone
c) Osteoblasts resorbing the bone
d) Osteoclasts resorbing the bone
e) Osteocytes making new bone

As the tooth moves in the direction of the force bone formation has to occur behind it on the
tension side. Osteoblasts are the cells responsible for bone formation.

8. Which of the following statements best describes this patient’s occlusion?

185

Yassin Al-Safadi (Safadi92@hotmail.com)


a) Class II division 1 incisor relationship with a class I molar relationship with the lower
second premolar in lingual crossbite
b) Class II division 2 incisor relationship with half a unit class II molar relationship with the
lower second premolar in buccal crossbite« CORRECT ANSWER
c) Class I incisor relationship with a full unit class II molar relationship with the lower
second premolar in buccal crossbite

i
ad
d) Class II division 1 incisor relationship with half a unit class II molar relationship with the
lower second premolar in lingual crossbite
af
e) Class II division 2 incisor relationship with a full unit class II molar relationship with the
lower second premolar in buccal crossbite
-S
This patient has a class II division 2 incisor relationship: the lower incisor edge lies posterior to
Al

the cingulum plateau of the upper incisors and the upper incisors are retroclined. In a class I
molar relationship, the mesiobuccal cusp of the upper first permanent molar occludes with the
in

mesiobuccal groove of the lower first permanent molar. In this case as the upper molar is half a
unit anterior to the lower molar the molar relationship is half a unit II. The lower second
ss

premolar is in buccal crossbite as it lies buccal to the opposing upper teeth.


Ya

9. Which of the following cephalometrical points are required for measuring the
maxillary-mandibular plane angle?
a) Sella, menton, gonion, anterior nasal spine
b) Gonion, menton, anterior nasal spine, posterior nasal spine « CORRECT ANSWER
c) Pogonion, menton, anterior nasal spine, posterior nasal spine
d) A point, pogonion, menton, posterior nasal spine
e) Posterior nasal spine, pogonion, menton, anterior nasal spine

The maxillary plane is represented by a line from the anterior nasal spine to the posterior nasal
spine. The mandibular plane is represented by a line from gonion to menton. The maxillary-
mandibular plane angle is measured at the intersection of these two lines.

10. What is the definition of the cephalometrical landmark gonion?


a) The most inferior point on the mandibular symphysis in the midline
b) The most anterior point on the mandibular symphysis

186

Yassin Al-Safadi (Safadi92@hotmail.com)


c) The most posterior inferior point on the angle of the mandible « CORRECT ANSWER
d) The lowermost point of the mandibular symphysis in the midline
e) The most posterior point on the profile of the mandible between the chin and alveolar
crest

Gonion is the most posterior inferior point on the angle of the mandible. Gnathion is the most
inferior point on the mandibular symphysis in the midline. Pogonion is the most anterior point
on the mandibular symphysis. Menton is the lowermost point of the mandibular symphysis in
the midline. The B point is the most posterior point on the profile of the mandible between the
chin and alveolar crest.

11. To what is the arrow pointing?

i
ad
af
-S
Al
in

a) Tuberculate supernumerary
b) Denticle
ss

c) Supplemental tooth
d) Odontome
Ya

e) Conical supernumerary « CORRECT ANSWER

This small conical tooth is the supernumerary most commonly found in the permanent dentition
and usually presents as a mesiodens. Although it may be found high and inverted into the palate
in a horizontal position, in most cases, the long axis of the tooth is normally inclined. The conical
supernumerary can result in rotation or displacement of the permanent incisor, but rarely
delays eruption.

12. What is the IOTN category of this patient?

187

Yassin Al-Safadi (Safadi92@hotmail.com)


i
a) 1

ad
b) 2
c) 3
d) 4
af
e) 5 « CORRECT ANSWER
-S
This patient has an infra-occluding/submerging upper deciduous second molar, an impacted
Al

lower second premolar and hypodontia due to a congenitally missing lower second premolar.
in

13. Which statement best describes how twin block appliances work?
a) A majority of skeletal effects with a minority of dental effects
ss

b) A majority of dental effects with no skeletal effects


c) A majority of dental effects with a minority of skeletal effects « CORRECT ANSWER
Ya

d) A majority of skeletal effects with no dental effects


e) A majority of skeletal and dental effects

Twin block appliances are functional appliances used in treating class II malocclusions. The
majority of treatment effects seen are due to dental changes such as upper incisor
retroclination, lower incisor proclination, distal tipping of the upper molars and mesial
movement of the lower molars. They also have minor skeletal effects such as restraint of
maxillary growth and acceleration of mandibular growth.

14. Which active component is commonly used to move a tooth mesio-distally within the
arch?
a) Palatal finger spring « CORRECT ANSWER
b) T spring
c) Robert’s retractor
d) Z spring

188

Yassin Al-Safadi (Safadi92@hotmail.com)


e) Labial bow

This spring is fabricated from 0.5 mm stainless steel and is used to move teeth mesiodistally
within the arch. T and Z springs are used to push teeth buccally whereas a Robert’s retractor and
a labial bow are used to retract the upper incisors palatally.

15.Which of the following cases is an ideal case for functional appliance therapy?
a) A 16-year-old girl with a moderate class II malocclusion and uncrowded arches
b) A 12-year-old girl with a moderate class III malocclusion and uncrowded arches
c) A 16-year-old girl with a moderate class III malocclusion and crowded arches
d) A 12-year-old girl with a moderate class II malocclusion and uncrowded arches «
CORRECT ANSWER
e) A 12-year-old girl with a class I malocclusion and crowded arches

Functional appliances are most useful in treating class II malocclusions especially when used
during the pubertal growth spurt. Older patients who have a moderate skeletal II or III
discrepancy would require orthognathic intervention to correct the discrepancy.

i
ad
16. What is the definition of anchorage?
a) The resistance to unwanted tooth movements « CORRECT ANSWER
af
b) The use of inter-maxillary elastics
-S
c) The resistance to treating a malocclusion
d) The resistance to favourable tooth movement
Al

e) The use of extra-oral traction

Anchorage is based on Newton’s third law – every force has an equal and opposite reaction.
in

When a force is applied to a tooth by a brace the reaction of this force will act on the other
ss

teeth. Anchorage is the resistance to unwanted tooth movement of these anchor teeth.

17. Fixed appliances can achieve tooth movement in all three spatial planes. What type of
Ya

tooth movement is achieved with a second order bend?


a) Bucco-lingual movement of the root apex to correct the inclination of teeth
b) Vertical movement of the crown to correct discrepancies in crown height
c) Bucco-lingual movement of the crown to correct the inclination of teeth
d) Movement in the horizontal plane to account for differences in bucco-lingual thickness
of teeth
e) Movement in the vertical plane to correct the mesio-distal angulation of teeth «
CORRECT ANSWER

A second order bend achieves movement in the vertical plane to correct the mesio-distal
angulation (or tip) of teeth. A first order bend achieves movement in the horizontal plane to
account for differences in bucco-lingual thickness of teeth (in-out discrepancies) where as a
third order bend achieves bucco-lingual movement of the root apex to correct the inclination (or
torque) of teeth.

189

Yassin Al-Safadi (Safadi92@hotmail.com)


18. If a patient has a full unit II molar relationship then:
a) The mesiobuccal cusp of the upper first permanent molar occludes with the
mesiobuccal groove of the lower first permanent molar
b) The distobuccal cusp of the upper first permanent molar occludes with the mesiobuccal
groove of the lower first permanent molar« CORRECT ANSWER
c) The mesiobuccal cusp of the upper first permanent molar occludes with the distobuccal
groove of the lower first permanent molar
d) The mesiobuccal cusp of the upper first permanent molar occludes with the distobuccal
cusp of the lower first permanent molar
e) The mesiobuccal groove of the upper first permanent molar occludes with the
distobuccal cusp of the lower first permanent molar

In a class I molar relationship, the mesiobuccal cusp of the upper first permanent molar occludes
with the mesiobuccal groove of the lower first permanent molar. Therefore in a full unit II molar
relationship, the upper molar is a full unit anterior to the lower molar and therefore the
distobuccal cusp of the upper first permanent molar occludes with the mesiobuccal groove of

i
the lower first permanent molar.

ad
19. Early loss of which upper teeth is the most likely cause of impacted upper second
premolar teeth?
af
a) Both first and second deciduous molars
-S
b) First permanent molars
c) Second deciduous molars « CORRECT ANSWER
Al

d) First deciduous molars


e) Both first and second permanent molars
in

The most likely cause is early loss of second deciduous molars, which leads to space loss because
ss

of mesial migration of the upper first permanent molar.


Ya

20. Using the Veau classification of clefts, what type of cleft is this patient most likely to
have?

190

Yassin Al-Safadi (Safadi92@hotmail.com)


a) Class I
b) Class II
c) Class III « CORRECT ANSWER
d) Class IV
e) Class V

This patient had a unilateral cleft lip and palate and therefore would be Veau class III. Veau class
I is an isolated cleft of the soft palate Veau class II is a cleft of the hard and soft palate Veau class
IV is a bilateral cleft of the lip and palate Veau class V does not exist.

21. Which of the following features are most commonly associated with a digit sucking
habit?
a) An increased overjet with a deep overbite and no crossbites
b) A unilateral crossbite with a decreased overjet and an deep overbite
c) An increased overjet with a deep overbite and a unilateral crossbite
d) A unilateral crossbite with a reduced overjet and an incomplete bite
e) A unilateral crossbite with an increased overjet and an incomplete overbites « CORRECT

i
ad
ANSWER

af
Digit sucking habits are commonly associated with a unilateral cross bite with an increased
overjet and an incomplete overbite. The finger acts like a brace and proclines the upper labial
-S
segment increasing the overjet. At the same time the finger limits eruption of the lower incisors
and may even retrocline them. The resultant effect is an incomplete overbite, or in more severe
Al

cases an anterior open bite. During sucking the tongue adopts a lower position and the cheeks
exert pressure on the buccal teeth resulting in upper arch narrowing and a cross bite.
in

22. Which of the following is correct regarding cleft lip?


ss

a) affects 1 in 500 live births


b) affects boys more than girls « CORRECT ANSWER
Ya

c) has a 25% subsequent increased risk if a previous child has been affected
d) has a declining incidence
e) is associated with palate defects in 20% of cases

Cleft lip has an increasing incidence with 1/750 affected live births. The condition is more
common in males. The subsequent risk of future affected children is 5%, rising to 9% with two
affected siblings. Cleft lip has an association with palate defects in up to 50% of cases.

23. Which of the following eruption sequences is chronologically correct in the permanent
dentition?
a) Lower canine, upper central incisor, upper canine, lower first premolar
b) Lower central incisor, lower lateral incisor, lower canine, lower first molar
c) Lower first molar, upper central incisor, lower lateral incisor, upper second molar «
CORRECT ANSWER
d) Upper first premolar, lower central incisor, lower second premolar, upper first molar

191

Yassin Al-Safadi (Safadi92@hotmail.com)


e) Upper lateral incisor, lower canine, lower central incisor, lower second premolar

The following eruption sequence is perceived to be normal in the permanent dentition:

• lower first molar

• lower central incisor

• upper first molar

• upper central incisor

• lower lateral incisor

• upper lateral incisor

• lower canine

• upper first premolar

i
ad
• lower first premolar

• upper canine af
-S
• lower second premolar

• upper second premolar


Al

• lower second molar


in

• upper second molar


ss

24. When extracting a lower 1st permanent molar for orthodontic purposes in a child
Ya

under local anaesthetic, which is the most reliable form of anaesthesia?


a) Buccal and lingual infiltration
b) Buccal infiltration
c) Inferior alveolar nerve block
d) Inferior alveolar nerve block & long buccal block « CORRECT ANSWER
e) Maxillary buccal and palatal infiltration

Infiltration anaesthesia is rarely effective in the adolescent in the posterior mandible. Inferior
alveolar nerve block alone will not anaesthetise the buccal mucosa hence cause discomfort
during extraction. The most reliable form of anaesthesia for extraction of this tooth would be
inferior alveolar nerve block & long buccal block.

25. A 6-year-old patient presents with a large unilateral ulcerated area on the buccal
mucosa in the lower right quadrant of the mouth. Two days previously they had the

192

Yassin Al-Safadi (Safadi92@hotmail.com)


lower right deciduous molars extracted under local anaesthetic. What is the most
likely diagnosis?
a) Allergy
b) Chemical burn
c) Major recurrent apthous stomatitis
d) Self-inflicted trauma « CORRECT ANSWER
e) Vesiculo bullous disease

With younger children, the feeling of numbness that comes with a local anaesthetic can be
disorientating and a sensation that they may have never felt before. Often they can bite the
anaesthetised area, either accidentally while eating or out of inquisitiveness, without feeling
pain or discomfort until the anaesthetic has worn off.

26. What is the most common cause of non-carious tooth surface loss in the primary
incisors?
a) Abfracture
b) Abrasion

i
ad
c) Attrition
d) Erosion « CORRECT ANSWER
e) Trauma
af
-S
Over 50% of primary incisors were found to have some form of erosion, according to the 1993
National Child Dental Health Survey. Abfracture are rare in children, while trauma and abrasion
Al

are seen but not as commonly as erosion.

27. A 14-year-old patient presents complaining of teeth which are constantly breaking
in

down. Clinically, they appear to have a brownish/blueish hue. Radiographically, pulp


ss

chambers have been obliterated. What is the most likely diagnosis?


a) Amelogenesis imperfecta
Ya

b) Caries
c) Chronological hypoplasia
d) Dentinogenesis imperfecta « CORRECT ANSWER
e) Molar incisor hypomineralisation

The abnormal dentine and weaker enamel:dentine junction results in frequent chipping of the
teeth. The teeth exhibit a brownish/blueish dicolouration and pulp chambers become
obliterated over time as abnomal dentine is deposited.

28. An 8-year-old patient presents with an unerupted upper left central incisor tooth and
a history of severely intruded upper left deciduous incisors at 4 years of age. What is
the most likely diagnosis?
a) Absent upper left permanent central incisor
b) Dilacerated upper left permanent central incisor « CORRECT ANSWER
c) Eruption cyst associated with the upper left permanent central incisor

193

Yassin Al-Safadi (Safadi92@hotmail.com)


d) Mesiodens supernumary tooth
e) Microdont upper left permanent central incisor

Intrusion of a primary incisor can cause the root of the tooth to come into contact with the
permanent successor. The outcome of such trauma depends on the force. In severe intrusions
the crown of the permanent successor may be displaced or its orientation altered which can
result in dilaceration.

29. What type of dental traumatic injury requires a splinting regimen of 4 weeks of rigid
splinting?
a) Dentoalveolar fracture « CORRECT ANSWER
b) Enamel dentine fractured incisor
c) Incisor root fracture
d) Intrusion
e) Luxation injury

Rigid fixation requires any splint to extend to two teeth either side of the fractured

i
ad
tooth/root/bone. Functional splinting requires any splint to extend to one tooth either side of
the fractured tooth/root. All dental trauma requiring splinting should be functionally splinted
af
only whereas dentoalveolar fractures should be rigidly splinted.
-S
30. When investigating an ectopic unerupted maxillary canine tooth, which is the most
likely finding?
Al

a) It is buccally placed
b) It is hypoplastic
c) It is in the line of the arch
in

d) It is microdont
ss

e) It is palatally placed « CORRECT ANSWER


Ya

Approximately 80% of ectopic maxillary canine teeth are found to be palatally placed.

31. A 7-year-old patient attends with a buccal sinus associated with an upper 1st
deciduous molar. Which of the following is the most appropriate definitive treatment?
a) Prescribe antibiotics
b) Incise and drain the sinus
c) Extraction of the tooth « CORRECT ANSWER
d) A ferric sulphate pulpotomy
e) Remove the caries and place an amalgam

The tooth in this case is non vital and the only definitive treatment for this tooth is to extract it.
Antibiotics are not effective treatment for localised infection such as this. Incising and draining
the sinus would put the patient through an unnecessary procedure. Removing the caries and

194

Yassin Al-Safadi (Safadi92@hotmail.com)


placing an amalgam would not address the issue of the non vital pulp. A ferric sulphate
pulpotomy is only effective in vital teeth.

32. A 4-year-old child with a history of trauma to the lower lip, presents with a soft
fluctuant fluid-filled swelling of the lower lip. Which one of the following is the most
likely cause?
a) Bohn’s Nodules
b) Minor salivary gland tumour
c) Mucocele « CORRECT ANSWER
d) Pyogenic granuloma
e) Rannula

The most common cause of a lower lip swelling is a mucocele, a collection of saliva due to
damage of the minor salivary gland or blockage of the duct. Bohn’s nodules are gingival cysts
seen in neonates. Minor salivary gland tumours are very rare and tend to present in the upper
lip. Pyogenic granuloma is a gingival condition. Rannula is essentially a mucocele but occurs in
the floor of the mouth.

i
ad
33. What is the best restorative solution for an upper 2nd deciduous molar which has a
af
mesio-occlusal cavity where breakdown of the marginal ridge has occurred in a 6-year-
old co-operative patient?
-S
a) Amalgam
b) Compomer
Al

c) Composite
d) Glass Ionomer cement
in

e) Preformed metal crown « CORRECT ANSWER


ss

Preformed metal crowns have the best longevity of restorations in class II cavities of deciduous
teeth. As the patient is co-operative, dressing the tooth with a glass ionomer cement would be
Ya

an inappropriate treatment.

34. Which of the following behaviour management techniques are not widely used in the
United Kingdom?
a) Hand over mouth « CORRECT ANSWER
b) Modelling
c) Positive reinforcement
d) Systematic desensitisation
e) Tell-show-do

In the United Kingdom, the use of physical restraint for dental treatment is unacceptable. Tell-
show-do, modelling and positive reinforcement are simple techniques which can be used
effectively in anxious children. Systematic desensitisation involves working through fears from
the mildest to the most anxiety-provoking and may be useful in cases where simple behaviour
management techniques are ineffective.

195

Yassin Al-Safadi (Safadi92@hotmail.com)


35. An 11-year-old attends your surgery having intruded their upper central incisor 7mm.
Which one of the following is the recommended treatment for this tooth?
a) Leave the tooth to re-erupt spontaneously
b) Luxate the tooth with forceps and leave to erupt
c) Reposition the tooth using a fixed orthodontic appliance
d) Reposition the tooth using a removable orthodontic appliance
e) Surgically reposition and splint the tooth « CORRECT ANSWER

As the patient is 11-years-old, the apex of the tooth will be closed. More than 6mm of intrusion
represents a severe intrusive injury and either leaving this tooth, luxating the tooth or trying to
orthodontically reposition the tooth will be ineffective. Even with surgical repositioning and
splinting of the tooth, the tooth has a poor prognosis due to root resorption and the
patient/parents/carers should be warned of this.

36. In a tooth that has sustained an enamel/dentine fracture, which of the following
occurs most commonly?
a) External inflammatory root resorption

i
ad
b) Internal inflammatory root resorption
c) Loss of vitality « CORRECT ANSWER
d) Pulp canal obliteration
af
e) Replacement resorption
-S
Despite the pulp not being initially involved in the trauma, ingress of bacteria can cause the
Al

tooth to become non vital. External inflammatory root resorption occurs in non vital teeth with
infected canals, but is initiated by damage to the periodontal ligament. Internal inflammatory
in

root resorption is also associated with non vital teeth, but occurs rarely. Replacement resorption
is associated with severe luxation or avulsion injuries especially where the tooth has been stored
ss

dry. Pulp canal obliteration is also associated with luxation injuries.


Ya

37. Which of the following medicaments are used in the promotion of apexification of an
immature non vital incisor?
a) Antibiotic/steroid paste
b) Formocresol
c) Non-setting calcium hydroxide « CORRECT ANSWER
d) Setting calcium hydroxide
e) Zinc oxide eugenol cement

Non-setting calcium hydroxide has a long history of use in apexification. Antibiotic/steroid paste
can be used in cases where there is infection, but do not promote apexification. Formocresol
and zinc oxide eugenol cements have been used in non vital pulp therapy of deciduous teeth.

38. Which of the following medical conditions is not associated with delayed eruption of
teeth?
a) Cleidocranial Dysplasia

196

Yassin Al-Safadi (Safadi92@hotmail.com)


b) Down syndrome
c) Hereditary gingival fibromatosis
d) Hyperthyroidism « CORRECT ANSWER
e) Turner’s syndrome

Hyperthyroidism can be associated with premature eruption of teeth.

39. An 8-year-old patient attends with a painless lower right 1st permanent molar which
requires extraction. The patient has a Class I occlusion with no crowding. What feature
will determine the ideal time to extract the tooth to maximise space closure?
a) Clinically, the lower 2nd molar is fully erupted
b) Clinically, the lower right 2nd premolar is fully erupted
c) Radiographically, none of the root of the lower right 2nd molar has formed
d) Radiographically, the bifurcation of the lower right 2nd molar has formed « CORRECT
ANSWER
e) Radiographically, the roots of lower right 2nd molar are fully formed

i
ad
When extracting a lower 1st permanent molar in a Class I case where there is no crowding, the
aim is to ensure that as much space closure occurs as possible. From a clinical point of view,
af
once the 2nd premolar and 2nd permanent molar have erupted, it is too late for space closure
to occur. Optimum extraction time is judged radiographically and this is when furcation of the
-S
2nd permanent molar has just formed. If none of the root has formed, the
Al

space closure does not tend to occur.

40. A 2-year-old attends your practice. His parents report a one week history of fever and
in

oral pain, and then 2 days ago they noticed that he had red and ulcerated gums. He
ss

has no other lesions on his body. What is the most likely causative agent?
a) Herpes Simplex virus I « CORRECT ANSWER
Ya

b) Herpes Simplex virus II


c) Fusobacterium nucleatum
d) Spirochete bacteria
e) Varicella zoster virus

Herpes simplex virus II is usually associated with genital herpes. Varicella zoster virus is
commonly known as ‘chicken pox’; approximately 50% of children with varicella have oral
lesions, but usually have other lesions on other parts of their body. Fusobacterium and
Spirochates are associated with NUG which is not commonly seen in children of this age.

41. Histologically, what percentage of primary molars where the loss of the marginal ridge
has occurred have been found to have irreversible pulpitis?
a) Less than 20%
b) Between 20% and 30%
c) Between 30% and 40%

197

Yassin Al-Safadi (Safadi92@hotmail.com)


d) Between 40% and 50%
e) Over 50% « CORRECT ANSWER

Various studies have demonstrated this, the latest being Duggal et al. 2002. It underlines the
importance of pulp therapy when restoring primary molars.

42. With regard to young people of 16- and 17-years-old, which one of the following is
true?
a) They are able to consent or refuse treatment and this cannot be overridden even if it is
considered in their best interests
b) They are able to refuse dental treatment, but their parents/guardians must consent to
dental treatment on their behalf
c) They are entitled to consent to their own dental treatment, but should they refuse
treatment, this can be overridden in some cases where it is felt to be in their best
interests« CORRECT ANSWER

i
ad
d) They are not entitled to consent to their own treatment under any circumstances
e) They are subject to Gillick competence
af
Young people are considered to be adults at the age of 16 years and therefore consent to their
-S
own treatment. However 16- and 17-year-olds are not able to refuse treatment and their wishes
in this respect can be overridden by the Courts if it is felt to be in their best interests. In reality,
Al

there are few cases in dentistry where this is likely to occur. Gillick competence only applies to
children under the age of 16 years.
in

43. Which one of the following medications has been associated with gingival
ss

enlargement in children?
a) Carbamazepine
Ya

b) Clonazepam
c) Lamotrigine
d) Phenytoin « CORRECT ANSWER
e) Sodium valproate

Gingival enlargement is seen in approximately 50% of patients. Fortunately, other newer anti-
epileptic medication such as carbamazepine or sodium valproate do not cause gingival
enlargement.

44. A mother attends the dental practice with her 4-year-old son. She wants some advice
on how much and what type of toothpaste to use. They live in an area with no water
fluoridation and you assess that her son is a low caries risk. Which one of the
following do you advise?
a) Brush his teeth twice daily with a pea sized amount of 600ppm fluoride toothpaste
b) Brush his teeth twice daily with a pea sized amount of 1000ppm fluoride

198

Yassin Al-Safadi (Safadi92@hotmail.com)


c) Brush his teeth twice daily with a pea sized amount of 1350-1500ppm fluoride
toothpaste « CORRECT ANSWER
d) Brush his teeth twice daily with a smear of 1000ppm fluoride toothpaste
e) Brush his teeth twice daily with a smear of 1350-1500ppm fluoride toothpaste

Low fluoride toothpastes (600ppm fluoride) are no longer recommended. For children under 3
years of age, twice daily brushing with a smear of 1000ppm is advised. For those between 3 and
6 years old, twice daily brushing with a pea sized amount of 1350-1500ppm paste is
recommended. The patient should spit, but not rinse with water after brushing as the effect of
the fluoride is reduced.
(http://www.dh.gov.uk/en/Publicationsandstatistics/Publications/PublicationsPolicyAndGuidanc
e/DH_102331)

45. What is defined as the overlap of incisors in the horizontal plane?


a) Anterior open bite
b) Cross bite
c) Lateral open bite

i
ad
d) Overbite
e) Overjet « CORRECT ANSWER

This is the definition of overjet.


af
-S
Overbite refers to the extent of vertical overlap of the maxillary central incisors over the
Al

mandibular central incisors.

Crossbite is an occlusal irregularity where a tooth (or teeth) has a more buccal or lingual position
in

than its corresponding tooth in the upper or lower arch.


ss

46. What is the definition of a Class II division 1 malocclusion?


a) The lower incisor edges lie anterior to the cingulum plateau of the upper incisors and
Ya

the upper central incisors are proclined or of average inclination


b) The lower incisor edges lie posterior to the cingulum plateau of the upper incisors and
the upper central incisors are proclined or of average inclination« CORRECT ANSWER
c) The lower incisor edges lie posterior to the cingulum plateau of the upper incisors and
the upper central incisors are retroclined
d) The upper incisor edges lie posterior to the cingulum plateau of the upper incisors and
the upper central incisors are retroclined
e) The upper incisors are retroclined and there is an increase in overjet

By definition, in a Class II division 1 malocclusion the lower incisor edges must lie posterior to
the cingulum plateau. The upper incisors may be proclined or of average inclination.

47. What are the circumstances in which a Class I canine relationship occurs?

199

Yassin Al-Safadi (Safadi92@hotmail.com)


a) The upper permanent canine occludes anterior to the embrasure between the lower
permanent canine and first premolar
b) The upper permanent canine occludes in the embrasure between the lower first and
second premolars
c) The upper permanent canine occludes in the embrasure between the lower permanent
canine and first premolar« CORRECT ANSWER
d) The upper permanent canine occludes in the embrasure between the lower permanent
canine and lateral incisor
e) The upper permanent canine occludes posterior to the embrasure between the lower
permanent canine and first premolar

By definition, a Class I canine relationship is one in which the upper permanent canine occludes
in the embrasure between the lower permanent canine and first premolar.

48.Which of the following statements about the Leeway space is correct?


a) It is always greater in the upper arch than in the lower one
b) It is usually equal in the upper and lower arches

i
ad
c) It relates to the total space taken up by the combined mesiodistal widths of the
deciduous canine, first and second premolar teeth
af
d) It relates to the difference between the combined mesiodistal widths of the permanent
canine, first and second premolar teeth that is greater than that of their deciduous
-S
precursors
e) It relates to the difference between the combined mesiodistal widths of the permanent
Al

canine, first and second premolar teeth that is less than that of their deciduous
precursors« CORRECT ANSWER
in

The Leeway space relates to the difference between the combined mesiodistal widths of the
ss

permanent canine, first and second premolar teeth that is less than that of the deciduous
precursors. The Leeway space is also greater in the lower arch, which enables the lower first
Ya

permanent molar to drift further mesially than the upper first permanent molar. This allows the
molar relationship in the developing dentition to change from a cusp-to-cusp to a Class I
relationship.

49. A patient presents with an impacted and unerupted permanent canine tooth. What
would their IOTN be?
a) 1 (none)
b) 2 (little)
c) 3 (moderate)
d) 4 (great)
e) 5 (very great) « CORRECT ANSWER

Impacted unerupted teeth have an Index of Orthodontic Treatment Need (IOTN) of 5i.

50. Which of the following ANB values would represent a Class III malocclusion?

200

Yassin Al-Safadi (Safadi92@hotmail.com)


a) 0° « CORRECT ANSWER
b) 2°
c) 3°
d) 4°
e) 6°

An ANB value of < 2° represents a Class III malocclusion.

51. Which of the following types of tooth movement can be achieved with a removable
appliance?
a) Bodily movement
b) Correction of rotation
c) Rapid maxillary expansion
d) Tipping « CORRECT ANSWER
e) Torque

Tipping is the only type of tooth movement that can be achieved with a removable appliance. All

i
ad
of the others require a fixed appliance or, in the case of rapid maxillary expansion, a fixed
expansion device.

52.
af
Which of the following statements about rapid maxillary expansion is true?
-S
a) It can be used in the mandible
b) It can only be carried out in adults
Al

c) It can only be carried out in the deciduous dentition


d) It involves expansion of the mandibular symphysis
e) It is a form of distraction osteogenesis « CORRECT ANSWER
in
ss

Rapid maxillary expansion involves distracting the two halves of the maxilla apart using a
maxillary expander to allow new bone to form within the midpalatal suture.
Ya

53. What is the angulation of the maxillary–mandibular planes angle (MMPA)?


a) 27° « CORRECT ANSWER
b) 45°
c) 93°
d) 109°
e) 126°

This is the standard angulation ± 5°.

54. Which of the following is a component of removable appliances?


a) Active self-ligating system
b) Adam’s clasp « CORRECT ANSWER
c) Bracket
d) Module

201

Yassin Al-Safadi (Safadi92@hotmail.com)


e) Passive self-ligating system

An Adam’s clasp is a form of retention for removable appliances.

55. Which one of the following conditions is associated with premature loss of teeth?
a) Down syndrome
b) Hereditary gingival fibromatosis
c) Hypophosphatasia « CORRECT ANSWER
d) Hypothyroidism
e) Williams syndrome

Hypophosphatasia is an inborn error of metabolism in which there is a deficiency of the enzyme


alkaline phosphatase, which is involved in hard tissue formation. This condition results in bone
and cartilage defects. The other conditions are associated with delayed eruption of teeth or
failure of eruption of teeth.

56. Which teeth are affected by caries in the condition known as ‘nursing-bottle decay’?

i
a) All of the primary teeth

ad
b) Only the upper anterior teeth
c) Only the upper central incisors af
d) The lower anterior teeth
-S
e) The upper anterior teeth and the upper first primary molar teeth « CORRECT ANSWER

Nursing decay is a pattern of decay that typically affects the upper anterior teeth and upper first
Al

primary molar teeth. Occasionally the lower first primary molar teeth may be affected. The
lower anterior teeth are spared, due to the tongue.
in

57. Which of the following medicaments is recommended for primary vital pulp therapy?
ss

a) Beechwood creosote
b) Cresophene
Ya

c) Ferric sulphate « CORRECT ANSWER


d) Setting calcium hydroxide
e) Sodium hypochlorite

Ferric sulphate is a haemostatic agent that has gained popularity as a vital pulpotomy agent. It is
thought to work by causing agglutination of blood proteins due to the reaction of blood with
both ferric and sulphate ions. These agglutinated proteins form plugs that occlude the
capillaries. The use of beechwood creosote is no longer advocated. The International Agency for
Research on Cancer (IARC) has classified formaldehyde as a carcinogen. Therefore although it
has not been banned, it is not recommended as the first choice of treatment. Sodium
hypochlorite is used as an irrigant in non-vital teeth during root canal therapy. Calcium
hydroxide is not used in vital pulp therapy in primary teeth, nor is Cresophene used. The other
material that has been shown to have a high success rate in vital pulpotomy of primary teeth is
mineral trioxide aggregate (MTA).

202

Yassin Al-Safadi (Safadi92@hotmail.com)


58. Which virus causes hand, foot and mouth disease?
a) Coxsackie A virus « CORRECT ANSWER
b) Epstein–Barr virus
c) Herpes simplex virus
d) Influenza virus
e) Mumps virus

Hand, foot and mouth disease is caused by the Coxsackie A virus. It usually affects infants and
children. It is spread through direct contact with the mucus, saliva or faeces of an infected
person. It typically occurs in small epidemics in schools, usually during the summer and autumn
months. The normal incubation period is 3–7 days.

59. What is the definition of an extrusion injury?


a) Abnormal loosening of the tooth but no displacement
b) Complete displacement of the tooth from its socket
c) Displacement of the tooth into the alveolar bone
d) No abnormal loosening or displacement, but tenderness when the tooth is percussed

i
ad
e) Partial displacement of the tooth from its socket « CORRECT ANSWER

• af
Avulsion is the complete displacement of the tooth from its socket.
-S
• Intrusion is the displacement of the tooth into the socket with associated fracture of the
socket wall.
Al

• Lateral luxation is displacement of the tooth away from its socket in a direction other
than axial.
in

• Subluxation is abnormal loosening of the tooth, but without any displacement of the
ss

tooth.
Ya

• In concussion there is no abnormal loosening or displacement of the tooth, but the


tooth is tender to percussion.

60. What is the definition of a complicated crown fracture?


a) A fracture of the enamel and dentine that involves the pulp « CORRECT ANSWER
b) A fracture that involves the dentine, cementum and pulp
c) An incomplete crack in the enamel without loss of tooth substance
d) Loss of only the enamel
e) Loss of only the enamel and dentine

• A complicated crown fracture involves the enamel, dentine and pulp.

• Uncomplicated fractures do not involve the pulp.

203

Yassin Al-Safadi (Safadi92@hotmail.com)


• An incomplete crack in the enamel surface without loss of tooth tissue is known as an
infraction.

• Root fractures are defined as affecting the cementum, dentine and pulp.

61. What is the prevalence of non-syndromic cleft lip and palate in the UK?
a) 1 in 20
b) 1 in 300
c) 1 in 400
d) 1 in 700 « CORRECT ANSWER
e) 1 in 1000

In the UK the prevalence of cleft lip and palate is 1 in 700 live births. The incidence is between 1
in 300 and 1 in 1000. The prevalence varies according to family history, ethnicity, gender and
geographical origin. Cleft lip and cleft palate are among the most common congenital

i
abnormalities.

ad
62. In embryology, when does fusion of the secondary palate occur?
a) At 2–3 weeks in utero
af
b) At 3–4 weeks in utero
-S
c) At 4–5 weeks in utero
d) At 5–6 weeks in utero
Al

e) At 7–8 weeks in utero « CORRECT ANSWER


in

The secondary palate forms as a result of fusion of the palatal shelves. These develop as
extensions of the primary palate, which closes at 5–6 weeks. The primary palate is formed by
ss

fusion of the lateral nasal process with the medial nasal process. The lip is formed by fusion of
the maxillary processes. Failure of these processes will result in clefts of the lip, alveolus and
Ya

palate.

63. Which of the following statements about fluoride is correct?


a) All children under the age of 5 years should use toothpaste with a fluoride
concentration of 500 ppm
b) Fluoride supplements are recommended for all children
c) It exerts its anticariogenic effects mainly pre-eruptively
d) It is more effective in preventing pit and fissure caries than in preventing caries on
smooth surfaces
e) The certain lethal dosage is 32–64 mg/kg « CORRECT ANSWER

• Fluoride exerts its anticariogenic effects primarily post-eruptively, and is effective on


smooth surfaces.

204

Yassin Al-Safadi (Safadi92@hotmail.com)


• Fluoride supplements are recommended for children who are deemed to be at high risk
and who live in an area where the concentration of fluoride in the water is less than 0.3 ppm.

• It is recommended that all children use toothpaste with a fluoride concentration of


1000–1500 ppm.

64. Which of the following materials has been shown to be the clinically most successful
restoration for a class II cavity in a primary molar tooth?
a) Compomer
b) Composite
c) Glass ionomer
d) Stainless steel crown
e) None of the above « CORRECT ANSWER

A recent Cochrane Review has shown that no one restoration is better than another for

i
restoration of primary teeth. Stainless steel crowns have been shown to have higher success

ad
rates in primary molars in different studies, but the evidence is still not conclusive.

65.
af
Which condition is associated with early onset periodontal disease?
a) Amelogenesis imperfecta
-S
b) Dentinogenesis imperfecta
c) Ehlers Danlos syndrome « CORRECT ANSWER
Al

d) Haemophilia A
e) Treacher Collins syndrome
in

Ehlers Danlos syndrome is a group of connective tissue disorders affecting the formation of
ss

collagen. There are 10 different subtypes of this condition. Inheritance can be autosomal
dominant, autosomal recessive or X-linked depending on the subtype. The condition is
Ya

characterised by hyperextensible skin, tendency to bruise and hypermobility of joints.


Premature exfoliation of teeth due to early onset periodontal disease is associated with type
VIII.

66. Which one of the following statements is correct concerning molar incisor
hypomineralisation (MIH)?
a) MIH affects incisors only
b) MIH affects more males than females
c) MIH is a genetic condition
d) MIH typically affects the first permanent molars and incisors « CORRECT ANSWER
e) Teeth have an opalescent appearance

Molar incisor hypomineralisation can be defined as hypomineralisation of systemic origin of one


to four first permanent molars, associated frequently with affected incisors. The expression of

205

Yassin Al-Safadi (Safadi92@hotmail.com)


this can vary from patient to patient, and also in the mouth, with a molar in one quadrant being
mildly affected while in another quadrant a molar is severely affected, resulting in post-eruptive
breakdown.

67. Which of the following is the ‘active component’ in an upper removal appliance to
correct a localised anterior crossbite between UR1 and LR1? Please select one option.
a) Adams clasps
b) Baseplate
c) Posterior bite plane
d) Southend clasp
e) Z-spring « CORRECT ANSWER

Anterior crossbites are used to correct localised anterior crossbites. The principles of removable
appliance design can be summarised using the acronym ARAB.

A is the active component. Springs are most commonly used. Z springs or T springs are used. The
smallest diameter of the wire used is 0.5 mm.

i
ad
R is retention of the appliance. Typically Adams clasps are used and these engage into the
undercut on fully erupted molars or premolars. They are fabricated from 0.7 mm stainless steel
af
wire.
-S
A is anchorage. This is defined as the source of resistance to the forces generated in reaction to
the active components of the appliance. It is required to prevent unwanted movements.
Al

B is baseplate. This connects all the components of the appliance.


in

The active component is therefore the Z-spring, which will procline the UR1.
ss

68. Which one of the following is the most likely cause of an asymmetrical anterior open
Ya

bite (AOB):
a) Bilateral condylar fracture
b) Digit sucking habit « CORRECT ANSWER
c) Endogenous tongue thrust
d) Incompetent lips
e) Skeletal development

Digit sucking habits may result in AOBs. The other options are likely to result in a more
symmetrical distribution of the AOB, apart from incompetent lips which will not directly cause
an AOB. The incidence of digit sucking as a cause of AOB decreases with age (12% at 9 years of
age, and 5% at 12 years of age). A habit of 6 hours or more per day can develop a malocclusion.

69. The British Standards (1983) incisor classification for a Class II Division 2 incisor
relationship is:

206

Yassin Al-Safadi (Safadi92@hotmail.com)


a) The lower incisor edge lies anterior to the cingulum plateau of the upper incisors. The
overjet is reduced or reversed.
b) The lower incisor edges lie anterior to the cingulum plateau of the upper incisors. The
overjet may be reduced, normal or increased, and the upper incisors are retroclined.
c) The lower incisor edges lie posterior to the cingulum plateau of the upper incisors. The
upper central incisors are proclined or of average inclination and there is an increase in
overjet.
d) The lower incisor edges lie posterior to the cingulum plateau of the upper incisors. The
upper central incisors are retroclined. The overjet is usually minimal or may be
increased.« CORRECT ANSWER
e) The lower incisor edges occlude with or lie immediately below the cingulum plateau of
the upper incisors.

In Class II Division 2 incisor relationships, the upper central incisors are usually retroclined, and it
is common for the upper lateral incisors to be proclined. The Class II element comes from the
lower incisor edges being posterior to the cingulum plateau of the upper central incisors. The

i
overjet depends on the degree of retroclination of the upper incisors.

ad
70. Which one of the following is an example of an upper removable appliance (URA):
a) Clark twin blocks
af
b) ELSAA (expansion and labial segment alignment appliance) « CORRECT ANSWER
-S
c) Frankel II appliance
d) Herbst appliance
Al

e) Transpalatal arch
in

The ELSAA is a URA that can be used prior to functional appliance therapy in order to align and
procline the upper incisors. This is useful to get full advancement of the mandible when you are
ss

in the functional appliance phase. Full advancement of the mandible will allow for greater
correction of the overjet with your functional appliance. The Frankel II, Herbst and Clark twin
Ya

blocks are all functional appliances. The transpalatal arch is a fixed anchorage reinforcement
device.

71. Which one of the following options shows the correct frequency of developmentally
absent teeth (from most to least):
a) Mandibular central incisors > maxillary lateral incisors > mandibular second premolars >
maxillary second premolars
b) Maxillary lateral incisors > mandibular second premolars > maxillary second premolars >
mandibular central incisors
c) Maxillary second premolars > mandibular second premolars > maxillary lateral incisors >
mandibular central incisors
d) Mandibular second premolars > maxillary lateral incisors > maxillary second premolars >
mandibular central incisors« CORRECT ANSWER

207

Yassin Al-Safadi (Safadi92@hotmail.com)


e) Mandibular second premolars > maxillary second premolars > maxillary lateral incisors >
mandibular central incisors

Although the frequency of congenitally absent teeth varies between ethnic groups, the
mandibular second premolars (1–6%) are affected more than the maxillary lateral incisors (1–
4%). Agenesis of the mandibular incisors is rare (0.08–0.23%), as are missing maxillary and
mandibular canines along with first molars, which are usually associated with severe
hypodontia.

72. Which one of the following is correct concerning haemophilia A?


a) Affects females more than males
b) Affects 1:1000 individuals
c) Is also known as Christmas disease
d) Is an X-linked recessive disorder « CORRECT ANSWER
e) Is less common that haemophilia B

Haemophilia A is an X-linked recessive disorder affecting 5 per 100,000 individuals. It is due to

i
ad
deficiency in factor VIII. As it is X-linked, males are affected and females are carriers. It is 10
times more common than haemophilia B (Christmas disease). All daughters of an affected male
af
will be carriers. Sons of a carrier have a 50% chance of having the disease.
-S
Al
in
ss
Ya

208

Yassin Al-Safadi (Safadi92@hotmail.com)


[Periodontics
i
MCQs] ad
af
-S
Al

[With Answers]
in
ss
Ya

209

Yassin Al-Safadi (Safadi92@hotmail.com)


1. Look at the radiograph below and choose one option that best reflects the BPE score
for the lower right sextant.

i
ad
af
-S
Al

a) 3
b) 1
c) 2
in

d) 4
ss

e) * « CORRECT ANSWER
Ya

• Comment on this Question

In the radiograph shown above a furcation can clearly be seen. The BPE recording would then be
that of a star and will require further periodontal assessment.

2. A periodontal probing depth of 4 mm on the basic periodontal examination would be


classified as which one of the following?
a) 0
b) 1
c) 2
d) 3 « CORRECT ANSWER
e) 4

210

Yassin Al-Safadi (Safadi92@hotmail.com)


• Comment on this Question

The BPE uses the CPITN probe with its first black band between 3.5 and 5.5 mm. The probing
depth between this band would give a BPE of 3, providing no other probing depth of that
sextant is greater than 5.5.

3. Which one of the following would not be considered a plaque retentive factor?
a) Subgingival calculus
b) Well fitting crown « CORRECT ANSWER
c) Severe imbrication of the teeth
d) Supragingival calculus
e) Fixed orthodontic retention

i
ad
• Comment on this Question
af
A well fitting crown would not be deemed to be a plaque retentive factor; however a poorly
-S
fitted crown would be deemed a plaque retentive factor.

4. Which one of the following is the greatest risk factor for periodontal disease?
Al

a) Plaque retentive factors


b) Down’s syndrome
in

c) Shortened dental arch


d) Smoking « CORRECT ANSWER
ss

e) Intravenous drug abuse


Ya

• Comment on this Question

Smoking is well documented as the biggest risk factor in periodontal disease.

5. Which one of the following is the correct name for the instrument in the picture
below.

211

Yassin Al-Safadi (Safadi92@hotmail.com)


a) Periodontal hoe
b) Periodontal curette
c) Gracey curette
d) Reverse action hoe

i
ad
e) Sickle scaler « CORRECT ANSWER

af
-S
• Comment on this Question
Al

The instrument photographed above is a Sickle scaler.


in

6. Which one of the following predictable treatments of this recession defect is correct?
ss
Ya

212

Yassin Al-Safadi (Safadi92@hotmail.com)


i
ad
a) Rotational flaps
af
-S
b) Palatal soft tissue graft
c) Extraction of the tooth and replacement with a conventional bridge
Al

d) Emdogain with advancement flap « CORRECT ANSWER


e) Membrane placement over the defect
in
ss
Ya

• Comment on this Question

Emdogain is a well documented, effective and easy-to-apply gel that supports the predictable
re-growth of hard and soft tissues, leading to the regeneration of tooth attachment.

7. In the absence of oral hygiene measures, gingival inflammation is detectable at:


a) 2h
b) 24 h
c) 48 h « CORRECT ANSWER
d) 2 weeks
e) 2 months

213

Yassin Al-Safadi (Safadi92@hotmail.com)


• Comment on this Question

Following the absence of oral hygiene measures, gingivitis is histologically detectable at 48 h.


Prevention of this inflammation would involve the total removal of all plaque within 48 h.

8. Chlorhexidine is a chemical antiseptic. Which one of the following statements is false?


a) It is most effective immediately after the use of toothpaste « CORRECT ANSWER
b) Can be present in gels and rinses
c) Its mechanism of action is membrane disruption
d) Is bactericidal as well as bacteriostatic
e) Activity is reduced in the presence of blood, soap and pus

• Comment on this Question

i
ad
Chlorhexidine is neutralized by common toothpaste additives such as sodium lauryl sulphate
and sodium monofluorophosphate. It is best to keep a 30-min to 2-h interval between brushing
and using the mouthwash. af
-S
9. Which one of the following statements with regard to biofilms is correct?
a) The initial adhesion of the micro-organisms is via structures such as pili
Al

b) All species must be able to adhere to the tooth surface


c) The external bacteria become anaerobic
d) The biofilm is held together and protected by extracellular matrix « CORRECT ANSWER
in

e) The composition of the biofilm is not dependant upon position in the mouth
ss
Ya

• Comment on this Question

The initial adhesion of micro-organisms is via Van der Waals force. Some species are not able to
attach to the tooth surface but can anchor themselves either to the matrix or the colonists. The
internal bacteria become anaerobic and start to produce acids. The biofilm will be dependant
upon the position within the mouth.

10. Which one of the following statements about the periochip is correct?
a) The periochip is used in the treatment of periodontitis « CORRECT ANSWER
b) The periochip is completely synthetic
c) Use is confined to pockets less than 5 mm
d) Is associated with an increase in pocket probing depths
e) Is an atopical antibiotic

214

Yassin Al-Safadi (Safadi92@hotmail.com)


• Comment on this Question

The periochip contains chlorhexidine gluconate in a biodegradable matrix of hydrolysed gelatin.


It may be used in pockets of all depths to provide an antimicrobial action. It can be useful in the
reduction of periodontal pockets.

11. The most predictable use of guided bone regeneration occurs in which one of the
following?
a) One-walled defects
b) Two-walled defects
c) Three-walled defects « CORRECT ANSWER
d) Grade 2 furcation defects
e) Grade 3 furcation defects

i
ad
af
-S
• Comment on this Question

The most predictable use of guided bone regeneration occurs in three-walled defects.
Al

12. Which one of the following statements regarding the platform switching concept in
in

implant dentistry is FALSE?


a) The platform switching concept uses an increased diameter of abutment compared to
ss

the implant size« CORRECT ANSWER


b) The platform switching concept uses a reduced diameter of abutment compared to the
Ya

implant size
c) The platform switching concept is associated with a reduction in crestal bone resorption
d) The platform switching concept can be used in both submerged and unsubmerged
surgical techniques
e) The platform switching concept provides a horizontal platform to offset the biological
width surrounding a dental implant

• Comment on this Question

The platform switching concept involves using a reduced diameter of abutment compared to
the implant size, e.g. placing a 3.8 mm-wide abutment on a 4.8 mm-wide implant. It may be

215

Yassin Al-Safadi (Safadi92@hotmail.com)


used both as a submerged or unsubmerged implant. It is associated with a reduction in crestal
bone resorption due to horizontally offsetting the biological width.

13. Which one of the following statements regarding bisphosphonates is true?


a) The use of oral bisphosphonates contraindicates the use of dental implants
b) Bisphosphonates increase the osteoblastic activity
c) Bisphosphonates can only be given intravenously
d) Bisphosphonates are contraindicated in osteoporosis
e) Bisphosphonates encourage osteoclasts to undergo apoptosis « CORRECT ANSWER

• Comment on this Question

Bisphosphonates are a class of drugs that prevent the loss of bone mass, used to treat
osteoporosis and similar diseases. They may be administered orally or intravenously, which

i
ad
encourage osteoclasts to undergo apoptosis. Current research would indicate that the
placement of implants in patients taking oral bisphosphonates is not contraindicated. The use of
af
iv bisphosphonates does contraindicate the placement of dental implants as it affects the ability
of the implant to osseointegrate.
-S
14. Which one of the following statements regarding Aggregatibacter
Al

actinomycetemcomitans (AA) is false?


a) AA is a gram negative non motile rod
in

b) AA is a leucotoxin killing polymorphonuclear leukocytes (PMNs) and monocytes


c) AA does not invade tissues « CORRECT ANSWER
ss

d) AA is particularly associated with the onset of localized aggressive periodontitis


e) AA may be present as part of the biofilm in periodontally involved teeth
Ya

• Comment on this Question

AA can invade tissues due to its leucotoxin ability.

15. Which one of the following is associated with the greatest increase in risk of
periodontal disease?
a) Malocclusion
b) Pregnancy
c) Poor diet
d) Type 1 diabetes mellitus « CORRECT ANSWER

216

Yassin Al-Safadi (Safadi92@hotmail.com)


e) Type 2 diabetes mellitus

• Comment on this Question

Both type 1 and type 2 diabetes mellitus are associated with periodontal disease. In particular,
poorly controlled diabetes. Type 1 does have an increase in attachment loss compared to type 2.

16. Which one of the following is the correct definition for a xenograph?
a) Tissue transplanted from the same individual, same site
b) Tissue transplanted from the same individual, different site
c) Tissue transplanted from a different individual, same species
d) Tissue transplanted from a different species « CORRECT ANSWER
e) Synthetically constructed tissue
17. Buccal recession associated with implants is most commonly associated with which

i
ad
one of the following?
a) Greater than 2mm of buccal plate
b)
af
Immediately placed implants « CORRECT ANSWER
c) Thick gingival biotype
-S
d) Platform switched implants
e) All ceramic restorations
Al
in
ss

• Comment on this Question


Ya

Immediately placed implants have a 20% risk of aesthetic compromise. Following extraction, the
blood supply to the labial or buccal bone is disrupted with associated loss of buccal plate.
Platform switched implants are associated with a reduction in crestal bone loss and there is no
difference between ceramic and conventional abutments.

18. Which one of the following is the most reliable indicator of active periodontal
disease?
a) Bleeding on probing « CORRECT ANSWER
b) Presence of subgingival calculus
c) Probing depths of over 7 mm
d) DNA presence of periodontal pathogens
e) Plaque

217

Yassin Al-Safadi (Safadi92@hotmail.com)


• Comment on this Question

At present, the most reliable indicator of active periodontal disease is bleeding on probing.
Presence of periodontal pathogens does not indicate any active disease. The probing depths
although associated with periodontal disease are not an indicator of current or future disease.

19. Which one of the following is the correct definition of osseointegration?


a) Osseointegration is the direct structural and functional connection between living bone
and the surface of a load-bearing artificial implant« CORRECT ANSWER
b) Osseointegration is the fusion of a titanium implant to bone
c) Osseointegration is the connection of a titanium implant to living bone
d) Osseointegration is the direct structural and functional connection between living bone
and the surface of an artificial implant
e) Osseointegration is the seamless fusion of an autogenic graft to the donor site

i
ad
• Comment on this Question
af
-S
Option A is the universally accepted definition of osseointegration, as described by Branemark.
Al

20. Which one of the following treatments would not be considered to be in the initial
phase of perimplantitis treatment?
in

a) Occlusal evaluation and equilibration


b) Anti-smoking advice
ss

c) Subgingival irrigation with 0.12% chlorhexidine


Ya

d) Non-surgical debridement
e) Surgical debridement « CORRECT ANSWER

• Comment on this Question

As in the treatment of periodontal disease, the initial treatment will involve non-surgical anti-
infective therapy. This may be sufficient to re-establish gingival health or may be followed by
surgical therapy.

21. Which one of the following statements about the Florida probe is incorrect?
a) Reduces inter- and intra-operator error by using a constant force
b) Records the pocket depth and not attachment loss

218

Yassin Al-Safadi (Safadi92@hotmail.com)


c) The tip is placed at the bottom of the sulcus
d) The sleeve is positioned at the gingival margin
e) The correct measurement is visually assessed « CORRECT ANSWER

• Comment on this Question

The Florida probe uses a constant force of 15N and is a computerised probe. The tip is placed at
the bottom of the sulcus and the blue sleeve positioned at the gingival margin. The difference
between is measured by the computer and activated with a foot control.

22. Which one of the following examinations may be used as a screen to identify patients
for further investigations?
a) Community Periodontal Index of Treatment Need (CPITN) « CORRECT ANSWER
b) 6-point pocket chart

i
ad
c) Gingival index
d) Plaque index
e) Bleeding index
af
-S
Al

• Comment on this Question


in

The CPITN is designed to be a screen to be completed on every examination. It can not diagnose
ss

periodontal disease. It is used to identify those patients who require further periodontal
investigation.
Ya

23. A 25-year-old man attends your clinic who has not been seen by a dentist for the last
10 years. He is complaining of mobile upper central incisors. These have probing
depths of 9 mm and are bleeding on probing. There is bleeding on probing in all other
sextants but no probing depths above 3mm. Which one of the following is the most
likely diagnosis?
a) Juvenile periodontitis
b) Refractory periodontitis
c) Localized aggressive periodontitis « CORRECT ANSWER
d) Generalized periodontitis
e) Necrotising ulcerative periodontitis

219

Yassin Al-Safadi (Safadi92@hotmail.com)


• Comment on this Question

Under the classification of periodontal disease 1999, juvenile and refractory periodontitis has
been reclassified as either localized or generalized aggressive periodontitis. The presence of 9-
mm pockets around the central incisors indicates that this is not an ulcerative periodontitis.

24. Which one of the following is the correct pressure applied when using the BPE?
a) 5g
b) 10-15g
c) 20-25g « CORRECT ANSWER
d) 30g
e) 35g

• Comment on this Question

i
ad
The British Society of Periodontology reccommends 20-25g of pressure for BPE.
af
25. Which one of the following statements is incorrect regarding surgical regenerative
-S
treatment of peri-implantitis lesions?
a) A three-walled defect is more successfully treated than a single walled defect
Al

b) The use of bone regenerative materials without barrier membranes show good clinical
improvements« CORRECT ANSWER
c) Impeccable oral hygiene is a prerequisite for successful treatment
in

d) Should follow a course of non-surgical treatment


ss

e) The use of bone regenerative materials with barrier membranes show good clinical
improvements
Ya

• Comment on this Question

Initial treatment of a peri-implantitis lesion should involve oral hygiene instruction and non
surgical techniques. This can then be supplemented by surgical regenerative treatment. A three
walled defect will be the most predictable to treat and a barrier membrane is essential in
excluding soft tissue invasion to allow for bone regeneration.

26. Regarding the links between periodontitis and risk for artherosclerosis, which of the
following is untrue?
a) Periodontitis has been associated with an increase in cardiovascular events
b) Periodontitis causes only localized inflammation « CORRECT ANSWER

220

Yassin Al-Safadi (Safadi92@hotmail.com)


c) Endothelial dysfunction is associated with atherosclerosis
d) Intensive periodontal therapy results in improvement of endothelial dysfunction
e) Periodontitis and atherosclerosis share the greatest risk factor of smoking

• Comment on this Question

Evidence to date is consistent with the notion that severe generalized periodontitis causes
systemic inflammation and endothelial dysfunction. Periodontitis has effects that reach beyond
the oral cavity and its treatment and prevention may contribute to the prevention of
atherosclerosis.

27. The abbreviation CPITN stands for which one of the following?
a) Community Periodontal Index of Treatment Need « CORRECT ANSWER
b) Complex Periodontal Index of Treatment Need

i
ad
c) Community Periodontal Index of Tissue Necrosis
d) Complex Plaque Index of Tooth Necrosis
e) Complex Plaque Indicator of Treatment Need
af
28. Which one of the following inter-dental aids is most effective at removing inter-dental
-S
plaque?
a) Dental floss
Al

b) Electric toothbrushes
c) Inter-dental brushes « CORRECT ANSWER
in

d) Triangular toothpicks
ss

e) Dental tape
Ya

• Comment on this Question

A recent systematic review by Clot et al. 2008 reported that inter-dental brushes are able to
remove more dental plaque than dental floss or wood-sticks.

29. Which one of the following associated with dentifrices does not offer advantages in
controlling gingivitis?
a) Amine fluoride/stannous fluoride
b) Chlorhexidine
c) Fluoride « CORRECT ANSWER
d) Stannous fluoride
e) Triclosan

221

Yassin Al-Safadi (Safadi92@hotmail.com)


• Comment on this Question

Dentifrices containing amine fluoride/stannous fluoride, chlorhexidine, stannous fluoride and


Triclosan have been shown to be more effective in controlling gingivitis.

30. Which one of the following is not associated with an increased risk of periodontitis?
a) Cohen’s syndrome
b) Down’s syndrome
c) Epilepsy « CORRECT ANSWER
d) Glycogen-storage disease
e) Hypophosphatasia

i
ad
• Comment on this Question af
All the other conditions are named as systemic diseases associated with periodontitis in the
-S
1999 classification. Certain epileptic drugs can lead to gingival hyperplasia but not associated
with periodontitis in patients with good oral hygiene.
Al

31. Which of the following does not have an increased risk of severe periodontal disease?
in

a) A 21-year-old patient with evidence of erosion on the palatal surfaces of her upper
incisor teeth « CORRECT ANSWER
ss

b) A patient who smokes 10 or more cigarettes a day


c) A patient with a high percentage of bleeding on probing in relation to a low plaque
Ya

index
d) BPE scores of 3, 4 or * in patients under 35 years of age
e) Diabetic patient

• Comment on this Question

There is no association between tooth surface loss (tooth wear) and periodontal disease, so a
21-year-old patient with evidence of erosion on the palatal surfaces of her upper incisor teeth is
not at higher risk of developing severe periodontal disease. All of the other options are
associated with a high risk of periodontal disease.

222

Yassin Al-Safadi (Safadi92@hotmail.com)


• BPE scores of 3, 4 or * in patients under 35 years of age indicate advanced attachment
loss that is not consistent with the young age of these patients.

• Patients who smoke show greater attachment loss than non-smokers. Smokers also
respond less well to periodontal therapy than non-smokers.

• Diabetic patients show greater periodontal attachment loss than non-diabetic patients,
and the attachment loss is related to the severity of their condition and their degree of
glycaemic control.

• The persistence of high bleeding scores in patients with small amounts of plaque also
suggests a high risk of disease progression.

32. Which of the following is consistent with a BPE Code 1?


a) Attachment loss of more than 6 mm
b) Bleeding on probing « CORRECT ANSWER
c) Horizontal tooth mobility of more than 1 mm

i
d) Overhanging crown margin

ad
e) Subgingival calculus
af
-S
Al

• Comment on this Question

A BPE Code 1 identifies areas that exhibit bleeding on gentle probing.


in

Attachment loss of more than 6 mm corresponds to a BPE Code 4, while overhanging crown
ss

margins and subgingival calculus correspond to a BPE Code 2.


Ya

The BPE system does not include any assessment of tooth mobility.

33. With which of the following periodontal risk factors is fremitus associated?
a) Bleeding on probing
b) Furcation involvement
c) Occlusal trauma « CORRECT ANSWER
d) Pocketing
e) Pus formation

• Comment on this Question

223

Yassin Al-Safadi (Safadi92@hotmail.com)


Fremitus is a vibration or mobility felt on palpation of teeth when they close together. As such it
indicates increased occlusal loading or occlusal trauma.

34. Which of the following is a characteristic feature of chronic periodontitis?


a) Generalised pain from the gingivae
b) Gingival bleeding
c) Radiographic evidence of attachment loss « CORRECT ANSWER
d) Red appearance
e) Swollen gingivae

• Comment on this Question

Destructive disease is a feature of periodontitis. All of the other features could be found in
association with chronic gingivitis either alone or coexisting with chronic periodontitis.

i
ad
35. Which of the following is a cause of gingival overgrowth (gingival hyperplasia)?
a) Anaemia af
b) Antidepressant medications
-S
c) Immunosuppressive medications « CORRECT ANSWER
d) Lichen planus
Al

e) Sjogren syndrome
in
ss

• Comment on this Question


Ya

Gingival overgrowth may occur in patients who are taking certain immunosuppressive
medications, such as cyclosporin. Antidepressant medications, anaemia, lichen planus and
Sjogren syndrome are not associated with gingival overgrowth.

36. Furcation involvement is a local risk factor for periodontal disease. Which teeth are
most likely to be affected by furcation involvement?
a) Lower first premolars
b) Upper canines
c) Upper central incisors
d) Upper first molars « CORRECT ANSWER
e) Upper second premolars

224

Yassin Al-Safadi (Safadi92@hotmail.com)


• Comment on this Question

Furcation involvement can only occur in teeth with two or more roots. Of the teeth listed above,
only upper first molars are multi-rooted (they have three roots). All of the other teeth have one
root, and thus no furcation area.

37. Histologically, periodontal lesions have been classified as initial, early, established or
advanced. Which cell type is most common in the early lesion?
a) Eosinophils
b) Macrophages
c) Neutrophils « CORRECT ANSWER
d) Plasma cells (B cells)
e) T cells

i
ad
• Comment on this Question af
-S
The early lesion is still in the early stages of development of gingival/periodontal inflammation
(around 1 week). It is still characterised by cells of the innate immune response, of which
Al

neutrophils are the most common type. Cells of the adaptive immune response (such as B cells)
are more common in established and advanced lesions.
in

38. Which of the following bacteria is most commonly associated with aggressive
periodontitis?
ss

a) Aggregatibacter actinomycetemcomitans « CORRECT ANSWER


Ya

b) Lactobacillus acidophilus
c) Porphyromonas gingivalis
d) Streptococcus anginosus
e) Treponema vincentii

• Comment on this Question

• Treponema vincentii is associated with necrotising ulcerative gingivitis.

• Porphyromonas gingivalis is associated with chronic periodontitis.

• Lactobacillus acidophilus and Streptococcus anginosus are associated with dental caries.

225

Yassin Al-Safadi (Safadi92@hotmail.com)


39. Which of the following is a characteristic feature of necrotising ulcerative gingivitis?
a) Bleeding on probing
b) Furcation involvement
c) Grade I mobility
d) Periodontal pocketing
e) ‘Punched out’ interdental papillae « CORRECT ANSWER

• Comment on this Question

Furcation involvement, grade I mobility and periodontal pocketing are features of chronic
periodontitis, while bleeding on probing can be a feature of chronic gingivitis and chronic
periodontitis.

40. What percentage of sites with bleeding on probing go on to develop periodontal

i
ad
destruction (ie further pocket deepening)?
a) 0%
b) 30% « CORRECT ANSWER
af
c) 50%
-S
d) 70%
e) 100%
Al
in
ss

• Comment on this Question


Ya

Interestingly, studies in the early 1990s demonstrated that only 30% of sites with bleeding on
probing go on to develop periodontal destruction (pocket deepening). In other words, the
absence of bleeding on probing is considered to be a sign of stability, and treatment should be
aimed at achieving this result.

41. In a healthy individual, which of the following structures is attached to enamel at the
base of a healthy gingival crevice?
a) Attached mucosa
b) Dentogingival fibres
c) Junctional epithelium « CORRECT ANSWER
d) Rete pegs
e) Sulcular epithelium

226

Yassin Al-Safadi (Safadi92@hotmail.com)


• Comment on this Question

The attached mucosa is external to the gingival sulcus or crevice. The sulcular epithelium lines
the coronal part of the crevice, but is not attached to the enamel on the tooth. Rete pegs are
found on the inner aspect of the gingival epithelium. Dentogingival fibres attach the gingiva to
the cementum, but are found beneath the epithelium lining the crevice and the epithelial
attachment to the enamel.

42. Which of the following measures is not suitable for monitoring a patient’s response to
periodontal treatment?
a) Bleeding score
b) BPE score « CORRECT ANSWER
c) Mobility
d) Plaque score
e) Probing pocket depth

i
ad
af
-S
• Comment on this Question
Al

Probing pocket depth, bleeding score and mobility can be used to monitor the response of an
individual site or tooth to periodontal treatment. The plaque score can be used to monitor a
patient’s compliance with plaque control (which will have an effect on the underlying
in

periodontal condition). The Basic Periodontal Examination (BPE) is a screening tool rather than a
ss

monitoring tool. As it records the condition of the worst site in a sextant (of four to six teeth), it
does not monitor the responsiveness of individual sites to periodontal treatment.
Ya

When performing a Basic Periodontal Examination (BPE) for a new patient, you record the
following scores:

3/1/3

3/1/3

43. As this is a new patient, you then decide that radiographs are indicated. Which are the
most appropriate radiographs to prescribe for this patient?
a) Bitewings (horizontal or vertical) « CORRECT ANSWER
b) Intraoral periapical radiographs (IOPAs) of upper and lower posterior teeth
c) Lateral cephalogram
d) OPG (with no condyles)
e) Upper anterior occlusal

227

Yassin Al-Safadi (Safadi92@hotmail.com)


• Comment on this Question

BPE scores of 3 indicate pockets that have a depth of 3.5–5.5 mm. OPGs, while showing
generalised trends in bone levels, do not provide accurate undistorted alveolar bone levels for
the purposes of monitoring periodontal bone levels. An upper anterior occlusal is not indicated
in this case, as there is no pocketing around the upper anterior teeth. In any case, due to its
angulation and distortion of bone levels it is not appropriate for monitoring bone levels. A
lateral cephalogram is an orthodontic assessment and does not show periodontal bone levels.
Bitewings (either horizontal or vertical) are preferable to IOPAs of affected teeth as:

• the number of exposures (ie films) is reduced

• due to the angulation of the film relative to the X-ray beam, a more accurate
assessment can be made of the periodontal bone levels relative to the crown of the tooth.

i
ad
44. A 54-year-old man attends for an examination. He is a smoker. Intra-orally calculus
deposits are present. Radiographic examination reveals approximately 70% bone loss
af
affecting his lower molar teeth, and approximately 20% bone loss elsewhere
-S
throughout his dentition. What is his diagnosis?
a) Chronic generalised mild periodontitis, with localised advanced areas around the lower
Al

molars « CORRECT ANSWER


b) Chronic moderate periodontitis, with localised moderate periodontitis around the lower
in

molars
c) Generalised chronic periodontitis, with localised aggressive periodontitis around the
ss

lower molars
d) Localised aggressive periodontitis around the lower molars
Ya

e) Necrotising ulcerative gingivitis

• Comment on this Question

As this patient has calculus and is a smoker, he is more likely to have chronic periodontitis than
aggressive periodontitis. As there is 70% bone loss affecting the lower molars, this is localised
advanced disease (more than 30% of sites are affected).

Necrotising ulcerative gingivitis is an acute painful condition that causes destruction of the
gingival soft tissues.

45. Which of the following is a systemic risk factor for periodontal disease?

228

Yassin Al-Safadi (Safadi92@hotmail.com)


a) Anaemia
b) Diabetes mellitus « CORRECT ANSWER
c) Furcation involvement
d) Overhanging crown margins
e) Subgingival calculus

• Comment on this Question

Overhanging crown margins, subgingival calculus and furcation involvement are all local risk
factors for periodontal disease. There is no demonstrated link between anaemia and
periodontal disease. Diabetes mellitus is a systemic risk factor for periodontal disease and is
thought to affect the host response by altering neutrophil function and causing abnormal
collagen formation.

i
ad
46. A patient presents with gingival recession on the labial surface of a lower central
incisor tooth. Which of the following is not a possible cause?
a) Excessive tooth brushing
af
b) Frequent intake of acidic fruits (eg oranges) « CORRECT ANSWER
-S
c) Localised periodontitis
d) Pronounced frenal attachment (‘high frenum’)
Al

e) Recent orthodontic treatment leading to proclination of the affected tooth


in
ss
Ya

• Comment on this Question

There is no association between the frequent intake of acidic fruits and gingival recession,
although such habits will cause erosion and dentine sensitivity in patients who have already
experienced recession. A pronounced frenal attachment is thought to ‘pull’ on the gingival
tissues during function, causing local ischaemia. Although the evidence for the involvement of a
frenum in gingival recession has been questioned, removal of the frenum is indicated during a
gingival grafting procedure to restore the recessed area. It is also thought that a ‘high’ frenal
attachment makes it difficult for the patient to maintain adequate oral hygiene, and can
therefore predispose them to localised periodontitis, which in turn will cause gingival recession.
Proclination of teeth, particularly lower incisors, which have a thin labial plate, will result in
localised recession. Excessive tooth brushing can also traumatise the area and cause recession.

47. Which of the following statements about the association between diabetes mellitus
and periodontal status is correct?

229

Yassin Al-Safadi (Safadi92@hotmail.com)


a) Diabetic patients who control their condition by diet alone are still likely to develop
advanced periodontal destruction
b) Poorly controlled diabetic patients experience a similar degree of periodontal
destruction to non-diabetic patients
c) Responsiveness to periodontal treatment is not correlated with overall diabetic control
d) There is no difference in neutrophil function between diabetic and non-diabetic patients
e) There is evidence to suggest the existence of a bidirectional association between
diabetes mellitus and periodontal disease (ie where glycaemic control can affect
periodontal destruction, and the level of periodontal inflammation can affect glycaemic
control)« CORRECT ANSWER

• Comment on this Question

i
Periodontal destruction is correlated with glycaemic control, and as a result poorly controlled

ad
diabetic patients show increased periodontal destruction compared with non-diabetic patients.
Responsiveness to periodontal treatment is correlated with overall diabetic control, with poorly
af
controlled diabetic patients not responding as well to periodontal treatment as well-controlled
-S
or non-diabetic patients. The damage caused by diabetes mellitus appears to be related to
altered neutrophil function and altered collagen formation in diabetic patients. The evidence
Al

suggests that there is a bidirectional association between diabetes mellitus and periodontal
disease. Thus poorly controlled diabetes can cause increased susceptibility to periodontal
disease, and poorly controlled periodontal disease can cause deterioration in glycaemic/diabetic
in

control. Diabetic patients who control their condition by diet alone show better glycaemic
ss

control, and consequently do not experience advanced periodontal destruction due to their
condition.
Ya

48. Periodontal diseases are classified as localised or generalised. What is the threshold at
which a patient is diagnosed with generalised rather than localised disease, in terms
of the percentage of sites probed that have more than 4 mm of attachment loss?
a) 15%
b) 20%
c) 25%
d) 30% « CORRECT ANSWER
e) 35%

• Comment on this Question

230

Yassin Al-Safadi (Safadi92@hotmail.com)


The threshold for changing a diagnosis of periodontal disease from localised to generalised
occurs when more than 30% of sites exhibit pocketing of more than 4 mm.

49. Which of the following is an indication for periodontal surgery?


a) Chronic gingivitis
b) Fremitus
c) Grade II mobility
d) Pockets of less than 5 mm with bleeding on probing
e) Prior to crowning of teeth with short clinical crowns « CORRECT ANSWER

• Comment on this Question

Periodontal surgery is indicated for teeth with short clinical crowns (eg due to tooth wear), to
increase the length of the clinical crowns. Chronic gingivitis should be treated with oral hygiene

i
ad
instructions and prophylaxis. Fremitus (occlusal trauma) should be treated by occlusal
adjustment. Pockets of less than 5 mm with bleeding on probing should be managed with
af
subgingival scaling. Teeth with grade II mobility have a limited prognosis, and normally surgery
should not be attempted on them.
-S
50. Following non-surgical management, which of the following antibiotics is indicated for
Al

direct application to sites affected by localised aggressive periodontitis?


a) Amoxicillin
in

b) Cefuroxime
c) Erythromycin
ss

d) Metronidazole
e) Tetracycline « CORRECT ANSWER
Ya

• Comment on this Question

Direct application of tetracycline is indicated for the treatment of sites affected by localised
aggressive periodontitis. Amoxicillin and metronidazole may be used to treat generalised
aggressive periodontitis or chronic advanced periodontitis. Antibiotics should only be used
following, or in association with, non-surgical management.

51. What is a secondary local factor of periodontal disease? Select one option only.
a) An Every denture
b) Fractured central incisor

231

Yassin Al-Safadi (Safadi92@hotmail.com)


c) Hawley retainer « CORRECT ANSWER
d) HIV
e) Open contact between UR6 and UR7

• Comment on this Question

An Every denture is one that is designed to provide minimal periodontal interference, and while
all removable partial dentures can cause plaque to accumulate this minimises plaque
accumulation by having free gingival margins. A fractured central incisor would not in itself
cause a plaque trap around the gingival margin. A Hawley retainer and all orthodontic
appliances often cause plaque traps as they sit around the gingival tissues and can make it
difficult to clean them. HIV is a factor in periodontal disease, but it is a systemic factor not a
local factor. An open contact between teeth that enables cleaning without food packing should

i
not increase the risk of periodontal disease.

ad
52. Which one of the following is classed as a code 2 on the Silness and Loe plaque index?
af
a) Film of plaque visible only by removal on probe or by disclosing
b) Heavy accumulation of soft material filling the niche between the gingival margin and
-S
the tooth surface; the interdental region is filled with debris
c) Moderate accumulation of plaque that can be seen with the naked eye « CORRECT
Al

ANSWER
d) Soft debris covering more than one-third but not more than two-thirds of the tooth
in

surface
ss

e) Soft debris covering not more than one-third of the tooth surface
Ya

• Comment on this Question

The Silness and Loe plaque index was developed in 1964. The index is charted as below:

0: No plaque

1: Film of plaque visible only by removal on probe or by disclosing

2: Moderate accumulation of plaque that can be seen with the naked eye

3: Heavy accumulation of soft material filling the niche between the gingival margin and the
tooth surface; the interdental region is filled with debris

The others are taken from the Greene and Vermillion Oral Hygiene Index.

232

Yassin Al-Safadi (Safadi92@hotmail.com)


53. What treatment is indicated from the following basic periodontal examination (BPE)?

3 1 3

2 2 3

a) Oral hygiene instruction with supragingival scale


b) Oral hygiene instruction, 6 point pocket chart, supra and subgingival scale « CORRECT
ANSWER
c) Supragingival scale
d) Vertical bitewings, oral hygiene instruction, 6 point pocket chart
e) 6 point pocket chart, supra and subgingival scale

• Comment on this Question

i
ad
A code 3 on a BPE indicates the presence of a pocket between 3.5 and 5.5 mm. To assess the
extent of the problem, a 6 point pocket chart is indicated to assess the bone loss, whether a
af
vertical or horizontal defect. Oral hygiene instruction is indicated to improve the home care of
-S
the patient. A code 2 on a BPE indicates the presence of calculus that needs to be removed; if
the patient has pockets (indicated by a code 3), this would indicate a subgingival scale. Vertical
Al

bitewings can also be indicated depending on what radiographs are currently available.

54. Which one of the following is the main collagen fibre in the periodontal ligament?
in

a) Collagen I « CORRECT ANSWER


b) Collagen II
ss

c) Collagen III
Ya

d) Collagen IV
e) Collagen V

• Comment on this Question

The periodontal ligament has mostly collagen I, which consists of two α1 chains and one α2
chain. Collagen III is also present and this consists of is three α1 III chains.

55. Which one of the following micro-organisms is most associated with acute necrotising
ulcerative gingivitis?
a) Actinobacillus actinomycetemcomitans
b) Fusobacterium sp. obligate anaerobe « CORRECT ANSWER

233

Yassin Al-Safadi (Safadi92@hotmail.com)


c) Porphyromonas gingivalis
d) Prevotella intermedia
e) Streptococcus mutans

• Comment on this Question

Actinobacillus actinomycetemcomitans is most associated with rapidly progressing periodontal


lesions. Prevotella intermedia is associated with necrotising disease and rapidly progressing
disease, but Fusobacterium sp. is thought to be the principle pathogen in necrotising disease.
Porphyromonas gingivalis is associated with chronic periodontal disease and Streptococcus
mutans is associated with caries.

56. Which one of the following patients is likely to have true pocketing?
a) A 15-year-old boy who has poor oral hygiene

i
ad
b) A 30-year-old diabetic patient who has extensive calculus deposits and multiple
overhanging restorations
c) A 32-year-old woman who is 26 weeks pregnant
af
d) A 45-year-old man who smokes 20 cigarettes a day « CORRECT ANSWER
-S
e) A 52-year-old patient taking nifidipine
Al
in
ss

• Comment on this Question

False pocketing is present where the gingiva is enlarged or there is extensive gingivitis leading to
Ya

readings of pockets without tissue destruction. This can be due to pregnancy, poor oral hygiene
or gingival overgrowth. Gingival overgrowth is present with some medications including
phenytoin, ciclosporin and nifidine. Local factors such as calculus deposits and overhanging
restorations can cause gingival overgrowth and also give false readings. Smoking and diabetes
are both known to be risk factors for periodontal disease.

57. Which one of the following would be the most appropriate treatment of a grade 1
furcation lesion?
a) Extraction
b) Hemisection
c) Non-surgical management « CORRECT ANSWER
d) Root resection
e) Tunnel preparation

234

Yassin Al-Safadi (Safadi92@hotmail.com)


• Comment on this Question

A grade I furcation lesion would be horizontal loss of support that is not more than one-third of
the tooth width. Conservative management would be the first approach and this would be non-
surgical scaling and oral hygiene instruction. Tunnel preparation is where the whole furcation
area is investigated and widened to allow home care; this is not indicated in a grade I lesion as
there is minimal bone loss around the furcation. Root resection and hemisection involve elective
root treatment and then removal of one or more of the roots to allow good cleaning. Root
resection, hemisection and extraction are all aggressive treatments of an early furcation lesion.

58. Which one of the following is associated with periodontal disease in the permanent
dentition?
a) Apert syndrome

i
b) Chediak–Higashi syndrome

ad
c) Gorlin–Goltz syndrome
d) Papillon–Levefre syndrome « CORRECT ANSWER af
e) Ramsay Hunt syndrome
-S
Al

• Comment on this Question


in

Apert syndrome is a developmental deformity with early fusion of the cranial sutures. Chediak–
ss

Higashi syndrome is associated with neutropenia; periodontal disease is associated with it but
Ya

mainly in the deciduous dentition. Gorlin–Goltz syndrome is associated with odontogenic


keratocysts, bifid ribs and calcification of the falz cerebri. Ramsay Hunt syndrome is caused by
herpes zoster causing vesicles on the face and external auditory meatus associated with lower
facial palsy. Papillon–Levefre syndrome is an autosomal recessive disorder. It is characterised by
hyperkeratosis of the palms and soles. Initially there is normal dental development, then onset
of periodontal disease affecting both the deciduous and the permanent dentition.

235

Yassin Al-Safadi (Safadi92@hotmail.com)


[Pharmacology
i
MCQs] ad
af
-S
Al

[With Answers]
in
ss
Ya

236

Yassin Al-Safadi (Safadi92@hotmail.com)


Pharmacology MCQs
1. A 62-year-old diabetic man who was started on diclofenac for post-operative pain
develops abdominal pain and dark stools. On examination, he has melaena. By what
mechanism has this drug caused his symptoms?
a) Type-1 hypersensitivity reaction
b) Cytotoxicity
c) Induction of cyclo-oxygenase pathway
d) Inhibition of PGE2 and PGI2 action of mucosae « CORRECT ANSWER
e) Induction of thromboxane

• Comment on this Question

i
Gastrointestinal bleeding caused by gastric ulceration is a recognised adverse effect of NSAIDS

ad
(ibuprofen, diclofenac). NSAIDS act by inhibiting cyclo-oxygenase, so preventing the production
of classical prostaglandins that have gastro-protective function.
af
2. A 56-year-old woman attends for a routine appointment. She tells you she is
-S
undergoing chemotherapy for metastatic breast carcinoma and has noticed white
patches on her tongue and inside of her mouth for the last 4 days. She does not
Al

complain of a sore throat and is otherwise well in herself. Which would be the best
therapeutic agent?
in

a) Co-amoxiclav
ss

b) Flucloxacillin
c) Fluconazole « CORRECT ANSWER
Ya

d) Caspofungin
e) Aciclovir

• Comment on this Question

Mucosal candidiasis infections are commonly seen in immunosuppressed patients such as those
undergoing chemotherapy. Other diagnoses to consider would include herpes simplex infection,
but this usually presents with very painful vesicles rather than white patches and so aciclovir
would not be indicated. The best antifungal agent would be fluconazole orally, as caspofungin
can only be given intravenously and is reserved for severe invasive fungal infections.
Flucloxacillin is an antistaphylococcal antibiotic and co-amoxiclav is a broader spectrum
antibiotic, neither of which would be suitable for an uncomplicated fungal infection.

237

Yassin Al-Safadi (Safadi92@hotmail.com)


3. The exact pathogenesis of dental caries is complex but which bacteria are commonly
implicated in the disease process?
a) Viridans streptococci « CORRECT ANSWER
b) Staphylococcus aureus
c) Streptococcus pneumoniae
d) Enterococci
e) Group A beta–haemolytic streptococci

• Comment on this Question

Streptococci, staphylococci and enterococci are all Gram-positive cocci. Streptococci are divided
into α- or β-haemolysis. Common α-haemolytic streptococci include Streptococcus viridans and
Streptococcus pneumoniae, although the latter is unlikely to cause dental caries and is more

i
often associated with community acquired pneumonias. Viridans streptococci are groups of

ad
organisms commonly associated with dental caries, in particularStreptococcus mutans has been
implicated in the disease process. Enterococci are similar to streptococci and are principally
af
found in the gastrointestinal tract. Beta-haemolytic streptococci can cause a variety of infections
-S
including tonsillitis (Group A streptococci) and neonatal meningitis (Group B streptococci).

4. A 70-year-old gentleman attends for an emergency appointment complaining of


Al

severe, throbbing toothache. He says he cannot eat on the affected tooth and on
examination, you see a periapical abscess. You decide to drain the pus away but want
in

to cover this gentleman with antibiotics as the surrounding gum is very inflamed. On
ss

questioning, he tells you that when he last had penicillin he has an anaphylactic
reaction and needed intensive care unit (ITU) treatment. Which would be the most
Ya

appropriate antibiotic to use in this case?


a) Co-amoxiclav
b) Metronidazole
c) Amoxicillin and metronidazole
d) Flucloxacillin
e) Clindamycin « CORRECT ANSWER

• Comment on this Question

Anaphylaxis to penicillin administration is thankfully rare, occurring in approximately 1–5 per 10


000 course of penicillin administered. However, people with a history of anaphylaxis should NOT
receive any penicillin-containing antibiotics. Therefore, they cannot receive co-amoxiclav as it

238

Yassin Al-Safadi (Safadi92@hotmail.com)


contains amoxicillin or flucloxacillin, as it could cause a life-threatening reaction. This question
highlights the huge importance of taking a good drug and allergy history from a patient.
Metronidazole would be good cover for the anaerobes that are commonly found in dental
abscesses, but won’t kill mouth organisms such as aerobic streptococci. The best choice here,
therefore, would be clindamycin.

5. As you pass through the waiting room in your dental surgery, you notice a patient is
having difficulty breathing. You bring him into your room and he tells you, in very
short sentences, he suffers from asthma and has forgotten to take his inhaler this
morning. You notice he is breathing very fast, his pulse is racing and his chest is not
moving much. Although he is sitting upright, he appears quite tired and is becoming
increasingly anxious. The nurse knocks on the door to ask why you are late to see your
next patient. Which of the following would be the best next course of action?
a) Talk to the patient and reassure him you think this will be better in a short while
b) Ask the nurse to stay with the patient while you carry on seeing your other patients who
are getting irritated by the delay.

i
c) Leave the patient in your room alone while you discuss with the nurse about what to do

ad
next.
d) Call the patient’s mother and ask her to come and collect him
af
e) Stay with the patient, administer high flow oxygen and ask the nurse to phone for an
-S
ambulance. « CORRECT ANSWER
Al
in

• Comment on this Question


ss

Acute severe or life-threatening asthma is a medical emergency and demands immediate


Ya

treatment. Features of concern include difficulty completing full sentences, high respiratory rate
(particularly >30 resps per min), tachycardia, poorly moving chest wall, use of accessory muscles
and signs of tiredness. This patient should not be left alone, high flow oxygen and
bronchodilators eg nebulised salbutamol should be administered if available and an ambulance
called. Such patients will not improve over time without correct treatment and they can
decompensate very quickly, which is why they should not be left alone. Always enlist the help of
your staff to optimise your management of an acute situation.

6. A 55-year-old obese, Asian gentleman complains of increasingly severe central,


crushing chest pain and shortness of breath while he sits in the waiting room. He tells
you that he had a heart attack 1 year ago and this feels very similar and that he took
his 75 mg aspirin this morning but missed his cholesterol lowering tablet last night.
You suspect this gentleman is having a myocardial infarction (MI) and call an
ambulance. Which of the following medications are known to decrease mortality
during an acute MI?

239

Yassin Al-Safadi (Safadi92@hotmail.com)


a) Aspirin 300 mg « CORRECT ANSWER
b) Aspirin 75 mg
c) Simvastatin 40 mg
d) Warfarin 5 mg
e) Dipyridamole 200 mg

• Comment on this Question

Acute myocardial infarction (MI) is a medical emergency. Administration of 300 mg aspirin has
been shown decrease mortality and this should be given even if the patient has already had a
smaller daily dose of 75 mg. Statin tablets decrease mortality post-MI but are probably not of
benefit in the acute setting. Like aspirin, dipyridamole is an antiplatelet agent but has no
evidence base for use in acute MI. It is sometimes used in the acute management of stroke.

i
Warfarin is an anticoagulant and has no role in this setting.

ad
7. Oral steroids are often used in the treatment of chronic illnesses such as chronic
af
obstructive airways disease and rheumatological conditions. Which of the following
concurrent illnesses can be exacerbated by steroids?
-S
a) Asthma
b) Diabetes « CORRECT ANSWER
Al

c) Psoriasis
d) Inflammatory bowel disease
in

e) Systemic lupus erythaematous


ss
Ya

• Comment on this Question

Steroids are commonly used for conditions including systemic lupus erythaematous (SLE),
inflammatory bowel disease, asthma and psoriasis, so these are unlikely to be exacerbated by
their administration. Steroids are known to antagonise the action of insulin and so can cause
hyperglycaemia in patients not known to have diabetes but also worsen glycaemic control in
patients with known diabetes. Therefore, close monitoring of blood sugars should be
encouraged during steroid administration.

8. You are using lidocaine 2% w/v to provide a nerve block in a dental patient before
giving them a filling. Which of the following best describes the mechanism of
lidocaine’s action?
a) Agonism of the transmembrane sodium receptor

240

Yassin Al-Safadi (Safadi92@hotmail.com)


b) Antagonism of the outward potassium channels of the Na+/K+-ATPase pump
c) Blockade of the sodium ion channel « CORRECT ANSWER
d) Action on N-methyl-D-aspartate (NMDA) receptors and activation of an inward chloride
flux
e) Action on g-aminoobutyric acid (GABA) receptors

• Comment on this Question

Lidocaine diffuses as an uncharged base through the axonal membranes. After combining with
hydrogen ions to form a cationic species, they blockade the sodium receptor.

9. You are suturing a laceration on an 81 kg man under local anaesthesia. You have
lidocaine available without adrenaline. What is the maximum safe dose you can use in
this patient subcutaneously?

i
ad
a) 81 mg lidocaine
b) 162 mg lidocaine
c) 243 mg lidocaine « CORRECT ANSWER
af
d) 324 mg lidocaine
-S
e) 567 mg lidocaine
Al
in
ss

• Comment on this Question

This corresponds to 3 mg/kg, which is the maximum safe dose for lidocaine administration in the
Ya

UK. Answer E corresponds to the 7 mg/kg, which is the maximum safe dose for lidocaine with
adrenaline. Beyond these ranges, excessive plasma concentrations may produce unwanted
cardiac dysrhythmias and cardiovascular collapse.

10. Which of the following is the most accurate definition of the pKa?
a) The concentration required of a weak acid to produce a change in pH of a solution
b) The pH at which 50% of molecules in solution are ionised « CORRECT ANSWER
c) The pH at which 100% of molecules in solution are ionised
d) The concentration required of a weak base to produce a change in pH of a solution
e) A constant that determines how much drug is absorbed across a membrane

241

Yassin Al-Safadi (Safadi92@hotmail.com)


• Comment on this Question

pKa is given by the Henderson–Hasselbalch equation.

For a weak acid:

HA ↔ H+ + A– ; pKa = pH + log [HA]/[A-< /SUP> ].

For a weak base:

BH+ ↔ H+ + B ; pKa = pH + log [HA]/[A-< /SUP> ].

The pKa therefore provides useful information concerning into which compartment (stomach,
plasma or urine) the drug will be maximally absorbed. < /P>

11. Which of the following systems is affected by local anaesthetic toxicity earliest?
a) Cardiovascular
b) Respiratory

i
c) Neurological « CORRECT ANSWER

ad
d) Endocrine
e) Gastrointestinal af
-S
Al

• Comment on this Question


in

Neurological signs are often the earliest sign of local anaesthetic toxicity, followed by cardiac
signs at higher dosages. It is crucial to aspirate to avoid accidental intravenous administration.
ss

12. After prescribing a dose of post-operative antibiotics, you are called to the ward to
Ya

review the patient who is dyspnoeic with audible wheeze, has facial swelling and is
covered in a red rash. Her pulse is 120/min regular and blood pressure is 86/40 mmHg.
Oxygen saturations on 15 l of oxygen are 100%. You quickly diagnose anaphylactic
shock. Which is the most important immediate treatment?
a) 0.5 mg 1:1000 adrenaline given intravenously
b) 0.5 mg 1:1000 adrenaline given intramuscularly « CORRECT ANSWER
c) 10 ml 1:10,000 adrenaline given intravenously
d) 0.5 mg 1:1000 adrenaline given subcutaneously
e) 0.5 mg 1:1000 adrenaline given by sublingual route

• Comment on this Question

242

Yassin Al-Safadi (Safadi92@hotmail.com)


The correct dose and route is 0.5 mg of 1:1000 adrenaline given by intramuscular route. IV is
only recommended by specialist use (critical care/anaesthetics). All the other routes are
incorrect. This is essential knowledge, you must know the immediate management of
anaphylaxis:

• Early recognition of signs (facial swelling (angioedema), wheeze, hypotension,


tachycardia and decreased level of consciousness (late sign)).

• Assessment of airway, breathing and circulation – potentially you might face problems
with all three of these, but airway will be most immediate to require intervention.

• Establish airway, give high flow oxygen (reservoir bag).

• Intravenous access and iv fluids (to compensate for hypotension and tachycardia and
fluid is shifted to physiologically non-useful spaces).

• Chlorphenamine (10 mg iv)

i
ad
• Hydrocortisone (200 mg iv)

• Adrenaline 0.5 mg 1:1000 im (may be most important drug to give quickly depending on
af
the level of compromise from your ABC assessment).
-S
The RESUS council have provided excellent guidelines and information on frequent causes of
anaphylaxis, outcomes and treatment.
Al

http://www.resus.org.uk/pages/reaction.pdf
in

13. A diabetic patient on your operating list for dental extraction under local anaesthesia
ss

complains of feeling sweaty and unwell. He has been mistakenly instructed to fast
overnight and took his evening insulin as usual. You ask the nurse to check the blood
Ya

sugar, it reads as 1.8 mmol. If the patient was to become unconscious, what is the
most appropriate immediate pharmacological option?
a) Hypostop
b) Administer a sugary drink
c) 10 ml of 50% dextrose solution
d) 1 mg glucagon iv
e) 1 mg glucagon im « CORRECT ANSWER

• Comment on this Question

After assessing airway, breathing, circulation, disability and exposure, correcting the blood sugar
is the most immediate treatment in the treatment of hypoglycaemia. Administering a sugary

243

Yassin Al-Safadi (Safadi92@hotmail.com)


drink to an unconscious problem would potentially cause aspiration and create an A and B
problem. The correct IV treatment is 50 ml of 50% dextrose. Glucagon acts by breaking down
glycogen resulting in an increase in the free glucose concentration in plasma. It is administered
by intramuscular route.

14. In the assessment of a patient with severe anaphylactic shock, which part of
assessment is most crucially affected by the administration of intramuscular
adrenaline?
a) Airway « CORRECT ANSWER
b) Breathing
c) Circulation
d) Disability
e) Exposure

i
ad
• Comment on this Question
af
Adrenaline acts on α- and β-adrenergic receptors, the main action here is to provide
bronchodilatation and reduce airway compromise. The a agonism and β-1 agonism will affect
-S
the circulation by increasing blood pressure and cardiac output. While the affect on B and C are
important, it is the affect on A that is lifesaving.
Al

15. During a cardiac arrest, you are asked to administer adrenaline intravenously between
in

the second and third cycle of CPR. What is the correct IV dose?
a) 0.5 mg 1:1000 adrenaline
ss

b) 1 mg 1:10 000 adrenaline « CORRECT ANSWER


c) 0.5 mg 1:10 000 adrenaline
Ya

d) 5 mg 1:10 000 adrenaline


e) 50 mg 1:10 000 adrenaline

• Comment on this Question

This is the correct iv dose for adrenaline in cardiac arrest. The other options are incorrect in
terms of either concentration or dosage.

16. You are taking a medical history from a 55-year-old woman who tells you she is on
ramipril. What class of drug is ramipril?
a) B-blocker

244

Yassin Al-Safadi (Safadi92@hotmail.com)


b) Thiazide diuretic
c) Loop diuretic
d) Angiotensin II blocker
e) Angiotensin-converting enzyme (ACE) inhibitor « CORRECT ANSWER

• Comment on this Question

Ramipril is an angiotensin-converting enzyme (ACE) inhibitor. It acts be interrupting the rennin–


angiotensin–aldosterone mechanism, so reducing blood pressure.

17. Which topically applied antibiotic can be applied to facial wounds to prevent infection
and reduce scarring?
a) Pencillin
b) Erythromycin

i
ad
c) Chloramphenicol « CORRECT ANSWER
d) Metronidazole
e) Fluconazole
af
-S
Al

• Comment on this Question


in

Chloramphenicol is frequently applied to facial wounds to reduce infection and prevent scarring.
ss

18. Sulphonamides are synergistic bacteriostatic agents because in bacteria they:


Ya

a) Interfere with folinic acid production « CORRECT ANSWER


b) Inhibit RNA synthesis
c) Inhibit DNA synthesis
d) Inhibit cell wall synthesis
e) Interfere with folate synthesis

• Comment on this Question

In bacteria, antibacterial sulphonamides act as competitive inhibitors of the enzyme


dihydropteroate synthetase (DHPS).

19. Which one of the following causes a dry mouth?

245

Yassin Al-Safadi (Safadi92@hotmail.com)


a) Adrenaline
b) Atropine « CORRECT ANSWER
c) Amoxicillin
d) Anti-malarials
e) Angiotensin-converting enzyme (ACE) inhibitors

• Comment on this Question

Atropine is an antimuscarinic drug which leads to the decreased salivary outflow and the dry
mouth.

20. Which one of the following drugs should not be given to asthmatic patients?
a) Metronidazole
b) Amoxicillin

i
ad
c) Ibuprofen « CORRECT ANSWER
d) Captopril
e) Paracetamol
af
-S
Al

• Comment on this Question


in

Ibuprofen is contraindicated in asthma as it can cause bronchoconstriction and can precipitate


ss

an asthma attack.
Ya

21. Which one of the following commonly prescribed antibiotics produces a disulfiram-
like reaction on ingestion of alcohol?
a) Amoxicillin
b) Erythromycin
c) Clindamycin
d) Metronidazole « CORRECT ANSWER
e) Chloramphenicol

• Comment on this Question

Metronidazole has the classic disulfiram-like reaction with alcohol. Patients should always be
advised to avoid alcohol when taking this drug.

246

Yassin Al-Safadi (Safadi92@hotmail.com)


22. Which one of the following interacts with warfarin to decrease the patient’s INR?
a) Fluconazole
b) Vitamin K « CORRECT ANSWER
c) Metronidazole
d) Erythromycin
e) Aspirin

• Comment on this Question

Fluconazole, metronidazole and erythromycin potentiate warfarin’s action. Vitamin K interacts


with warfarin but causes a lowering of the INR. Aspirin does not affect the INR as it does not
interfere with the clotting cascade, only the function of the platelets.

23. Which one of the following is a sign or a symptom of lidocaine overdose?

i
ad
a) Light headedness « CORRECT ANSWER
b) Tachycardia
c) Rash
af
d) Hypertension
-S
e) Hyperventilation
Al
in
ss

• Comment on this Question

Signs and symptoms of a lidocaine overdose are: respiratory depression, hypotension,


Ya

bradycardia, confusion, convulsions and light headedness.

24.Which one of the following statements regarding tetracyclines is true?


a) They are narrow spectrum antibiotics
b) They are absorbed better when taken with milk
c) They may be used as a mouthwash in a dose of 45mg dissolved in a little water and held
in the mouth
d) They cause intrinsic staining of the teeth « CORRECT ANSWER
e) They cause extrinsic staining of the teeth

• Comment on this Question

247

Yassin Al-Safadi (Safadi92@hotmail.com)


Tetracyclines should not be given in pregnancy nor in patients who are under 12 years of age. A
250mg tetracycline capsule can be used as a mouthwash to prevent or treat infected oral
ulceration.

25. Which one of the following drugs does not induce gingival hyperplasia?
a) Nifedipine
b) Carbamazepine « CORRECT ANSWER
c) Phenytoin
d) Diltiazem
e) Ciclosporin

• Comment on this Question

Nifedipine and diltiazem are calcium channel blockers, phenytoin is an anti-epileptic and

i
ad
ciclosporin is an immunosuppressant. These all induce gingival hyperplasia.

26. Which one of the following drugs can be prescribed safely in pregnancy?
af
a) Metronidazole
-S
b) Paracetamol « CORRECT ANSWER
c) Prilocaine
Al

d) Miconazole
e) Methotrexate
in
ss
Ya

• Comment on this Question

Methrotrexate can be used to produce an abortion in the early stages of pregnancy. Prilocaine
can induce labour, especially when combined with felypressin

27. Which one of the following statements is correct about lidocaine?


a) A 2.2ml cartridge of 2% lidocaine contains 4.4mg of lidocaine
b) Lidocaine and prilocaine are esters
c) Esters are more likely to cause and allergic reaction than amides « CORRECT ANSWER
d) Amide local anaesthetics are metabolised by the liver
e) Prilocaine is more toxic than lidocaine

248

Yassin Al-Safadi (Safadi92@hotmail.com)


• Comment on this Question

The 2.2ml cartridge contains 44mg of lidocaine. Both lidocaine and prilocaine are amides and
therefore are less likely to cause an allergic reaction. Lidocaine is a much more toxic drug than
prilocaine.

28. Which one of the following statements regarding paracetamol is true?


a) It is excreted unchanged by the kidney
b) The maximum recommended daily dose in an adult is 4 g « CORRECT ANSWER
c) It is not antipyretic
d) It inhibits coughing
e) It should not be given in patients allergic to aspirin

• Comment on this Question

i
ad
Paracetamol is metabolised in the liver, is antipyretic, and does not inhibit coughing.

29. Patients who take warfarin should always:


af
-S
a) Carry a purple warning card
b) Have a therapeutic range of international normalised ratio (INR) between 2 and 3
Al

c) Stop all anticoagulants 3 days prior to tooth extractions


d) Have their blood regularly monitored to measure INR « CORRECT ANSWER
e) Wear a Medic Alert bracelet
in
ss
Ya

• Comment on this Question

The warning card is a yellow card, and they do not need to wear a Medic Alert bracelet. Their
doctor decides the appropriate level of INR, which may be as much as 4.5 for patients who have
had a valve replacement. They may not need to stop their anticoagulants for simple extractions
if their INR is below 3.5 as we should be able to cope with this level of haemorrhage.

30. Against which of the following diseases does a live attenuated vaccine not provide
protection?
a) Hepatitis B « CORRECT ANSWER
b) Measles
c) Mumps
d) Polio
e) Rubella

249

Yassin Al-Safadi (Safadi92@hotmail.com)


• Comment on this Question

Live attenuated vaccines are usually produced from the naturally occurring bacterium or virus,
which can still be infective, but rarely causes serious disease. Viruses are attenuated (weakened)
by growing them repeatedly.

Examples of live attenuated vaccines include the following:

• measles vaccine (as found in the MMR vaccine)

• mumps vaccine (as found in the MMR vaccine)

• rubella (German measles) vaccine (as found in the MMR vaccine)

• oral polio vaccine (OPV)

i
ad
• varicella (chickenpox) vaccine.

31. Which of the following is a toxoid vaccine?


af
a) Diphtheria « CORRECT ANSWER
-S
b) Hepatitis A
Al

c) Hepatitis B
d) Measles
e) Polio
in
ss
Ya

• Comment on this Question

Toxoid vaccines are obtained by treating toxins produced by bacteria or viruses with heat or
chemicals (eg formalin), thereby destroying their ability to cause illness. Although toxoids do not
cause disease, they stimulate the body to produce protective immunity in exactly the same way
as the natural toxins of bacteria or viruses.

Examples of toxoid vaccines include the following:

• diphtheria toxoid vaccine

• tetanus toxoid vaccine.

In both of these cases the toxoid vaccine may be given alone or as one of the components of the
DTP, DTaP or dT vaccines.

250

Yassin Al-Safadi (Safadi92@hotmail.com)


32. What type of drug is ramipril?
a) ACE inhibitor « CORRECT ANSWER
b) Antibiotic
c) Beta blocker
d) Calcium-channel blocker
e) Proton pump inhibitor

• Comment on this Question

Ramipril is an ACE inhibitor. Its uses include the following:

• treatment of high blood pressure

• treatment of heart failure

i
ad
• prevention of heart attack and stroke in people who are at high risk

• af
treatment of kidney problems associated with diabetes.
-S
Ramipril works by causing blood vessels to dilate, lowering the blood pressure and increasing
the flow of blood, and therefore oxygen, to the heart.
Al

33. Which of the following is not a major side-effect of ACE inhibitors?


a) Dizziness
in

b) Dry tickly cough


ss

c) Headache
d) Nausea
Ya

e) Osteoporosis « CORRECT ANSWER

• Comment on this Question

Common side-effects of ACE inhibitors, which affect less than 1 in 10 people who take this
medicine, include the following:

• dry, tickly cough

• nausea or vomiting

• dizziness

251

Yassin Al-Safadi (Safadi92@hotmail.com)


• headache

• dry mouth

• constipation

• diarrhoea.

34. What is the correct dose of aspirin for prophylaxis of cardiovascular events?
a) 7.5 mg
b) 75 mg « CORRECT ANSWER
c) 150 mg
d) 300 mg
e) 750 mg

i
ad
• Comment on this Question
af
Aspirin has two distinct uses in the prevention of cardiovascular events, namely primary
prevention and secondary prevention. Primary prevention is aimed at decreasing the incidence
-S
of stroke and heart attacks in the general population of individuals who do not have a history of
heart or vascular problems. Secondary prevention is aimed at the prevention of cardiovascular
Al

events in patients who are known to have cardiovascular disease.


in

35. What type of drug is salbutamol?


a) Alpha-1 agonist
ss

b) Beta-1 agonist
c) Beta-1 antagonist
Ya

d) Beta-2 agonist « CORRECT ANSWER


e) Beta-2 antagonist

• Comment on this Question

Salbutamol works by acting on receptors in the lungs called beta-2 receptors. When it stimulates
these receptors, it causes the muscles in the airways to relax, which in turn allows the airways to
open.

In conditions that are associated with narrowing of the airways, such as asthma and chronic
obstructive pulmonary disease (eg emphysema and chronic bronchitis), it is difficult for air to

252

Yassin Al-Safadi (Safadi92@hotmail.com)


enter and leave the lungs. As a result of its action in opening the airways, salbutamol makes it
easier to breathe.

Salbutamol is the drug most commonly taken via an inhaler device. Inhalation of the drug allows
it to act directly in the lungs, at the location where it is needed most. Administering salbutamol
via this route also reduces the risk of side-effects occurring in other parts of the body, as the
amount of drug that is absorbed into the bloodstream through the lungs is lower than if the
drug is given orally.

36. Which additional drug is added to amoxicillin to produce the drug known as co-
amoxiclav?
a) Alendronic acid
b) Clavenulol
c) Clavulanic acid « CORRECT ANSWER
d) Furosemide
e) Salicylic acid

i
ad
af
• Comment on this Question
-S
Amoxicillin is a moderate-spectrum antibiotic that is active against a wide range of Gram-
Al

positive organisms and a limited range of Gram-negative ones. It is usually the first choice of
drug within this class because it is better absorbed than other β-lactam antibiotics following oral
in

administration. Amoxicillin is susceptible to degradation byβ-lactamase-producing bacteria, and


so may be given with clavulanic acid to decrease its susceptibility. The incidence of β-lactamase-
ss

producing resistant organisms, including E. coli, appears to be increasing. Amoxicillin is


sometimes combined with clavulanic acid, a β-lactamase inhibitor, to increase the spectrum of
Ya

action against Gram-negative organisms, and to overcome bacterial antibiotic resistance


mediated through β-lactamase production.

37. What type of drug is alendronic acid?


a) Analgesic
b) Antibiotic
c) Beta blocker
d) Bisphosphonate « CORRECT ANSWER
e) Diuretic

• Comment on this Question

253

Yassin Al-Safadi (Safadi92@hotmail.com)


Alendronic acid is used to treat and prevent osteoporosis (a bone disease that causes bones to
become brittle and fragile, and thus susceptible to breakage and fractures). It does this by
preventing loss of bone mass and helping to rebuild lost bone, thereby reducing the risk of
fractures of the spine and hip.

Alendronic acid can be combined with cholecalciferol (also known as vitamin D3 ), which aids
the absorption of calcium by the bones. The body’s main source of cholecalciferol is exposure to
sunlight, although this vitamin is also present in small amounts in some foods (eg oily fish).

38. Which of the following has broad-spectrum activity against both Gram-positive and
Gram-negative bacteria?
a) Amoxicillin « CORRECT ANSWER
b) Gentamicin
c) Indomethacin
d) Metronidazole
e) Vancomycin

i
ad
af
• Comment on this Question
-S
Simple penicillins are only active against Gram-positive organisms, as they cannot cross the
Al

Gram-negative lipopolysaccharide outer membrane. The addition of an amino group makes the
penicillin molecule more hydrophilic, allowing it to cross the lipopolysaccharide layer of Gram-
in

negative bacteria. Amoxicillin, which has a free amino group, is therefore active against both
Gram-positive and Gram-negative bacteria.
ss

39. Which of the following drugs is not required in the treatment of anaphylaxis?
Ya

a) Adrenaline
b) Chlorpheniramine
c) Intravenous fluids
d) Oxygen
e) Prednisolone « CORRECT ANSWER

• Comment on this Question

Prednisolone is an orally administered drug. The steroid that should be given is hydrocortisone,
which is administered intravenously.

40. Against which of the following bacteria is the drug metronidazole active?

254

Yassin Al-Safadi (Safadi92@hotmail.com)


a) Clostridium difficile « CORRECT ANSWER
b) Haemophilus influenzae
c) Staphylococcus aureus
d) Streptococcus pneumoniae
e) Streptococcus pyogenes

• Comment on this Question

Metronidazole, which is taken up by diffusion, is selectively absorbed by anaerobic bacteria and


sensitive protozoa.

41. Which of the following drugs does not interact with warfarin to alter the INR of a
patient?
a) Aspirin « CORRECT ANSWER

i
ad
b) Erythromycin
c) Fluconazole
d) Metronidazole
af
e) Vitamin K
-S
Al
in

• Comment on this Question


ss

Fluconazole, metronidazole and erythromycin all potentiate the action of warfarin. Vitamin K is
the reversal agent for warfarin. Aspirin does not interfere with warfarin metabolism.
Ya

42. Which clotting factors are inhibited by warfarin?


a) II, III, VII and X
b) II, VII, IX and X « CORRECT ANSWER
c) II, VII, IX and XI
d) III, VII, IX and X
e) VII, IX, X and XI

• Comment on this Question

255

Yassin Al-Safadi (Safadi92@hotmail.com)


Warfarin inhibits the vitamin K-dependent synthesis of biologically active forms of the calcium-
dependent clotting factors II, VII, IX and X, as well as protein C, protein S and protein Z, which
have a regulatory role.

43. What is the maximum dose of lignocaine with adrenaline?


a) 0.7 mg/kg
b) 7 mg/kg « CORRECT ANSWER
c) 70 mg/kg
d) 700 mg/kg
e) 7 g/kg

• Comment on this Question

Drug Maximum dose for infiltration Maximum dose for plexus anaesthesia

i
ad
Lignocaine 4 mg/kg 5 mg/kg

Lignocaine with adrenaline 7 mg/kg


af
7 mg/kg
-S
Bupivacaine 2 mg/kg 2 mg/kg
Al

Bupivacaine with adrenaline 3 mg/kg 3 mg/kg

Prilocaine 6 mg/kg 7 mg/kg


in

Prilocaine with adrenaline/octapressin 8 mg/kg 8 mg/kg


ss

44. Which of the following channels does lignocaine act on?


Ya

a) Calcium
b) Magnesium
c) Phosphate
d) Potassium
e) Sodium « CORRECT ANSWER

• Comment on this Question

Lignocaine blocks the fast voltage-gated sodium channels and thus alters signal conduction in
nerves. If enough channels are blocked, the membrane of the postsynaptic neuron will not
depolarise, and so cannot transmit an action potential. This is what causes the anaesthetic

256

Yassin Al-Safadi (Safadi92@hotmail.com)


effect. Careful titration of the drug means that a high degree of selectivity can be achieved with
regard to the blockage of sensory neurons. At higher concentrations, other modalities of neuron
signalling will also be affected.

45. Which of the following drugs is the reversal agent for midazolam?
a) Flamazine
b) Fludrocortisone
c) Flumazenil « CORRECT ANSWER
d) Fluoroethane
e) Fluphenazine

• Comment on this Question

Flumazenil is of benefit if a patient becomes excessively drowsy after they have been given

i
ad
benzodiazepines for a diagnostic or treatment procedure. It has also been used as an antidote in
cases of benzodiazepine overdose.

46.
af
Which of the following statements about paracetamol is true?
-S
a) It acts locally
b) It has no anti-inflammatory properties
Al

c) It is anti-pyretic « CORRECT ANSWER


d) It is nephrotoxic in overdose
in

e) It is usually given in doses of 1.5 g


ss
Ya

• Comment on this Question

Paracetamol is a centrally acting drug that has both anti-pyretic and anti-inflammatory
properties. It is hepatotoxic in overdose, and is normally given in doses of 500–1000 mg four
times daily.

47. For which of the following medical conditions is the combined oral contraceptive pill a
risk factor?
a) Bowel cancer
b) Deep vein thrombosis « CORRECT ANSWER
c) Hypotension
d) Leukaemia
e) Non-Hodgkin’s lymphoma

257

Yassin Al-Safadi (Safadi92@hotmail.com)


• Comment on this Question

The combined oral contraceptive pill is a risk factor for deep vein thrombosis (DVT), and when a
patient is suspected of having a DVT, it is very important to establish this link as quickly as
possible.

48. Which of the following drugs causes onset of wheezing in a patient who is asthmatic?
a) Codeine
b) Diamorphine
c) Ibuprofen « CORRECT ANSWER
d) Morphine
e) Paracetamol

i
ad
• Comment on this Question
af
-S
The main contraindications for the use of ibuprofen are renal disease and asthma.
Al

49. What is the reversal agent for a paracetamol overdose?


a) Alkaline diuresis
in

b) Atropine
c) Methionine
ss

d) N-acetylcysteine « CORRECT ANSWER


e) Naloxone
Ya

• Comment on this Question

Intravenous N-acetylcysteine is indicated for the treatment of paracetamol overdose.

50. A complication of corticosteroid therapy is:


a) Abdominal striae
b) Hirsuitism « CORRECT ANSWER
c) Increased sweating
d) Muscle hypertrophy
e) Weight loss

258

Yassin Al-Safadi (Safadi92@hotmail.com)


• Comment on this Question

A complication of corticosteroid therapy is hirsuitism. This is due to an increase in circulating


androgens

51. A patient collapses in the dental practice surgery waiting room. They have been fitting
for 5 minutes, what is your next course of action?
a) Buccal midazolam « CORRECT ANSWER
b) Chest compressions
c) Intramuscular midazolam
d) Rectal diazepam
e) Two rescue breaths

i
ad
• Comment on this Question
af
-S
The UK Resuscitation Council guidelines for dental practice state the buccal midazolam is the
drug and delivery of choice for persistent fitting, after a rapid ABC assessment is carried out.
Al

52. You are considering your choice of local anaesthesia for a surgical procedure that you
in

anticipate to be difficult. Which one of the following agents provides the most
prolonged anaesthesia?
ss

a) Articaine
b) Bupivicaine « CORRECT ANSWER
Ya

c) Lignocaine
d) Mepivicaine
e) Prilocaine

• Comment on this Question

This is a long-acting local anaesthetic and works via blocking the initiation and propagation of
action potentials by blocking the sodium channels.

53. Which one of the following conditions or drug treatments can make a patient more
prone to post-extraction bleeding?
a) Controlled hypertension

259

Yassin Al-Safadi (Safadi92@hotmail.com)


b) Long-term excess alcohol consumption « CORRECT ANSWER
c) Long-term steroids
d) Previous radiotherapy to the jaws
e) Type II diabetic

• Comment on this Question

A bleeding tendency results from depressed synthesis of blood clotting factors and excess
fibrinolysins. PT, INR and APTT are all increased.

54. Which of the following is the most appropriate to check before administering
intravenous sedation?
a) The patient’s blood pressure « CORRECT ANSWER
b) The patient’s body temperature

i
ad
c) The patient’s height
d) The patient’s respiratory rate
e) The patient’s weight
af
-S
Al

• Comment on this Question


in

The patient’s blood pressure should be recorded as part of the pre-operative assessment along
ss

with other vital signs such as heart rate, oxygen saturation and ASA.
Ya

55. Which of the following is most commonly used for intravenous sedation during dental
treatment?
a) Diazepam
b) Flumazenil
c) Lorazepam
d) Midazolam « CORRECT ANSWER
e) Tempazepam

• Comment on this Question

260

Yassin Al-Safadi (Safadi92@hotmail.com)


Midazolam is the most commonly used benzodiazepine in dentistry for sedation. It is a very safe
and effective drug and is given incrementally and titrated to the desired response. Flumazenil is
the reversal, to be used only in an emergency.

56. Which of the following is the most common unwanted effect of non-steroidal anti-
inflammatories (NSAIDs)?
a) Adverse renal effects
b) Analgesic-associated nephropathy
c) Bronchospasm
d) Gastric disturbances « CORRECT ANSWER
e) Skin reactions

• Comment on this Question

i
ad
Adverse gastrointestinal events are the commonest unwanted effects of the NSAIDs and result
mainly from inhibition of COX-1.

57.
af
Beta-blockers are what type of antidysrhythmic drug?
-S
a) Class I
b) Class II « CORRECT ANSWER
Al

c) Class III
d) Class IV
in

e) Class V
ss
Ya

• Comment on this Question

Class V does not exist. Class I drugs block voltage-sensitive sodium channels. Class III drugs
substantially prolong the cardiac action potential, and class IV drugs are calcium antagonists.

261

Yassin Al-Safadi (Safadi92@hotmail.com)


[Radilogy
i
MCQs] ad
af
-S
Al

[With Answers]
in
ss
Ya

262

Yassin Al-Safadi (Safadi92@hotmail.com)


1. Carious lesions are difficult to diagnose and can be influenced by local factors. Which
one of the following statements is false?
a) Clinically carious lesions are larger than they appear radiographically
b) The image of the carious lesion can be affected dramatically depending on which technique
is used
c) The exposure and factors relating to can have a significant affect on the overall contrast and
therefore affecting the appearance of the carious lesion
d) The bucco-lingual extent of the lesion can always be determined with the parallelling
technique « CORRECT ANSWER
e) The presence of existing restorations maye completely overlie the undiagnosed carious
lesion

• Comment on this Question

i
ad
The bucco-lingual extent of the lesion can always be determined with the parallelling technique
af
The buccal-lingual extent of the lesion cannot always be assessed due to the superimposition
and two-dimensional image of plain film radiography.
-S
2. Dental caries are commonly misdiagnosed on radiographic interpretation due to
Al

which one of the following?


a) Full veneer crowns
in

b) Dental plaque or calculus


c) Cervical burnout « CORRECT ANSWER
ss

d) Composite restorations
e) Root caries
Ya

• Comment on this Question

Dental caries diagnosis on radiographs is not always straighforward due to two additional
radiographic shadows: firstly the radiolucent cervical burnout/translucency and the radiopaque
zone beneath amalgam restorations. Cervical burnout is a radiolucent shadow often evident at
the neck of teeth. It is an artifactual phenomenon created by the anatomy of the teeth and the
variable penetration of the X-ray beam.

3. Which saggital relationship described below is not represented via a lateral


cephalogram?

263

Yassin Al-Safadi (Safadi92@hotmail.com)


a) Jaws to the skull base
b) Maxilla to mandible
c) Teeth to each jaw
d) Soft tissues to hard tissues « CORRECT ANSWER
e) Teeth in the maxilla to the teeth in the mandible

• Comment on this Question

The soft tissue relationships are not assessed using the lateral cephalogram. The lateral
cephalogram is used to assess the saggital relationships of the jaws to the skull base, maxilla to
mandible, teeth in to each jaw and teeth in the maxilla to teeth in the mandible.

4. An orthodontic patient requires imageing for treatment planning, which radiograph


listed below is standardised and reproducible for all patients?

i
ad
a) Dental orthopantomogram (OPG)
b) Lateral cephalogram « CORRECT ANSWER
c) Lower standard occlusal
af
d) Upper standard occlusal
-S
e) Lateral skull radiograph
Al
in
ss

• Comment on this Question

Lateral cephalogram, or sometimes referred to as posterior–anterior (PA) jaws. The patient is


Ya

positioned in the forehead-nose posiiton, with the radiographic baseline horizontal and
perpendicular to the film.

5. Having seen the orthopantomogram (OPG), select which one of the following image
types would you request from the list below:

264

Yassin Al-Safadi (Safadi92@hotmail.com)


i
ad
a) Lateral oblique mandible
b) Upper standard occlusal film
c)
d)
Computerised tomography (CT) scan of sinuses af
Upper standard occlusal using parallax technique « CORRECT ANSWER
-S
e) Vertex occlusal
Al
in

• Comment on this Question


ss

Upper standard occlusal will not allow the parallax technique to be employed for the bucco-
Ya

palatal position of the canine. A vertex occlusal has several drawbacks and disadvantages
including the radiation beam being directly in the lens of the eye; the primary X-ray beam may
be in line with the reproductive organs; and there is lack of detail and contrast. An upper
standard occlusal slightly off the centre line will allow the parallax technique to be used.

6. A patient presents to the Emergency Department; the patient has already had the
image below taken. What second image would you request?

265

Yassin Al-Safadi (Safadi92@hotmail.com)


i
ad
a) Lateral cephalogram
b) Posterior–anterior mandible/jaws « CORRECT ANSWER
af
c) Bilateral lateral oblique mandible
-S
d) Occipito-mental (OM)
e) Computerised tomography (CT) facial bones
Al
in
ss

• Comment on this Question


Ya

A second image is always necessary when suspicious of facial fractures in order to create a
second dimension.

7. A patient presents with a swelling over the anterior mandible/lower central incisors.
Which one of the following methods of imaging would be your primary choice?
a) Orthopantomogram (OPG)
b) Posterior–anterior (PA) mandible image
c) Periapical radiograph of lower incisors « CORRECT ANSWER
d) Computerised tomography (CT) scan of mandible
e) Lower occlusal radiograph

• Comment on this Question

266

Yassin Al-Safadi (Safadi92@hotmail.com)


OPG has the disadvantage of the focal trough; the PA mandible image will not show a clear
enough picture of the area needed, as would a lower occlusal; and a CT scan is too much of a
radiation dose before a primary diagnosis has been made.

8. Which one of the following imaging modalities would be of choice for an unerupted
maxillary canine?
a) Vertex occlusal
b) Orthopantomogram (OPG) « CORRECT ANSWER
c) Computerised tomography (CT) scan of the maxilla
d) Upper oblique occlusal
e) Lower oblique occlusal

• Comment on this Question

i
ad
An OPG will give a more specific area of the location of the impacted canine – if it is impacted,
there is always the chance that it is absent. From there, further imageing can be requested to
af
give a precise location of the tooth and any associated pathology.
-S
9. Which soft tissue shadow from the list below cannot be visualised on an
orthopantomogram (OPG)?
Al

a) Soft palate
b) Nasolabial fold
in

c) Nasal cartilage
d) Nasal septum « CORRECT ANSWER
ss

e) Ear lobes
Ya

• Comment on this Question

The nasal septum is the vertical wall that divides the nose into two nasal cavities and is made up
of cartilage in the front and thin bone in the back.

10. Which one of the following statements is not a disadvantage of dental tomography?
a) Soft tissue and air shadows can overlie the required hard tissue structures
b) Ghost or artifactual shadows can overlie the structures in the focal trough
c) Patient movement in the vertical plane distorts that part of the image being produced at
that instant
d) The use of indirect-action film and intensifying screens results in some loss of image quality

267

Yassin Al-Safadi (Safadi92@hotmail.com)


e) Some patients do not conform to the shape of the focal trough and therefore gives a better
image « CORRECT ANSWER

• Comment on this Question

The focal trough in all patients gives a distorted image and therefore is a major disadvantage of
dental tomography.

11. A patient presents to the Emergency Department having being assaulted. As yet he
has not been seen by a doctor in the Emergency Department. His main complaint is of
double vision, altered sensation over his cheek, nausea and a headache. Which one of
the following is the next most appropriate step in his management?
a) Fine cut coronal computerised tomography (CT) orbits
b) Ophthalmological consultation

i
ad
c) Occipito-mental 30° plain film
d) Primary and secondary advanced trauma life support (ATLS) survey « CORRECT ANSWER
e)
af
Computerised tomography (CT) head to exclude intra-cranial injury
-S
Al

• Comment on this Question


in

All patients should have a primary survey according to the ATLS approach ie Airway, Breathing,
ss

and Circulation. Following from that assessment, a secondary survey will pick up non-life-
threatening injuries. This ABC approach applies to all patients irrespective of their
Ya

complaint/mechanism of injury. Of direct relevance here is nausea and a headache, which are
cardinal signs of a head injury.

12. A 56-year-old man has been diagnosed with a T2 N0 squamous cell carcinoma of the
right lateral tongue. As the maxillofacial SHO you are asked to complete the patients
imaging work up. He has already had computerised tomography (CT) and MRI of his
head and neck to stage his disease. Which one of the following would you request
next?
a) Angiogram of the head and neck
b) Chest X-ray « CORRECT ANSWER
c) Fully body positron emission tomography (PET) scan
d) Angiogram of the legs
e) Ultrasound scan of the carotid arteries

268

Yassin Al-Safadi (Safadi92@hotmail.com)


• Comment on this Question

Once the primary imageing of the lesion has been carried out, patients commonly undergo
imageing of the chest for metastatic tumours. CT scan of head, neck and chest, and sometimes
abdomen, are commonly carried out thereby eliminating the need for a chest X-ray. A full body
PET scan is sometimes carried out to detect other lesions, Angiograms are requested of the legs
when composite flaps eg fibula are to be harvested. Carotid artery ultrasound would be
requested with other complaints regarding the head and neck, it is unusual in oral cancer to
require this procedure, unless from a medical point of view.

13. A patient presents with a submandibular swelling, drooling saliva, elevated tongue,
pyrexia and tachycardia. Which one of the following would be your primary
management?

i
a) Computerised tomography (CT) scan of neck

ad
b) Orthopantomogram (OPG)
c) Ultrasound scan af
d) ABC assessment and contact senior « CORRECT ANSWER
-S
e) Commence broad spectrum iv antibiotics
Al
in

• Comment on this Question


ss

Dento-facial abscesses are common and can be serious. Patients with systemic symptoms (ie
Ya

pyrexia and tachycardia) need hospital admission and intense management. Those with
significant local symptoms that are cause of concern for airway management need urgent
management – which usually is not an imageing modality. Commonly these patients will have
already had an OPG and it can be seen where the offending tooth is. However, basic
resuscitation and discussion with seniors is important. It is not appropriate for these patients to
be sent to the radiology department for imageing, especially not one such as a CT scan for which
they need to lie down – thereby potentially causing an airway obstruction.

14. Which one of the following is the hallmark radiographically of an initial low grade
chronic inflammation?
a) Area of bone loss at the apex of the tooth
b) Widening of the periodontal ligament
c) Dense sclerotic bone evident around the tooth apex (sclerosing osteitis) « CORRECT
ANSWER
d) Well defined/circumscribed radiolucent area of bone at the apex

269

Yassin Al-Safadi (Safadi92@hotmail.com)


e) No apparent changes evident

• Comment on this Question

There is usually no apparent bone loss, but there is dense sclerotic bone around the tooth apex,
otherwise known as sclerosing osteitis.

15. Which one of the following is not a complication of a fractured mandible?


a) Trismus
b) Paraesthesia of the inferior alveolar nerve
c) Paraesthesia of the infra-orbital nerve « CORRECT ANSWER
d) Paraesthesia of the mental nerve
e) Malocclusion

i
ad
• Comment on this Question
af
-S
The infra-orbital nerve exits the skull through the infra-orbital foramen and is more commonly
Al

associated with a fractured zygoma, Le Fort or orbital fractures. All of the other answers are
commonly associated with a fractured mandible.
in

16. Which one of the following is the correct diagnosis likely to be for the patient who
ss

presents with the following radiograph?


Ya

a) Dentigerous cysts

270

Yassin Al-Safadi (Safadi92@hotmail.com)


b) Odontogenic keratocyst « CORRECT ANSWER
c) Radicular cyst
d) Ameloblastoma
e) Squamous cell carcinoma

• Comment on this Question

Keratocysts are commonly oval, extending along the body of the mandible with little
mediolateral expansion. The outline is usually smooth and well defined, sometimes corticated.

17. Which one of the following lesions below is not a multi-locular lesion?
a) Ameloblastoma
b) Central giant cell granuloma
c) Odontogenic keratocyst

i
ad
d) Stafne’s bone cavity « CORRECT ANSWER
e) Aneurysmal bone cyst
af
-S
Al

• Comment on this Question


in

Stafne’s bone cavity is typically a monolocular lesion.


ss

18. Acute maxillary antral sinusitis is not caused by which one of the following?
a) Upper respiratory tract infection
Ya

b) Oro-antral communication
c) Apical infections of the maxillary posterior teeth
d) Malignant tumour of the antrum « CORRECT ANSWER
e) Displacement of a root into the antral cavity

• Comment on this Question

Most commonly patients present with epistaxis or blockage of one or both of the nares, or even
dental pain in the maxilla. It is rare for patients with a malignant tumour to present with acute
sinusitis. Patients who do present with recurrent epistaxis should be treated with suspicion,
especially if they have high risk factors.

271

Yassin Al-Safadi (Safadi92@hotmail.com)


19. Which one of the following is the most likely complication following an isolated
fractured zygoma?
a) Oro-antral communication
b) Infra-orbital paraesthesia « CORRECT ANSWER
c) Diplopia
d) Subconjunctival haemorrhage
e) Mental nerve paraesthesia

• Comment on this Question

Infra-orbital paraesthesia is the most likely complication following the initial injury, diplopia is
usually present if the orbit is involved as well. Subconjunctival haemorrhage is a possible
complication, but again usually if the orbit is involved. Mental nerve paraesthesia is not

i
associated with an isolated zygoma, more likely to be a mandibular fracture.

ad
20. A patient is involved in an RTC (road traffic collision) and admitted to critical care. He
af
is intubated, ventilated, sedated and has his C-spine fully immobilised. Clinically, he
has gross facial swelling, bilateral racoon eyes, epistaxis and what looks like clear fluid
-S
from his nose, bilateral perforated tympanic membranes. He has an obviously mobile
mandible, although difficult to assess the extent of the injury due to the endotracheal
Al

tube. Which one of the following is the most appropriate imaging request for this
patient at this stage?
in

a) Occipito-mental views (OM)


ss

b) Dental tomogram (OPG)


c) Computerised tomography (CT) scan of facial bones
Ya

d) CT scan head and facial bones with 3D reformatting « CORRECT ANSWER


e) Posterior-anterior mandible (PA)

• Comment on this Question

This patient clearly has panfacial fractures that are reflected in the clinical signs. Racoon eyes
are classic sign of a skull fracture; clear fluid from the nose is significant with regards to a
cerebrospinal fluid (CSF) leak and, therefore, intra-cranial trauma; perforated tympanic
membranes may reflect anterior cranial fossa fracture/TMJ or condyle fractures; and he clearly
has a mandible fracture. As he is suspected to have panfacial fractures associated with a cranial
element, it would be most appropriate to scan his head and facial bones at the same time. 3D
reconstructions enable a better perspective for surgical planning and imageing for surgery.

272

Yassin Al-Safadi (Safadi92@hotmail.com)


21. Which one of the following statements is false with regard to cervical burnout?
a) When exposure factors are increased cervical burnout is more obvious
b) Cervical burnout is more difficult to diagnose if there is a restoration present
c) Cervical burnout is more difficult to diagnose if the parallelling technique is used « CORRECT
ANSWER
d) Cervical burnout is usually triangular in shape, which becomes less apparent towards the
centre of the tooth
e) Cervical burnout is difficult to diagnose when using the bitewing radiograph

• Comment on this Question

Cervical burnout is easier to diagnose using the parallelling technique due to the nature of the
angle of the cone.

i
ad
22. Which one of the following statements regarding cephalometrical planes and angles is
not a definition by the British Standards Glossary of Dental terms (BS4492, 1983)?
a)
af
SNB relates the anteroposterior position of the maxilla, as represented by the B point, to
the cranial base« CORRECT ANSWER
-S
b) SNB relates the anterioposterior position of the mandible, as represented by the B point, to
the cranial base
Al

c) The maxillary plane is a transverse plane through the skull represented by joining of the
anterior and posterior nasal spines
in

d) The SN plane is a transverse plane through the skull represented by the line joining sella
ss

and nasion
e) SNA relates the anteroposterior position of the maxilla, as represented by the A point to the
Ya

cranial base
23. A patient presents to the Emergency Department following a road traffic collision
(RTC) and you are called as the maxillofacial senior house officer on call to assess their
facial injuries. They complain of tenderness over the cheek bones, neck pain and
difficult biting their teeth together. As yet they have not been seen by the Emergency
Department SHO. Which radiograph is not indicated in this scenario?
a) Posterior–anterior (P) mandible
b) Orthopantomogram (OPG)
c) Submento-vertex (SMV) « CORRECT ANSWER
d) Occipito-mental view 10°
e) Occipito-mental view 30°

273

Yassin Al-Safadi (Safadi92@hotmail.com)


• Comment on this Question

The head positioning for this image means that it is contraindicated in patients with suspected
neck injuries, especially with an odontoid peg.

24. A 32-year-old man was treated for a fractured mandible and had post-operative
radiographs taken, which one of the following treatments has he received?

i
ad
af
-S
Al

a) Closed reduction and inter-maxillary fixation


in

b) Inter-maxillary fixation and conservative management of the fractures


ss

c) Open reduction and internal fixation


d) Open reduction and internal fixation with temporary inter-maxillary fixation in the form of
Ya

eyelet wires « CORRECT ANSWER


e) No surgical treatment

• Comment on this Question

The plate on the right angle represents an open reduction and internal fixation and the eyelet
wires which can be seen around the teeth in four areas represent temporary inter-maxillary
fixation.

25. Which one of the following is not an indication for taking bitewing radiographs?
a) Assessment of existing restorations
b) Assessment of the periodontal status

274

Yassin Al-Safadi (Safadi92@hotmail.com)


c) Detection of dental caries
d) Monitoring and progression of dental caries
e) Third molar assessment « CORRECT ANSWER

• Comment on this Question

Bitewings radiographs do not give an adequate image of the apex to the inferior alveolar canal.

26. Which one of the following conditions is sometimes associated with hypercementosis?
a) Cimento-osseous dysplasia
b) Giant cell granulomas
c) Hyperparathyroidism
d) Paget’s disease « CORRECT ANSWER
e) Stone’s bone cavity

i
ad
af
-S
• Comment on this Question
Al

Often showing separation and displacement of the teeth with extensive hypercementosis.

27. Which one of the following lesions is regarded by many as a cyst is actually in the
in

WHO classification as an odontogenic tumour?


ss

a) Adenomatoid odontogenic tumour


b) Calcifying odontogenic cyst « CORRECT ANSWER
Ya

c) Keratocyst
d) Odontogenic fibroma
e) Radicular cyst

• Comment on this Question

This is a rare lesion, but still many think of it as a cyst despite the WHO classifiying it as an
odontogenic tumour.

28. Which one of the following conditions is classically described on radiographic


appearance as ‘moth eaten’?
a) Hyperparathyroidism

275

Yassin Al-Safadi (Safadi92@hotmail.com)


b) Osteoarthritis
c) Osteomyelitis « CORRECT ANSWER
d) Osteoradionecrosis
e) Paget’s disease

• Comment on this Question

Classically, the area of bone affect is ragged, patchy or moth eaten with areas of radiolucency.
The outline of the area of destruction is irregular and poorly defined.

29. Which one of the following classical descriptions relates to congenital syphilis?
a) Brown staining of the teeth in incremental lines
b) Discoloured teeth
c) Hyperdontia

i
ad
d) Hypoplastic enamel and altered tooth shape « CORRECT ANSWER
e) Multiple pulp stones
af
-S
Al

• Comment on this Question


in

Brown staining is associated with fluorosis or tetracycline use, pulp stones are associated with
Ehlers–Danlos syndrome and hyperdontia can have various causes. Altered tooth shape is
ss

commonly described as ‘Hutchinson’s incisors’ where the crowns are small, screwdriver or
carre-shaped and often notched. It can also be described as ‘Moon’s or mulberry molars’, which
Ya

are dome shaped or nodular.

30. Which one of the following statements is true of a radicular cyst?


a) Usually found in adults and is the most common of all jaw cysts. « CORRECT ANSWER
b) Usually found in adults over 30 years, but a rare finding.
c) Found in all ages and less than 5% of all odontogenic cysts.
d) Usually found in adolescents, sometimes the elderly. Approximately 20% of all jaw cysts.
e) It is never found in those over 60 years of age.

• Comment on this Question

276

Yassin Al-Safadi (Safadi92@hotmail.com)


Radicular or ‘dental cysts’ are the most common form of jaw cyst. Those found in adults over 30
and are rare are usually lateral periodontal cysts (B), those which are less than 5% of all dental
cysts and of various ages are usually a keratocyst. Those found in adolescents and sometimes
the elderly (~20%) are commonly dentigerous cysts.

31. Which of the following lightens the image on X-ray?


a) Colour of the substance
b) Hot developer
c) Old developer fluids « CORRECT ANSWER
d) Old film
e) The shape of an object

• Comment on this Question

i
ad
Old developer fluids lighten the film after processing. Processing errors that lead to lightening of
the film include:

• underexposure
af
-S
• underdeveloped film (due to cold, dilute or exhausted developer, or insufficient time in
Al

developer).

32. Which metal is commonly used to produce electrons in simple X-ray machines?
in

a) Iron
b) Gold
ss

c) Lead
Ya

d) Silver
e) Tungsten « CORRECT ANSWER

• Comment on this Question

The cathode of the X-ray machine releases electrons into a vacuum, and the electrons hit the
anode target, where their interaction produces X-rays. The X-ray spectrum depends on the
voltage and the anode metal. Medical machines most commonly use tungsten, but copper and
molybdenum have been used in other fields of X-ray production.

33. What is the definition of attenuation?


a) Change in direction of a photon with or without the loss of energy

277

Yassin Al-Safadi (Safadi92@hotmail.com)


b) Deposition of energy
c) Increase in the wavelength of X-rays with modulation of field
d) Reduction in the intensity of the main X-ray due to scatter and absorption « CORRECT
ANSWER
e) Removal of an electron from a neutral atom

• Comment on this Question

Attenuation is the gradual loss of intensity of any kind of flow through a medium.

34. What is the estimated risk per million of developing fatal radiation-induced cancer
from an OPG?
a) 0.2 per million
b) 0.47 per million

i
ad
c) 1.0 per million « CORRECT ANSWER
d) 1.7 per million
e) 2.5 per million
af
-S
Al

• Comment on this Question


in

Somatic stochastic effects are a possible outcome of radiation exposure. However, their
ss

development is random and is a chance effect.


Ya

35. What is the estimated risk per million of developing fatal radiation-induced cancer
from an intra-oral film?
a) 0.2 per million « CORRECT ANSWER
b) 0.47 per million
c) 1.0 per million
d) 1.7 per million
e) 2.5 per million

• Comment on this Question

Somatic stochastic effects are a possible outcome of radiation exposure. However, their
development is random and is a chance effect.

278

Yassin Al-Safadi (Safadi92@hotmail.com)


36. Which metal is found inside a film packet to absorb secondary scatter?
a) Copper
b) Lead « CORRECT ANSWER
c) Silver
d) Titanium
e) Zinc

• Comment on this Question

Lead is the most commonly used shield against X-rays, due to its high density, stopping power,
ease of installation and low cost.

37. Which of the following is found in the fixer solution for X-ray-processing fluid?
a) Aluminium chloride « CORRECT ANSWER

i
ad
b) Ammonium monosulphate
c) Benzotriazole
d) Phenidone
af
e) Potassium carbonate
-S
Al
in

• Comment on this Question


ss

Aluminium chloride is added as a hardener. Ammonium thiosulphate removes desensitised


silver halide crystals.
Ya

38. Which of the following film faults would not result in a light//pale adult film?
a) Developer solution that is too hot « CORRECT ANSWER
b) Developer that is contaminated by fixer
c) Film pack that is back to front
d) On paediatric setting
e) Thick patient tissues

• Comment on this Question

279

Yassin Al-Safadi (Safadi92@hotmail.com)


If the developer fluid is too hot, it accelerates the chemical reaction involved in development of
the film, so the film becomes over-developed and dark. The other options would lead to a pale
film, due to either underexposure or underdevelopment.

39. For which of the following is ultrasound technology used?


a) Ano-rectal cancers
b) Assessment of breast lesions
c) Examination of carotid blood flow
d) Physiotherapy
e) All of the above « CORRECT ANSWER

• Comment on this Question

Ultrasound imaging is used in all of the above medical applications. It has been widely used

i
ad
throughout medicine since it was first pioneered in the 1940s by George Ludwig. Its major
benefits are that it is non-invasive and it has no associated radiation dose. Ultrasound scanning
af
machines are also relatively cheap, small, and easy to use.
-S
40. Which radiological lesion is associated with squamous-cell carcinoma of the lung?
a) Brown’s tumour « CORRECT ANSWER
Al

b) Cherubism
c) Eosinophilic granuloma
in

d) Odontogenic keratocyst
e) Stafne’s bone cavity
ss
Ya

• Comment on this Question

Squamous-cell carcinoma of the lung is associated with the release of parathyroid hormone-like
substance, and Brown’s tumour of hyperparathyroidism would fit this clinically.

41. Who developed a system of lines for assessing the midface on occipito-mental X-rays?
a) Avery
b) Campbell « CORRECT ANSWER
c) Hutchinson
d) Morello
e) Morgani

280

Yassin Al-Safadi (Safadi92@hotmail.com)


• Comment on this Question

In the 1950s, McGrigor and Campbell developed a system based on four axial levels.

42. What is the wavelength of X-rays?


a) 3 × 104 m to 100 μm
b) 100 μm to 700 nm
c) 0.01 nm to 10 nm « CORRECT ANSWER
d) 10 nm to 400 nm
e) 400 nm to 700 nm

• Comment on this Question

i
ad
• The wavelength of visible light is 400 nm to 700 nm.

• af
The wavelength of radio waves and radar is 3 x 104 m to 100 μm.
-S
• The wavelength of infrared light is 100 μm to 700 nm.
Al

• The wavelength of ultraviolet light is 10 nm to 400 nm.

• The wavelength of X-ray and gamma radiation is 0.01 nm to 10 nm.


in

43. Which metal is commonly used as the heat sink for the anode in simple X-ray
ss

machines?
a) Copper « CORRECT ANSWER
Ya

b) Gold
c) Iron
d) Lead
e) Tungsten

• Comment on this Question

When an X-ray is produced in an X-ray tube, the electrons are converted into energy, and 99% of
that energy is in the form of heat. The copper acts as a heat sink by absorbing and conducting
away the heat energy, thus preventing the machine from overheating.

44. What is the definition of ionisation?

281

Yassin Al-Safadi (Safadi92@hotmail.com)


a) Change in direction of a photon with or without the loss of energy
b) Deposition of energy
c) Increase in the wavelength of X-rays with modulation of field
d) Reduction in the intensity of the main X-ray due to scatter and absorption
e) Removal of an electron from a neutral atom « CORRECT ANSWER

• Comment on this Question

The removal of an electron from a neutral atom in the process of ionisation results in the
production of a negative ion (ie the electron) and a positive ion (ie the remaining atom).

45. What is the definition of scatter?


a) Change in direction of a photon with or without the loss of energy « CORRECT ANSWER
b) Deposition of energy

i
ad
c) Increase in the wavelength of X-rays with modulation of field
d) Reduction in the intensity of the main X-ray due to scatter and absorption
e) Removal of an electron from a neutral atom
af
-S
Al

• Comment on this Question


in

• Scatter is the change in direction of a photon with or without the loss of energy.
ss

• Absorption is the deposition of energy removed from the X-ray beam.


Ya

• Attenuation is the main decrease in intensity of an X-ray beam due to scatter and
absorption.

46. What is the average value of natural background radiation per annum in the UK?
a) 320 μSv
b) 550 μSv
c) 2000 μSv « CORRECT ANSWER
d) 4000 μSv
e) 10 000 μSv

• Comment on this Question

282

Yassin Al-Safadi (Safadi92@hotmail.com)


This represents the sum of all naturally occurring sources of radiation, but not man-made
radiation (eg dental X-rays, or radiation produced as a result of nuclear explosions).

47. What is the average value of cosmic background radiation per annum in the UK?
a) 320 μSv « CORRECT ANSWER
b) 550 μSv
c) 2000 μSv
d) 4000 μSv
e) 10 000 μSv

• Comment on this Question

This is the small component of natural background radiation that originates from sources
outside the earth’s atmosphere (mostly from the sun).

i
ad
48. What is the size of a bitewing film?
a) 22 × 22 mm af
b) 22 × 35 mm « CORRECT ANSWER
-S
c) 31 × 41 mm
d) 45 × 28 mm
Al

e) 57 × 76 mm
in
ss

• Comment on this Question


Ya

An upper standard occlusal film is 57 × 76 mm.

A periapical film is 31 x 41 mm.

49. Which of the following is a true radiological sign of orbital floor fracture?
a) Opacity in the antrum
b) Soft tissue swelling in the periorbital region
c) Surgical emphysema
d) ‘Tear-drop’ sign « CORRECT ANSWER
e) Zygomatico-Frontal distraction

283

Yassin Al-Safadi (Safadi92@hotmail.com)


• Comment on this Question

‘Tear-drop’ sign is pathognomonic of a blow-out fracture on occipito-mental 30º view.

50. Which of the following represents one of the four main types of scatter that are seen
at the nucleus of an atom with X-ray radiation?
a) Brownian motion
b) Pair production
c) Piezoelectric effect
d) Photodynamic effect
e) Rayleigh scattering « CORRECT ANSWER

• Comment on this Question

i
ad
The four main types of scatter are:

1. Compton effect af
2. Pair production
-S
3. Rayleigh scattering
Al

4. Photoelectric effect.
in

Brownian motion represents the random movement of particles, especially gas particles that are
suspended in a fluid.
ss

Piezoelectric effect is the mechanism by which ultrasonic scalers work. The piezoelectric effect is
Ya

understood as the linear electromechanical interaction between the mechanical and the
electrical state in crystalline materials.

Please note that while answers B (Pair production) & D (Photodynamic effect) are close, they are
not actually correct terms of scatter.

51. Which of the following is an indication for a cone beam computed tomography
(CBCT)?
a) Assessment of an undisplaced fracture of the mandible
b) Assessment of temporomandibular joint dysfunction
c) Mandibular open-reduction and rigid internal fixation post-operative review
d) Occlusal caries in lower molars
e) Relationship of the inferior alveolar nerve with the roots of an impacted third molar «
CORRECT ANSWER

284

Yassin Al-Safadi (Safadi92@hotmail.com)


• Comment on this Question

CBCT is becoming increasingly used in the analysis of complicated dental diagnoses.


Orthopantomogram would be the first-line assessment for third molars; however if this
indicated close proximity to the inferior alveolar nerve in 2D then a CBCT would be indicated to
further assess the risk of morbidity. Magnetic resonance imaging is more suitable for
temporomandibular joint dysfunction.

52. A patient attends the Emergency Department following an alleged assault and it is
thought that they have sustained hard tissue injuries to the facial skeleton. Which of
the following imaging methods would be most appropriate to screen the mandible?
a) Head computed tomography (CT)
b) Lateral cephalometric radiograph

i
c) Orthopantomogram (OPG)

ad
d) Posteroanterior (PA) mandible and lateral cephalometric radiograph
e) PA mandible and OPG « CORRECT ANSWER af
-S
Al

• Comment on this Question


in

These two radiographs show locations of fractures along the mandible, including the condyles,
displaying size of fracture, amount of displacement and loss in height if fractured condyle is
ss

involved.
Ya

53. A patient attends the Emergency Department following an alleged assault. It is


thought that they have sustained hard tissues injuries to the facial skeleton. Which of
the following imaging methods would be most appropriate to screen the zygoma?
a) Computed tomography (CT) orbit
b) OM 10 and 30 « CORRECT ANSWER
c) Orthopantomogram (OPG) and lateral cephalometric radiograph
d) Posteroanterior mandible and OPG
e) Reverse Towne’s

• Comment on this Question

285

Yassin Al-Safadi (Safadi92@hotmail.com)


Facial views are normally adequate to assess a suspected fracture of the zygoma, including
maxillary sinus, zygoma, arches and ZMC. If the orbital floor is involved then a CT may be
indicated but not as a first-line investigation.

54. Who was the individual responsible for relating the use of X-rays to medicine in 1895?
a) Chappelle
b) Grant
c) Roentgen « CORRECT ANSWER
d) Shaw
e) Williams

• Comment on this Question

Wilhelm Roentgen was a physicist attributed with the discovery of X-rays and their uses in the

i
ad
medical field.

55. Which of the following is encompassed by Campbell’s line 1 when assessing middle
af
third facial fractures?
-S
a) Left lateral wall antrum
b) Right angle mandible
Al

c) Right coronoid process


d) Right supraorbital ridge « CORRECT ANSWER
in

e) Right zygomatic arch


ss
Ya

• Comment on this Question

Campbell’s lines are used as a systematic approach to assessing middle third fractures. There are
four in total. Line 1 follows the contour encompassing the zygomaticofrontal sutures,
supraorbital ridges and frontal sinus.

56. A 55-year-old gentleman has just been diagnosed with T2N1 squamous cell carcinoma
on the lateral border of his tongue. He has a palpable lymph node adjacent to his
sternocleidomastoid. Which one of the following is the most appropriate to further
assess his disease?
a) Full body magnetic resonance imaging (MRI)
b) Head computed tomography (CT)
c) Head and chest CT

286

Yassin Al-Safadi (Safadi92@hotmail.com)


d) Orthopantomogram (OPG)
e) Orthopantomogram (OPG), head and chest CT « CORRECT ANSWER

• Comment on this Question

OPG, head and chest CT is most likely required in this patient. The disease is quite extensive and
the head and chest CT is required to assess size of the tumour, lymphatic involvement and
pulmonary secondary spread. In view of the likelihood of post-operative radiotherapy, a dental
assessment must also be made with the OPG to remove any potentially compromised teeth.

57. An orthopantomogram reveals a multilocular, radiolucent lesion in the right angle of


the mandible, not apparently of odontogenic origin? Which of these is within your
differential diagnoses?
a) Cementoblastoma

i
ad
b) Dentigerous cyst
c) Odontogenic myxoma « CORRECT ANSWER
d) Ossifying fibroma
af
e) Residual cyst
-S
Al
in

• Comment on this Question


ss

Odontogenic myxomas appear most commonly as multilocular radiolucencies with ill-defined


borders. Cementoblastomas stem from cementum, whereas ossifying fibromas are formed from
Ya

the periodontal membrane.

58. When radiographically assessing the extent of caries in a tooth, which of the following
is correct?
a) D1: only one side of the tooth is affected by caries
b) D2: carious lesions extending into the pulp
c) D2: clinically detected caries extending to the cemento-enamel junction (CEJ) «
CORRECT ANSWER
d) D4: all surfaces of the tooth are affected by caries
e) D4: lesions only extending into the dentine

287

Yassin Al-Safadi (Safadi92@hotmail.com)


• Comment on this Question

D1: lesions extending within enamel only; D2: to the CEJ; D3: within cementum but not
incorporating >50%; D4: lesions extend into the pulp.

i
ad
af
-S
Al
in
ss
Ya

288

Yassin Al-Safadi (Safadi92@hotmail.com)


[Restorative
i
MCQs] ad
af
-S
Al

[With Answers]
in
ss
Ya

289

Yassin Al-Safadi (Safadi92@hotmail.com)


1. Which one of the following is the correct definition for CAD CAM?
a) Computer-aided design, computer-aided manufacturing « CORRECT ANSWER
b) Computer-aided design, ceramic added machined
c) Computer-aided design, computer-aided modifications
d) Ceramic advantage design, ceramic advantage milled
e) Ceramic assessed design, computer assessed milled

• Comment on this Question

CAD CAM is short for Computer aided design, computer aided manufacturing. It involves the
milling of a fixed prosthesis.

2. Which one of the following is the definition for a restoration that encompasses the
mesial and occlusal aspects of an upper right first permanent molar?

i
ad
a) Class 1 cavity
b) Class 2 cavity « CORRECT ANSWER
c) Class 3 cavity
af
d) Class 4 cavity
-S
e) Class 5 cavity
Al

3. Which of the following is the correct setting reaction of amalgam?


in

a) Ag3Sn2 + Hg2 → Ag2Sn2 + Ag6Hg3


ss

b) Ag3Sn + Hg2 → Ag2Hg3 + Sn7Hg


c) AgSn + Hg → Ag2Sn + SnHg + AgHg
Ya

d) Ag3Sn + Hg → Ag3Sn + Ag2Hg3 + Sn7Hg « CORRECT ANSWER


e) Ag2Sn3 + Hg → Ag2Sn3Hg

• Comment on this Question

Ag3Sn + Hg → Ag3Sn + Ag2Hg3 + Sn7Hg

4. Under the Kennedy classification, a partial denture that replaces a bi-lateral free end
saddle will be classified as:
a) Class 1 « CORRECT ANSWER
b) Class 2
c) Class 3

290

Yassin Al-Safadi (Safadi92@hotmail.com)


d) Class 4
e) Class 5

• Comment on this Question

• Class 1: bilateral edentulous areas posterior to the remaining teeth.

• Class 2: unilateral edentulous area posterior to the remaining teeth.

• Class 3: a unilateral edentulous area with natural teeth anterior and posterior.

• Class 4: a single but bilateral (crossing the midline) edentulous area anterior to the
remaining natural teeth.

5. The Golden proportion is a ratio that has a long history in mathematics, plus Greek

i
ad
and Egyptian architecture and is now used in planning dental aesthetics. Which one of
the following is the correct description?
a) AB:BC = AC:BC
b) AB:CD = AC:BC
af
-S
c) AB:BC = AC:BC
d) AB:BC = AC:BC = 1.618:1 = 1:0.618
Al

e) AB:BC = AC:AB = 1.618:1 = 1:0.618 « CORRECT ANSWER


in
ss

6. Which one of the following statements regarding conventional preparation of veneers


is false?
Ya

a) May be more effective than minimal preparation in severe tetracycline cases


b) Usually require temporary restorations
c) Can either retain the incisal edge or reduce it by 1 mm
d) Reduction should be within enamel
e) Reduction should be to dentine to allow a better bond « CORRECT ANSWER

• Comment on this Question

The conventional preparation of a veneer will be 0.3 mm cervically, 0.5 mm centrally with the
incisal edge either being retained or reduced by 1 mm. The preparation should stay within the
enamel for increased bonding, as well as keeping the preparation as minimal as possible.

291

Yassin Al-Safadi (Safadi92@hotmail.com)


7. While studying occlusion, which one of the following is correct for the abbreviation
RCP?
a) Retracted condylar position
b) Retruded condylar pathway
c) Retruded condylar position
d) Retruded contact position « CORRECT ANSWER
e) Retracted contact position

• Comment on this Question

The retruded contact position is the occlusal position when the first tooth contact occurs on the
mandibular path of closure with the condyles in the retruded axis position.

8. Which one of the following preparations would be considered to be best practice for a

i
ad
full gold crown?
a) Chamfer finishing line, 30° taper
af
b) Chamfer finishing line, 10° taper « CORRECT ANSWER
c) Shoulder finishing line, 0° taper
-S
d) Shoulder finishing line, 10° taper
e) Shoulder finishing line, 30° taper
Al
in
ss

• Comment on this Question


Ya

A shoulder finish is more appropriate for porcelain crowns as further reduction of tooth tissue is
required for the application of porcelain. The taper should not ideally exceed 20° but will require
a slight taper to ensure adequate seating of the crown.

9. Which one of the following is the correct definition of Bennett's angle?


a) The angle formed by the sagittal plane and the path of advancing condyle during lateral
mandibular movement, as viewed in the horizontal plane« CORRECT ANSWER
b) The angle formed by the transverse plane and the path of advancing condyle during
anterior mandibular movement, as viewed in the horizontal plane
c) The angle formed by the sagittal plane and the path of advancing condyle during
anterior mandibular movement, as viewed in the transverse plane
d) The angle formed by the transverse plane and the path of advancing condyle during
lateral mandibular movement, as viewed in the horizontal plane

292

Yassin Al-Safadi (Safadi92@hotmail.com)


e) The angle formed by the sagittal plane and the path of advancing condyle during lateral
mandibular movement, as viewed in the transverse plane

10. Which one of the following is the correct purpose of a facebow?


a) A facebow registration relates the maxilla to the mandibular cast in its true position
b) A facebow registration relates the maxillary cast to the mandibular cast in centric
occlusion
c) A facebow registration relates the maxillary occlusal plane to a fixed plane on the skull
so that this relationship can be transferred to the articulator« CORRECT ANSWER
d) A facebow registration relates the mandibular occlusal plane to a fixed plane on the
skull so that this relationship can be transferred to the articulator
e) A facebow registration relates the maxillary cast to the mandibular cast to obtain the
true curve of spee

i
ad
11. Which one of the following has the best 10-year survival rate, when replacing a single
upper central incisor? af
a) Cantilever adhesive bridge from the lateral incisor
-S
b) Double-winged adhesive bridge with wings on the lateral incisor and corresponding
central incisor
Al

c) Cantilever conventional bridge, central incisor abutment


d) Fixed/ movable bridge
e) Osseointegrated dental implant « CORRECT ANSWER
in
ss
Ya

• Comment on this Question

Current published data gives a dental implant the greatest success rate, with up to 99% surviving
the ten-year period. A cantilever adhesive bridge from a lateral incisor has a very poor success
rate and double winged adhesive bridges have a tendency to debond on the wing allowing
recurrent caries under the debonded wing. Any conventional bridge will reduce the life
expectancy of the abutment.

12. Overdentures are a useful treatment option. Which one of the following options is not
correct?
a) Overdentures benefit from ridge preservation
b) Overdentures benefit from increase proprioception
c) Overdentures may have increased retention, from the use of precision attachment
d) Requires reduced maintenance of the remaining tooth structure « CORRECT ANSWER

293

Yassin Al-Safadi (Safadi92@hotmail.com)


e) Implant retained lower complete overdentures have a very high success rate

• Comment on this Question

The remaining tooth structure does not require reduced maintenance.

13.Which one of the following statements regarding copy dentures is true?


a) Copy dentures have additional clinical steps
b) Copy dentures require additional adaptation and habituation
c) Copy dentures may be indicated when all previous dentures have not been tolerated
d) Copy dentures allow for the reproduction of satisfactory features of an existing denture
while allowing for the controlled modification of the fitting surface« CORRECT ANSWER
e) Copy dentures do not record centric occlusion

i
ad
• Comment on this Question
af
-S
Copy dentures have a reduced number of clinical steps as they will record the centric jaw
Al

relationship at the same time as secondary impressions. They are particularly useful in patients
who have reduced adaptation but have had successful dentures in the past.
in

14. Which one of the following is classifications is used for an occlusal restoration?
ss

a) Class 1 « CORRECT ANSWER


b) Class 2
Ya

c) Class 3
d) Class 4
e) Class 5

15. Which one of the following is the current guidelines for the time interval between
bitewing radiographs in a patient with a high risk of caries?
a) 1 month
b) 2–3 months
c) 6–12 months « CORRECT ANSWER
d) 2 years
e) 3 years

294

Yassin Al-Safadi (Safadi92@hotmail.com)


16. A patient attends your clinic with a root-filled premolar that has fractured at gum
level. Which one of the following treatment options represents the poorest
restorative prognosis?
a) Post and core, crown « CORRECT ANSWER
b) Crown lengthening, post and core, crown
c) Orthodontic extrusion, post and core, crown
d) Extraction and implant placement
e) Extraction and provision of a denture

• Comment on this Question

In a case of a tooth that has fractured at gingival level, a ferrule effect is required to ensure that
the forces of mastication are transmitted to both the tooth and the post. A tooth without a

i
ferrule of 1.5 mm will offer no form of resistance to transverse forces and the entire load is

ad
taken by the post, with failure inevitable. Orthodontic extrusion or crown lengthening will be
required for a ferrule effect to occur before restoration.
af
17. This patient attends your clinic complaining of the appearance of his root filled upper
-S
right lateral incisor. Which one of the following treatment options is the most
appropriate?
Al
in
ss
Ya

a) Scale and polish


b) All ceramic crown
c) Veneer preparation within dentine

295

Yassin Al-Safadi (Safadi92@hotmail.com)


d) Extraction and conventional bridge
e) Internal and external bleaching « CORRECT ANSWER

• Comment on this Question

The intrinsic staining will not be removed by a scale and polish, and the other options are more
destructive of tooth tissue.

18. Which one of the following areas of a crown preparation for a porcelain bonded crown
should have the most reduction? (Full porcelain occlusal coverage.)
a) Buccal, apical third
b) Buccal, coronal third
c) Distal finishing line
d) Non working cusp

i
ad
e) Working cusp « CORRECT ANSWER

af
-S
• Comment on this Question
Al

The working cusp should be reduced by the greatest amount, typically 2–2.5 mm, in order for
in

adequate strength of metal and porcelain.


ss

19. Which one of the following is the correct definition of Bennett shift?
a) Bennett shift is the movement of the non-working condyle towards the working side «
Ya

CORRECT ANSWER
b) Bennett shift is the movement of the working condyle towards the working side
c) Bennett shift is the movement of the working condyle towards the non-working side
d) Bennett shift is the movement of the non-working condyle towards the non-working
side
e) Bennett shift is the movement of both condyles in anterior movement

20. Which one of the following is a disadvantage to immediate dentures?


a) Gives the patient an aesthetic advantage as can replace a tooth immediately
b) Reduced number of clinical stages, as CJR is normally registered
c) Is designed to fit the ridge immediately after extraction « CORRECT ANSWER

296

Yassin Al-Safadi (Safadi92@hotmail.com)


d) Can protect the socket post extraction
e) Can be a relatively inexpensive way to temporise before implant placement

• Comment on this Question

Following any tooth extraction, the underlying socket will initially be filled with a blood clot
which will then start to remodel. The socket can be initially protected by the immediate
denture, but during the next 6 months of remodelling, the fit of the denture will become poorer.
It may then need to be remade.

21. Which one of the following maybe associated with an unretentive denture?
a) A fully extended denture
b) Soft lining « CORRECT ANSWER
c) Implant retained bars

i
ad
d) Implant retained magnets
e) Precision attachments
af
-S
Al

• Comment on this Question


in

All of the other answers are known to be increase the retentiveness of dentures. There is no
evidence to suggest that a soft lining increases the retentiveness of a denture.
ss

22. Which one of the following combinations produces the lowest stress within a root
Ya

when used as a post and core?


a) Cast metal post and core
b) Composite resin post and core
c) Composite resin core in combination with a glass fibre post « CORRECT ANSWER
d) Parallel prefabricated metal post
e) Tapered prefabricated metal post

• Comment on this Question

Stress analysis of the different post and core restoration methods has suggested that abutment
build-up using composite resin core in combination with a glass fibre post produced the lowest
stress concentration.

297

Yassin Al-Safadi (Safadi92@hotmail.com)


23. Which one of the following is an example of attrition?
a) Tooth surface loss due to a pipe smoking habit
b) Cervical lesions due to aggressive tooth brushing
c) Flattening of the canine in a 'tooth grinder' « CORRECT ANSWER
d) Palatal tooth surface loss in a bulimia patient
e) Pitting of the enamel due to dietary acids

• Comment on this Question

Both A and B are examples of abrasion, as a third party is involved. Options D and E are both
examples of erosion as it is the acids that are causing the tooth surface loss. Attrition will be
tooth surface loss due to tooth-to-tooth contact.

24. Which one of the following has the most minimum preparation?

i
ad
a) Conventional veneer preparation
b) Full gold crown
c) Porcelain jacket crown
af
d) Adhesive bridge « CORRECT ANSWER
-S
e) Conventional bridge
Al
in
ss

• Comment on this Question

Current design of adhesive bridge is for minimal or even no preparation of the tooth. The bridge
Ya

is better designed as a single cantilever with a wrap around design. If the wing is in occlusal
contact, it will be left to Dahl the opposing tooth.

25. Which of the following is the most commonly missing congenitally absent tooth?
a) Lower lateral incisor
b) Lower first premolar
c) Upper central incisor
d) Upper lateral incisor « CORRECT ANSWER
e) Upper first premolar

• Comment on this Question

298

Yassin Al-Safadi (Safadi92@hotmail.com)


The most commonly congenitally absent teeth are the third permanent molars followed by the
maxillary lateral incisors. They are maybe difficult to treatment plan, as the first option will be
whether to open or close the space, as invariably the canine erupts too close to the mid-line. If
the gap is opened then the correct restoration will need to be treatment planned, denture,
bridge or implant.

26. When designing a partial denture, which one of the following steps comes first?
a) Direct retention
b) Minor connectors
c) Support « CORRECT ANSWER
d) Indirect retention
e) Major connectors

i
• Comment on this Question

ad
The correct sequence for designing a partial denture is:

• saddles
af
-S
• support
Al

• pathway of insertion
in

• direct retention
ss

• major connectors
Ya

• minor connectors

• indirect retention

• simplification.

27. Different pigments within different materials may absorb light to differing degrees at
different wavelengths. This factor is why crowns that match perfectly in natural day
light may appear different shades in other lighting conditions. This phenomenon is
known by which one of the following?
a) Colour vision
b) Light vision
c) Metamerism « CORRECT ANSWER
d) Prometamerism
e) Visual acuity

299

Yassin Al-Safadi (Safadi92@hotmail.com)


28. Which one of the following conditions is most likely for you to choose an inlay rather
than an onlay
a) Group function
b) Height:width > 1:1
c) Restoration size less than 1/3 buccal-lingual width « CORRECT ANSWER
d) Parafuctional habits
e) Severely undermined cusps

• Comment on this Question

The other answers would contraindicate the use of an inlay. It is more prudent to provide some
cuspal support that an onlay provides.

29. Which one of the following will be first in the design of a partial denture?

i
ad
a) Direct retention
b) Indirect retention
c) Major connectors
af
d) Pathway of insertion « CORRECT ANSWER
-S
e) Support
Al
in
ss

• Comment on this Question

The design of a partial denture has come from Davenport et al.


Ya

The patient shown in the photograph below requires a lower removable partial denture.

30. The patient shown in the photograph below requires a lower removable partial
denture.

300

Yassin Al-Safadi (Safadi92@hotmail.com)


Assuming that the distance between the gingival margin on the lingual surface of the
lower anterior teeth and the base of the lingual sulcus is 7 mm, which of the following
is the most appropriate major connector to prescribe in this case?

a) Continuous plate
b) Dental bar

i
ad
c) Lingual bar « CORRECT ANSWER
d) Lingual plate
e) Sublingual bar af
-S
Al

• Comment on this Question


in

A lingual bar is the most appropriate major connector because it meets the criteria of being rigid
ss

and hygienic in nature. This requires a distance of 7 mm between the gingival margins of the
adjacent teeth and the functional depth of the sulcus (3.5 mm height of lingual bar for rigidity,
Ya

and 3 mm of clearance for hygienic reasons).

A lingual plate is the next best option, and can be used where the distance between the gingival
margins of the adjacent teeth and the functional depth of the sulcus is less than 7 mm.
However, as this connector covers the gingival margins and lingual surfaces of the adjacent
teeth, it can be plaque retentive, and is therefore associated with a risk of periodontal disease.

The other options (ie sublingual bar, dental bar and continuous plate) are no longer used in
contemporary practice, as they are associated with tolerance problems.< /P>

31. Composite resin and amalgam are often used for restoring posterior teeth. Which of
the following scenarios is better suited for amalgam than for composite resin?
a) Aesthetically visible region
b) Deep cavity
c) Replacement restoration

301

Yassin Al-Safadi (Safadi92@hotmail.com)


d) Small occlusal cavity
e) Subgingival cavity margins « CORRECT ANSWER

• Comment on this Question

Amalgam materials are more appropriate when a cavity margin extends subgingivally, as they
are less moisture sensitive than composite resin. Composite resins are more suitable for small
occlusal cavities (as they are more minimally invasive) and are indicated for aesthetic regions. A
deep cavity and a replacement restoration can be provided in either composite resin or
amalgam.

32. Which of the following reactions describes the setting mechanism of resin-composite
materials?
a) Acid–base reaction

i
ad
b) Free radical polymerisation « CORRECT ANSWER
c) Ionic bond formation
d) Sol–gel transition
af
e) None of the above
-S
Al
in

• Comment on this Question


ss

Resin-composite materials set as a result of a polymerisation reaction (ie the linking of smaller
monomer units to form a growing polymer chain). This reaction is initiated by free radicals,
Ya

which are formed when camphorquinone (the activator molecule within most resin-composite
materials) is exposed to incident light of wavelength 470 nm. A sol–gel transition is the setting
reaction within hydrocolloid impression materials, with the reaction being irreversible in
alginate, but reversible in agar. Acid–base reactions occur in the setting reaction of certain
cements (eg glass ionomer cements). Ionic bond formation occurs during the formation of
certain alloys.

33. What is the predominant metal in a typical dental amalgam?


a) Copper
b) Mercury
c) Silver « CORRECT ANSWER
d) Tin
e) Zinc

302

Yassin Al-Safadi (Safadi92@hotmail.com)


• Comment on this Question

In typical dental amalgams, silver represents 67–74% of the total amalgam. The other metals are
tin (25–28%), copper (0 – 6%, although the percentage can be higher in copper-enriched
amalgams), zinc (0–2%) and mercury (0–3%).

34. How many root canals are typically present in an upper first molar tooth?
a) 1
b) 2
c) 3
d) 4 « CORRECT ANSWER
e) 5

i
ad
• Comment on this Question
af
-S
Around 69% of upper first molars have four canals. These consist of a large palatal canal, a
distobuccal canal and two mesiobuccal canals. In those upper first molars that have three
Al

canals, the second mesiobuccal canal is absent.


in

35. You have fitted a set of complete dentures (upper and lower) for a 70-year-old man.
After 2 weeks he returns to your surgery complaining of difficulty making ‘f’ and ‘v’
ss

sounds. What is the most likely reason for this problem?


a) Inadequate freeway space
Ya

b) Inappropriate position of incisal edges of upper teeth « CORRECT ANSWER


c) Occlusal error
d) Palate on upper complete denture is too thick
e) Upper molar teeth are set too far buccally

• Comment on this Question

Production of ‘f’ and ‘v’ sounds involves the use of the incisal edge of the upper teeth and the
lower lip. Therefore inappropriate tooth positioning can result in failure to enunciate these
sounds correctly.

303

Yassin Al-Safadi (Safadi92@hotmail.com)


36. When recording an impression for an upper complete denture, it is common to
compress the tissues in the area of the vibrating line to develop a peripheral seal.
Where is this vibrating line found?
a) At the junction between the hard and soft palate
b) At the junction between the non-mobile and mobile tissues of the soft palate «
CORRECT ANSWER
c) On the hard palate
d) On the mobile tissues of the soft palate
e) On the non-mobile tissues of the soft palate

• Comment on this Question

The tissues in the anterior 2 mm of the soft palate are non-mobile, and it is useful to compress

i
them (in the so-called ‘post-dam area’) to enhance retention. Extending the denture beyond the

ad
vibrating line means that it will be displaced by the mobile soft palate tissue, and the patient will
then complain that the denture is loose. Leaving the denture margin on the hard palate will
af
reduce the retention of the complete denture, and if the denture is designed to compress
-S
tissues in this region, the area will probably ulcerate.

37. When recording a master impression for an upper complete denture, which of the
Al

following impression materials is used with a ‘close-fitting’ (ie non-spaced) non-


perforated special tray?
in

a) Alginate
ss

b) Condensation-cured silicone
c) Impression plaster
Ya

d) Polyether
e) Zinc oxide and eugenol-based cements « CORRECT ANSWER

• Comment on this Question

Due to the relative viscosities of these impression materials, zinc oxide and eugenol-based
cements are the most suitable impression material. All of the other impression materials listed
would require spacing of the impression tray. Alginate usually requires tray perforation to
enhance retention. Impression plasters are rarely used nowadays, except occasionally as part of
a mucostatic technique (avoiding compression of the denture-bearing tissues), and spacing and
perforation are then indicated. Polyether is a stiff viscous material, and tray spacing is indicated.

304

Yassin Al-Safadi (Safadi92@hotmail.com)


Condensation-cured silicones require spacing, but these materials are rarely used nowadays as
significant contraction occurs during setting.

38. Removable partial dentures are retained using clasps. Which of the following factors
does not determine the flexibility of a clasp?
a) Clasp length
b) Clasp thickness
c) Cross-sectional design
d) Degree of horizontal undercut in which the clasp sits « CORRECT ANSWER
e) Material used in the clasp construction

• Comment on this Question

The following factors influence clasp flexibility.

i
ad
• Cross-sectional design: round clasps can flex in all directions, whereas a half-round clasp
will have reduced flexibility in the plane parallel to its shorter axis.
af
-S
• Clasp length: longer clasps are more flexible than shorter ones.

• Clasp thickness: thicker clasps are less flexible than thinner ones.
Al

• Material used: cobalt chromium clasps are stiffer than gold clasps.
in

The degree of horizontal undercut in which the clasp sits does not affect clasp flexibility. In fact
ss

it is the other way round, as clasps must be designed to be of sufficient flexibility to adapt
around necessary undercuts.
Ya

39.Which of the following statements about tooth surface loss is correct?


a) Abrasion is often related to parafunctional bruxism habits
b) Amalgam fillings are affected more by erosion than dentine
c) Attrition results in the formation of cup-shaped lesions on the incisal edges of the lower
incisors
d) Dietary causes of erosion can include water of neutral pH
e) Loss of tooth structure can lead to pulpal complications « CORRECT ANSWER

• Comment on this Question

305

Yassin Al-Safadi (Safadi92@hotmail.com)


• Abrasion is tooth wear caused by the action of external agents on the surface of teeth,
such as over-vigorous tooth brushing.

• Erosion is associated with frequent consumption of low pH carbonated drinks.

• Untreated tooth wear can lead to pulpal involvement and the need for root canal
treatment or extraction.

• Erosion causes loss of tooth tissue, such as dentine, whereas restorative materials such
as amalgam are more resistant to dissolution.

• Attrition (tooth-to-tooth wear) causes flattened lesions on the incisal edges of affected
teeth. Cup-shaped lesions are associated with erosion.

40. To which of the following anatomical structures should a root filling ideally be
extended?
a) Anatomical apex
b) Apical constriction « CORRECT ANSWER

i
ad
c) Apical foramen
d) Radiographic apex
e) None of the above
af
-S
Al

• Comment on this Question


in

A root filling should ideally be extended to the apical constriction. The latter is a narrowing
ss

within the root canal system approximately 0.5–1 mm away from the apical foramen. This is
regarded as the junction between the intracanal and extracanal tissues.
Ya

The radiographic apex is the radiographic representation of the anatomical apex of the tooth.
The apical foramen itself is usually 1 mm coronal to this on the lateral surface of the root. In
general, filling a root canal as far as the radiographic apex will mean that the canal is ‘overfilled’
(ie the root filling material is beyond the apical constriction).

41. When performing a re-root canal treatment of a tooth with a failed root canal
treatment, which of the following is the most appropriate irrigating solution to use?
a) Chlorhexidine « CORRECT ANSWER
b) EDTA
c) Local anaesthetic without adrenaline
d) Saline
e) Sodium hypochlorite

306

Yassin Al-Safadi (Safadi92@hotmail.com)


• Comment on this Question

Management of failed root canal treatments is problematic. Identification of the exact cause of
failure is often difficult, particularly when the radiographic appearance of the completed root
canal filling appears to be acceptable. Evidence shows that a substantial proportion of failed
root canal fillings are associated withEnterococcus faecalis. It has been demonstrated that this
organism is sensitive to chlorhexidine, which is therefore considered to be the most appropriate
irrigating solution for use during re-root canal treatment.

Neither local anaesthetic nor saline are particularly useful as root canal irrigants, even when
performing primary root canal treatments. Although these materials are useful lubricants, they
have little antibacterial action. Sodium hypochlorite is commonly used during primary root canal
treatments where, in addition to being a lubricant, it has tissue-dissolving and antibacterial
actions. However, its antibacterial action only lasts for about 15 minutes, so regular irrigation is
indicated. EDTA is not a root canal irrigant but a chelating agent, which is used for ‘softening’

i
ad
dentine and thus enabling its removal during canal shaping.

42. Which of the following intra-oral regions has the highest success rate for implant
af
placement?
-S
a) Anterior mandible « CORRECT ANSWER
b) Anterior maxilla
Al

c) Posterior mandible
d) Posterior maxilla
e) Upper premolar region
in
ss
Ya

• Comment on this Question

The highest success rates for implant placement are seen in the anterior mandible. This is
thought to be due to the fact that it contains a higher proportion of dense cortical bone relative
to softer cancellous bone. It is also relatively free from limiting anatomical structures, which are
found in other areas such as the floor of the nose, the maxillary sinus and the inferior dental
canal, and which limit the length of implants that can be placed, which in turn affects implant
success. The success rates for implant surgery can be arranged in descending order as follows:

• anterior mandible

• anterior maxilla

• posterior maxilla

307

Yassin Al-Safadi (Safadi92@hotmail.com)


• posterior mandible.

43. Which of the following impression materials has the greatest dimensional accuracy
when measured 3 days after being recorded in the dental surgery?
a) Addition-cured silicone « CORRECT ANSWER
b) Agar
c) Alginate
d) Condensation-cured silicone
e) Polysulphide

• Comment on this Question

Addition-cured silicone is the most dimensionally stable impression material in the above list.
Both condensation-cured silicone and polysulphide impression materials contract on setting,

i
ad
due to the loss of setting reaction by-products. Alginates and agar need to be poured quickly
after being removed from the mouth. As they are water based they can dry out rapidly and
af
contract (a process called syneresis) or take water up water and swell (a process called
imbibition) if left to stand or handled incorrectly.
-S
44. Which of the following materials has the greatest resistance to wear in the oral
Al

environment?
a) Compomers
in

b) Conventional glass ionomer cements


c) Resin composites « CORRECT ANSWER
ss

d) Resin-modified glass ionomer cements


e) All have equal wear resistance
Ya

• Comment on this Question

Due to the incorporation of resin polymers and the presence of fillers, resin composites offer
the greatest wear resistance. As the resin and filler content is highest in resin composites, lower
in compomers, lower still in resin-modified glass ionomer cements and lowest in conventional
glass ionomer cements, the wear resistance of these materials decreases in the same order,
being highest in resin composites and lowest in conventional glass ionomer cements.

308

Yassin Al-Safadi (Safadi92@hotmail.com)


45. A patient presents with a throbbing pain that can be localised to the lower right
quadrant. However, the patient cannot identify the tooth that is affected. What is the
most likely diagnosis?
a) Apical periodontitis
b) Atypical odontalgia
c) Cracked tooth syndrome
d) Irreversible pulpitis « CORRECT ANSWER
e) Reversible pulpitis

• Comment on this Question

• In irreversible pulpitis, the pulp has been damaged by invading bacteria. Typically a
throbbing pain is felt. However, the patient cannot identify the tooth affected, as there are no

i
proprioceptive fibres within the dental pulp. Proprioceptive fibres are found in the periapical

ad
tissues, and when the infection/pulpal damage spreads to this region, the patient will complain
of a throbbing pain and can identify the tooth involved.af
• Reversible pulpitis is associated with a sharp pain which is relieved once the irritant has
-S
been withdrawn. It is not possible to identify the affected tooth.
Al

• Cracked tooth syndrome is usually characterised by a sharp pain when biting on the
affected tooth, or when the biting pressure is released. It is often difficult to localise the affected
in

tooth.
ss

• Atypical odontalgia is characterised by atypical facial pain and an apparently normal


tooth (sometimes referred to as ‘phantom toothache’).
Ya

• Apical periodontitis is inflammation of the periodontium at the apex of the tooth,


normally caused by leakage of the products of pulp death into the surrounding tissues.

46. When preparing an upper central incisor tooth for a porcelain-fused-to-metal crown
with a labial porcelain margin, what is the appropriate marginal configuration?
a) A chamfer
b) A deep chamfer
c) A feather edge
d) An inverse bevel
e) A shoulder « CORRECT ANSWER

309

Yassin Al-Safadi (Safadi92@hotmail.com)


• Comment on this Question

Due to the constraints and demands of ceramic materials, a shoulder is the appropriate
marginal configuration for a labial porcelain margin on a porcelain-fused-to-metal crown. A
chamfer is an appropriate finish line for metal. A deep chamfer can be used for all ceramic
crowns. Feather edges were historically used with margins of some cast gold restorations, but
are rarely used nowadays. Inverse bevels are a design of surgical flap that is used with some
periodontal surgical techniques.

47. Which of the following is a characteristic of microfilled composites but not hybrid and
macrofilled composites?
a) Increased dimensional stability
b) Increased fracture resistance
c) Increased opacity
d) Increased surface smoothness « CORRECT ANSWER
e) Increased wear resistance

i
ad
af
• Comment on this Question
-S
Microfilled composites have a lower filler content than other composite materials. This results in
Al

microfilled composites having:

• reduced wear resistance


in
ss

• reduced opacity (ie they are more translucent)

• reduced fracture resistance


Ya

• reduced dimensional stability (they show increased dimensional contraction)

• a more polishable and smooth surface than other composites.

48. What is the definition of indirect retention?


a) Linking of the denture saddles
b) Resistance to horizontal forces on abutment teeth caused by retentive components
during removal of the denture
c) Resistance to horizontal forces on denture flanges
d) Resistance to movement of the denture towards the underlying ridge following the
application of axial loads
e) Resistance to rotational movements of the denture saddle away from the underlying
ridge around an axis of rotation« CORRECT ANSWER

310

Yassin Al-Safadi (Safadi92@hotmail.com)


• Comment on this Question

• Linking of the denture saddles describes the major connector.

• Reciprocation is defined as resistance to horizontal forces on abutment teeth caused by


retentive components during removal of the denture.

• Bracing is defined as resistance to horizontal forces on denture flanges.

• Support is defined as resistance to movement of the denture towards the underlying


ridge following the application of axial loads.

49. What is the most appropriate wavelength of the incident curing light for a resin
composite that contains camphorquinone as an activator?

i
a) 470 nm « CORRECT ANSWER

ad
b) 470 µm
c) 4.7 mm af
d) 470 mm
-S
e) None of the above
Al
in

• Comment on this Question


ss

Selection of this wavelength is dependent on the absorption spectrum of camphorquinone.


Ya

When light that includes this wavelength is used to irradiate unpolymerised resin composite
that contains camphorquinone (a photo-initiator), it will cause initiation (addition of a monomer
unit to the activated camphorquinone molecule), and the polymerisation reaction will
commence.

Some useful mathematical points with regard to the options listed above:

• 1 nm = 1 nanometre = 10-9 m

• 470 mm = 470 × 10-3 m, so this value is 1 000 000 times too large

• 470 µm = 470 micrometres = 470 × 10-6 m, so this value is 1000 times too large

• 4.7 mm = 470 × 10-5 m, so this value is 10 000 times too large.

50. Which one of the following statements regarding gingival recession is correct?

311

Yassin Al-Safadi (Safadi92@hotmail.com)


a) Epidemiological studies show that more than 50% of the population have one or more
sites with recession of at least 3 mm
b) In the UK, this is more common in woman
c) The aetiology never involves inflammation
d) The gingival biotype is an important determinant of gingival recession « CORRECT
ANSWER
e) The width of the attached gingiva is considered to be an important determinant of
gingival recession

• Comment on this Question

The gingival biotype is an important determinant of gingival recession. A thick gingival biotype
contains a thick layer of connective tissue that is robust and less likely to be undermined as a

i
consequence of inflammation and resultant recession. A thin gingival biotype is more prone to

ad
recession because of the vulnerability of thin connective tissue.
af
51. Complete the following statement, selecting one option from the list below:
-S
Surgical crown lengthening is indicated…
Al

a) for the surgical periodontal management of conical roots


b) if the clinical crown height cannot provide retention for a restoration « CORRECT
ANSWER
in

c) in cases of loss of bony support of over 6 mm


ss

d) in the presence of furcation defects


e) to increase the crown:root ratio
Ya

• Comment on this Question

Surgical crown lengthening procedures are indicated if the clinical crown height is insufficient to
provide adequate retention for a restoration. Thus, surgical crown lengthening procedures
facilitate the restoration of teeth with poor retentive properties.

52. Which one of the following statements regarding patient satisfaction with dentures is
correct?
a) Cost of dentures does not influence patient satisfaction with dentures
b) Denture quality positively influences patient satisfaction with dentures « CORRECT
ANSWER

312

Yassin Al-Safadi (Safadi92@hotmail.com)


c) Denture wearing experience negatively influences patient satisfaction with dentures
d) Increase in vertical dimension negatively influences patient satisfaction with dentures
e) Patient’s personality does not influence patient satisfaction with dentures

• Comment on this Question

Many studies have shown that dentures made to high quality result in increased patient
satisfaction with dentures.

53. Which of the following statements regarding the use of a dental bar as a major
connector is most likely to apply?
a) It facilitates indirect retention « CORRECT ANSWER
b) It facilitates patient adaptation to the appliance
c) It is the most aesthetic option of major connectors

i
ad
d) It is the most hygienic option of major connectors
e) It is useful when teeth are markedly inclined lingually
af
-S
Al

• Comment on this Question


in

The dental bar facilitates indirect retention, as it runs along the lingual surfaces of the teeth.
ss

54. Which of the following statements regarding periodontitis and systemic conditions is
most likely to be correct?
Ya

a) Atherosclerosis is a major risk factor for periodontitis


b) Diabetes is not a risk factor for periodontitis
c) There is no evidence of an association between oral health and respiratory disease
d) There is no increased risk of stroke in patients with periodontitis
e) There is potential for pathogens to gain access to the systemic circulation through an
ulcerated pocket wall.« CORRECT ANSWER

• Comment on this Question

Potential pathogens can gain access to the systemic circulation through an ulcerated
periodontal pocket wall. When they have entered the circulation they have the potential to
affect systemic conditions.

313

Yassin Al-Safadi (Safadi92@hotmail.com)


55. Which one of the following statements regarding subgingival debridement is most
applicable?
a) It should be confined to patients who are non-smokers
b) It should be confined to sites that are easy for the patient to access
c) It should be confined to sites that exhibit persistent bleeding on probing
d) It should be confined to sites that show pocket depth equal to or greater than 4 mm «
CORRECT ANSWER
e) It should be confined to sites with re-infection and suppuration

• Comment on this Question

It should be confined to sites that show pocket depth equal to or greater than 4 mm irrespective
of the presence of the other factors mentioned in the first, second, third and fifth options. The

i
statement of subgingival debridement to be confined to sites that are easy for the patient to

ad
access is clearly false as the procedure should be carried out at sites that are not easily
assessable by the patient, thus requiring professional intervention.
af
56. Which one of the following statements regarding powered toothbrushes is most likely
-S
to be correct?
a) All are superior to manual brushing
Al

b) All remove staining better than manual brushes


c) Rotation-oscillating types tend to remove more plaque than manual brushes « CORRECT
in

ANSWER
ss

d) Sonic brushes are more effective than rotation-oscillating brushes


e) Ultrasonic brushes are more effective than rotation-oscillating brushes
Ya

• Comment on this Question

Rotation-oscillating powered toothbrushes tend to remove more plaque than manual brushes.
This statement is supported by ample research findings.

57. A particulate graft material derived from bovine bone is an example of:
a) Allograft
b) Alloplast
c) Autograft
d) Osseograft
e) Xenograft « CORRECT ANSWER

314

Yassin Al-Safadi (Safadi92@hotmail.com)


• Comment on this Question

Definition of xenograft: a surgical graft of tissue from one species to an unlike species (or genus
or family). A graft from a bovine to a human is a xenograft. The prefix ‘xeno-’ means foreign. It
comes from the Greek word ‘xenos’ meaning stranger, guest or host. (Xeno- and xen- are variant
forms of the same prefix.)

58. Which one of the following statements regarding direct composite restorations for
treatment of toothwear is most appropriate?
a) Patients always complain of functional problems in relation to their occlusion
b) These are more wear resistant than ceramics
c) These increase costs because of long treatment times
d) These may restore aesthetics and function « CORRECT ANSWER

i
e) These must be used in at least 4 mm thickness

ad
af
-S
• Comment on this Question
Al

The application of chair-side composite resin restorative material to worn teeth allows the worn
teeth to be built up to their previous (unworn) shape. This regained shape restores the function
in

and appearance of the worn teeth.


ss

59.Which of the following statements applies to the Dahl concept?


a) It has been adapted to use bonded resin composite material
Ya

b) It is used to create space to restore worn anterior teeth


c) It uses the concept of intrusion of anterior teeth and eruption of posterior teeth «
CORRECT ANSWER
d) It uses the concept of intrusion of anterior teeth eruption of posterior teeth
e) Originally involved the use of a metal bite raising appliance

• Comment on this Question

The statement of intrusion of anterior teeth eruption of posterior teeth does not apply to the
Dahl concept, which is based on the concept of intrusion of anterior teeth and eruption of
posterior teeth.

315

Yassin Al-Safadi (Safadi92@hotmail.com)


i
ad
af
-S
Al
in
ss
Ya

316

Yassin Al-Safadi (Safadi92@hotmail.com)

You might also like